0% found this document useful (0 votes)
4K views128 pages

Sat Practice Test Master 5

Uploaded by

Hamza Waleed
Copyright
© © All Rights Reserved
We take content rights seriously. If you suspect this is your content, claim it here.
Available Formats
Download as PDF, TXT or read online on Scribd
0% found this document useful (0 votes)
4K views128 pages

Sat Practice Test Master 5

Uploaded by

Hamza Waleed
Copyright
© © All Rights Reserved
We take content rights seriously. If you suspect this is your content, claim it here.
Available Formats
Download as PDF, TXT or read online on Scribd
You are on page 1/ 128

PART VII

FIVE PRACTICE TESTS


CHAPTER 14 Preface to Practice Tests

PRACTICE TEST 1

PRACTICE TEST 2

PRACTICE TEST 3

PRACTICE TEST 4

PRACTICE TEST 5
Practice Test 1 493

practice test
SECTION 1: READING TEST
65 Minutes • 52 Questions

TURN TO SECTION 1 OF YOUR ANSWER SHEET TO ANSWER THE QUESTIONS IN THIS


SECTION.

Directions: Each passage (or pair of passages) in this section is followed by a number of multiple-
choice questions. After reading each passage, select the best answer to each question based on
what is stated or implied in the passage or passages and in any supplementary material, such as
a table, graph, chart, or photograph.

QUESTIONS 1–10 ARE BASED ON THE expected to cook, dutifully. The blushes
FOLLOWING PASSAGE. caused by the careless scrutiny of some
passengers as she had entered the car
The following passage has been taken from the 35 were strange to see upon this plain, under-
“The Bride Comes to Yellow Sky,” by Stephen class countenance, which was drawn in
Crane. It is a western short story that was first placid, almost emotionless lines.
published in 1898. The protagonist is Jack They were evidently very happy. “Ever
Potter, who returns to the town of Yellow Sky been in a parlor-car before?” he asked,
with his bride. 40 smiling with delight.
The great Pullman was whirling “No,” she answered; “I never was. It’s
onward with such dignity of motion that fine, ain’t it?”
a glance from the window seemed simply “Great! And then after a while we’ll
Line to prove that the plains of Texas were go forward to the diner, and get a big
5 pouring eastward. Vast flats of green 45 layout. Finest meal in the world. Charge
grass, dull-hued spaces of mesquite and a dollar.”
cactus, little groups of frame houses, “Oh, do they?” cried the bride.
woods of light and tender trees, all were “Charge a dollar? Why, that’s too much—
sweeping into the east, sweeping over the for us—ain’t it Jack?”
10 horizon, a precipice. 50 “Not this trip, anyhow,” he answered
A newly married pair had boarded this bravely. “We’re going to go the whole
coach at San Antonio. The man’s face was thing.”
reddened from many days in the wind and Later he explained to her about the
sun, and a direct result of his new black trains. “You see, it’s a thousand miles
15 clothes was that his brick-colored hands 55 from one end of Texas to the other; and
were constantly performing in a most this train runs right across it, and never
conscious fashion. From time to time he stops but for four times.” He had the
looked down respectfully at his attire. He pride of an owner. He pointed out to
sat with a hand on each knee, like a man her the dazzling fittings of the coach,
20 waiting in a barber’s shop. The glances he 60 and in truth her eyes opened wider as
devoted to other passengers were furtive she contemplated the sea-green figured
and shy. velvet, the shining brass, silver, and glass,
The bride was not pretty, nor was the wood that gleamed as darkly brilliant
she very young. She wore a dress of as the surface of a pool of oil. At one end
25 blue cashmere, with small reservations 65 a bronze figure sturdily held a support for
of velvet here and there, and with steel a separated chamber, and at convenient
buttons abounding. She continually places on the ceiling were frescoes in
twisted her head to regard her puff olive and silver.
sleeves, very stiff, straight, and high. To the minds of the pair, their
30 They embarrassed her. It was quite 70 surroundings reflected the glory of their
apparent that she had cooked, and that she marriage that morning in San Antonio.
GO TO THE
NEXT PAGE
www.petersons.com
494 PART VII: Five Practice Tests

This was the environment of their new 4 As used in line 2, “dignity” most nearly
estate; and the man’s face in particular means
Line beamed with an elation that made him
75 appear ridiculous to the negro porter. This (A) splendor.
individual at times surveyed them from (B) respectability.
afar with an amused and superior grin.
(C) superiority.
On other occasions he bullied them with
skill in ways that did not make it exactly (D) gracefulness.
80 plain to them that they were being bullied.
He subtly used all the manners of the 5 Jack and his bride might best be described
most unconquerable kind of snobbery. He as
oppressed them, but of this oppression
they had small knowledge, and they (A) firm and resolute in their decisions.
85 speedily forgot that infrequently a number (B) nervous and fearful about their trip.
of travelers covered them with stares of
(C) awkward and self-conscious in the
derisive enjoyment. Historically, there
setting.
was supposed to be something infinitely
humorous in their situation. (D) amazed and bewildered by the
landscape.
1 The passage might best be described as
6 Which choice provides the best evidence
(A) an analysis of a man’s acceptance of for the answer to the previous question?
his social status.
(A) Lines 5–10 (“Vast flats …
(B) an account of a couple’s anticipation
precipice.”)
of married life.
(B) Lines 17–22 (“From time … furtive
(C) a description of train travel in
and shy.)
nineteenth-century Texas.
(C) Lines 41–46 (“‘No,’ she … a
(D) a criticism of class consciousness in
dollar’.”)
the nineteenth century.
(D) Lines 69–75 (“To the … negro
porter”)
2 It can be inferred from the passage that
Jack
7 In lines 32–37 (“The blushes … emotion-
(A) wants to impress his new bride. less lines.”), why does the narrator note
(B) is likely a farmhand or rancher. that the bride’s blushing seemed so out of
place on her face?
(C) is used to being treated as an inferior.
(D) wants to change his station in life. (A) To emphasize the degree to which
other passengers are staring
3 Which choice provides the best evidence (B) To express the bride’s extreme
for the answer to the previous question? happiness with her marriage
(C) To communicate that the bride is
(A) Lines 17–20 (“From time … barber’s lacking in self-confidence
shop.”)
(D) To underscore that the bride is
(B) Lines 23–30 (“The bride … strikingly unattractive
embarrassed her.”)
(C) Lines 43–52 (“‘Great! … whole
thing.”)
(D) Lines 69–75 (“To the … negro
porter.”)

Master the New SAT®


Practice Test 1 495

practice test
8 As used in line 73, “estate” most nearly branch, not among the twigs, but on
means the body of the branch itself. Yet I have
known instances where it was attached
(A) project. by the side to an old moss-grown trunk;
(B) interests. 10 and others where it was fastened on a
strong rank stalk, or weed, in the garden;
(C) property.
but these cases are rare. In the woods it
(D) standing. very often chooses a white oak sapling to
build on; and in the orchard, or garden,
9 In lines 75–87 (“This individual … deri- 15 selects a pear tree for that purpose. The
sive enjoyment.”), the narrator maintains branch is seldom more than ten feet from
that the the ground. The nest is about an inch in
diameter, and as much in depth. A very
(A) porter mocks the couple, making Jack complete one is now lying before me,
and his bride believe they are being 20 and the materials of which it is composed
catered to. are as follows: —The outward coat is
(B) couple barely notices the formed of small pieces of bluish grey
contemptuous way they are treated lichen that vegetates on old trees and
by the porter. fences, thickly glued on with the saliva of
25 the bird, giving firmness and consistency
(C) porter is openly hostile to the couple,
to the whole, as well as keeping out
making the trip painful for them.
moisture. Within this are thick matted
(D) couple has become used to the rude layers of the fine wings of certain flying
behavior of the porter. seeds, closely laid together; and lastly, the
30 downy substance from the great mullein,
10 What is the main rhetorical effect of lines and from the stalks of the common fern,
14–18 (“and a … his attire”)? lines the whole. The base of the nest is
continued round the stem of the branch,
(A) To illustrate how nervous and to which it closely adheres; and, when
awkward the groom is 35 viewed from below, appears a mere
(B) To show how much the groom is mossy knot or accidental protuberance.
used to using his hands The eggs are two, pure white, and of
equal thickness at both ends. … On a
(C) To convey how unaccustomed the
person’s approaching their nest, the little
groom is to wearing dress clothes
40 proprietors dart around with a humming
(D) To show how happy the groom is sound, passing frequently within a few
about being married inches of one’s head; and should the
young be newly hatched, the female will
QUESTIONS 11–21 ARE BASED ON THE resume her place on the nest even while
FOLLOWING PASSAGE. 45 you stand within a yard or two of the
spot. The precise period of incubation I
The following passage has been taken from am unable to give; but the young are in
American Ornithology by Alexander Wilson, the habit, a short time before they leave
a Scottish-American naturalist. Dubbed “the the nest, of thrusting their bills into the
Father of American Ornithology,” Wilson is 50 mouths of their parents, and sucking
regarded as the greatest American ornithologist what they have brought them. I never
after Audubon. His nine-volume American Orni- could perceive that they carried them any
thology was published between 1808 and 1814. animal food; tho, from circumstances that
About the twenty-fifth of April will presently be mentioned, I think it
the Hummingbird usually arrives in 55 highly probable they do. As I have found
Pennsylvania; and about the tenth of May their nest with eggs so late as the twelfth
Line begins to build its nest. This is generally
of July, I do not doubt but that they
5 fixed on the upper side of a horizontal
GO TO THE
NEXT PAGE
www.petersons.com
496 PART VII: Five Practice Tests

frequently, and perhaps usually, raise two 11 The author is mostly concerned with
Line broods in the same season.
60 The hummingbird is extremely fond (A) describing the characteristics of the
of tubular flowers, and I have often stopt, hummingbird.
with pleasure, to observe his maneuvers (B) explaining how hummingbirds build
among the blossoms of the trumpet their nests.
flower. When arrived before a thicket of
(C) discussing the reasons hummingbirds
65 these that are full blown, he poises, or
are interesting.
suspends himself on wing, for the space
of two or three seconds, so steadily, that (D) interpreting the meaning of certain
his wings become invisible, or only like hummingbird behaviors.
a mist; and you can plainly distinguish
70 the pupil of his eye looking round with 12 Based on lines 1–4 (“About the twenty-
great quickness and circumspection; the fifth of April … to build its nest.”), it can
glossy golden green of his back, and generally be assumed that hummingbirds
the fire of his throat, dazzling in the
sun, form altogether a most interesting (A) take two weeks to build their nests.
75 appearance. The position into which his (B) migrate elsewhere for the winter.
body is usually thrown while in the act
(C) cannot be found in places farther
of thrusting his slender tubular tongue
north.
into the flower, to extract its sweets, is
exhibited in the figure on the plate. When (D) are mostly solitary animals.
80 he alights, which is frequently, he always
prefers the small dead twigs of a tree, or 13 As used in line 5, “fixed” most nearly
bush, where he dresses and arranges his means
plumage with great dexterity. His only
note is a single chirp, not louder than (A) adjusted.
85 that of a small cricket or grasshopper, (B) intended.
‘ generally uttered while passing from
(C) aligned.
flower to flower, or when engaged in fight
with his fellows; for when two males (D) fastened.
meet at the same bush, or flower, a battle
90 instantly takes place; and the combatants 14 In lines 18–36 (“A very complete one …
ascend in the air, chirping, darting and accidental protuberance.”), the author
circling around each other, till the eye describes the hummingbird nest in
is no longer able to follow them. The layers most likely to establish that the
conqueror, however, generally returns to hummingbird
95 the place, to reap the fruits of his victory.
I have seen him attack, and for a few (A) uses nearby plant debris in the nest.
moments tease the King-bird; and have (B) builds compact, complicated nests.
also seen him, in his turn, assaulted by a
(C) builds the nest over a period of time.
humble-bee, which he soon put to flight.
100 He is one of those few birds that are (D) constructs a nest that is waterproof.
universally beloved; and amidst the sweet
dewy serenity of a summer’s morning,
his appearance among the arbours of
honeysuckles, and beds of flowers, is
105 truly interesting.

Master the New SAT®


Practice Test 1 497

practice test
15 It can be inferred from the passage that 19 In lines 93–95 (“The conqueror … fruits
hummingbirds of his victory.”), “fruits of his victory”
refers to
(A) stay in the nest for a short period of
time. (A) the dead twigs of a tree.
(B) are fiercely protective of their eggs (B) a female hummingbird.
and young. (C) the nectar of a flower.
(C) have a sweet, though notably quiet, (D) the other combatant.
song.
(D) feed only on the sweet nectar of 20 Which choice provides the best evidence
flowers.
for the answer to the previous question?

16 Which choice provides the best evidence (A) Lines 1–4 (“About the twenty-
for the answer to the previous question? fifth … its nest.”)
(B) Lines 75–79 (“The position … on the
(A) Lines 38–46 (“On a person’s … the plate.”)
spot.”)
(C) Lines 79–83 (“When he alights, …
(B) Lines 46–51 (“The precise … dexterity.”)
brought them.”)
(D) Lines 90–93 (“and the combatants …
(C) Lines 51–55 (“I never … they do.”) follow them.”)
(D) Lines 83–88 (“His only … with his
fellows;”) 21 The author most likely references both the
kingbird and the bee in lines 96–99 (“I
17 As it is used in line 82, “dresses” most have seen him … put to flight.”) to
nearly means
(A) highlight the many dangers that
(A) oils. confront hummingbirds.
(B) clothes. (B) describe how the hummingbird stays
(C) shuffles. close to its nest.
(D) smooths. (C) show that the hummingbird’s size
does not limit its ability to defend
itself.
18 In lines 68–69, the author notes that the
(D) emphasize that the hummingbird is
hummingbird’s “wings become invisible,
an especially aggressive bird.
or only like a mist” to

(A) show how transparent the wings are. QUESTIONS 22–31 ARE BASED ON
(B) emphasize how fast the wings are THE FOLLOWING PASSAGE AND
moving. SUPPLEMENTARY MATERIAL.
(C) point out that the sun reflects off the The passage is excerpted from information pro-
wings. vided by the National Oceanic and Atmospheric
(D) reiterate that the hummingbird is Administration (NOAA) at http://www.noaa.gov.
beautiful. Over half a mile taller … than Mt.
Everest, Mauna Kea in Hawai’i is more
than 6 miles tall, from its base on the
Line ocean floor to its summit two miles above
5 the surface of the Pacific Ocean. This
island mountain is only one of many
features found on the ocean floor. Besides
GO TO THE
NEXT PAGE
www.petersons.com
498 PART VII: Five Practice Tests

being the base for islands, the ocean floor of the last century, organisms in this deep,
Line also includes continental shelves and dark ecosystem rely on chemicals and
10 slopes, canyons, oceanic ridges, trenches, a process called chemosynthesis as the
fracture zones, abyssal hills, abyssal base of their food web and not on sunlight
plains, volcanoes, and seamounts. Not 60 and photosynthesis as in other previously
just rock and mud, these locations are the described ecosystems ….
sites of exotic ecosystems that have rarely Hydrothermal vents form along mid-
15 been seen or even explored. ocean ridges, in places where the sea
floor moves apart very slowly (6 to 18
Plate Tectonics and the Ocean Floor
65 cm per year) as magma wells up from
The shape of the ocean floor, its
below. (This is the engine that drives
bathymetry, is largely a result of a process
Earth’s tectonic plates apart, moving
called plate tectonics. The outer rocky
continents and causing volcanic eruptions
layer of the Earth includes about a dozen
and earthquakes.) When cold ocean water
20 large sections called tectonic plates that
70 seeps through cracks in the sea floor
are arranged like a spherical jigsaw
to hot spots below, hydrothermal vents
puzzle floating on top of the Earth’s hot
belch a mineral-rich broth of scalding
flowing mantle. Convection currents in
water. Sometimes, in very hot vents,
the molten mantle layer cause the plates
the emerging fluid turns black, creating
25 to slowly move about the Earth a few
75 a “black smoker,” because dissolved
centimeters each year. Many ocean floor
sulfides of metals (iron, copper, and
features are a result of the interactions
several heavy metals) instantaneously
that occur at the edges of these plates.
precipitate out of solution when they mix
The shifting plates may collide
with the cold surrounding seawater.
30 (converge), move away (diverge), or slide
80 Unlike plants that rely on sunlight,
past (transform) each other. As plates
bacteria living in and around the dark
converge, one plate may dive under the
vents extract their energy from hydrogen
other, causing earthquakes, forming
sulfide (HS) and other molecules that
volcanoes, or creating deep ocean
billow out of the seafloor. Just like plants,
35 trenches such as the Mariana Trench.
85 the bacteria use their energy to build
Where plates are pulled away (diverge)
sugars out of carbon dioxide and water.
from each other, molten magma flows
Sugars then provide fuel and raw material
upward between the plates, forming
for the rest of the microbes’ activities.
mid-ocean ridges, underwater volcanoes,
40 hydrothermal vents, and new ocean floor Why is chemosynthesis important?
crust. The Mid-Atlantic Ridge is an Chemosynthetic deep-sea bacteria
example of this type of plate boundary …. 90 form the base of a varied food web that
includes shrimp, tubeworms, clams, fish,
Marine Life and Exploration on the Ocean
crabs, and octopi. All of these animals
Floor
must be adapted to endure the extreme
Over the last decade, more than 1500
environment of the vents—complete
new species have been discovered in the
95 darkness; water temperatures ranging
45 ocean by marine biologists and other
from 2°C (in ambient seawater) to about
ocean scientists. Many of these newly
400°C (at the vent openings); pressures
discovered species live deep on the
hundreds of times that at sea level; and
ocean floor in unique habitats dependent
high concentrations of sulfides and other
on processes resulting from plate
100 noxious chemicals.
50 movement, underwater volcanoes, and
cold water seeps. The discovery of deep Why is photosynthesis important?
ocean hydrothermal vent ecosystems in Aquatic and terrestrial plants form the
1977 forced scientists to redefine living base of varied food webs that may include
systems on our planet. Considered one of small fish and crabs, larger fish, and
55 the most important scientific discoveries eventually, humans.

Master the New SAT®


Practice Test 1 499

practice test
plume
of black 100 m
‘smoke’

black
smoker
chimneys hydrothermal fluids
containing enter ocean at
sulphides up to 350°C cold
or more seawater
cold (typically
seawater 2–4°C)
metalliferous sulphides
sediment deposited in cracks
and veins
(stockwork)
oceanic
crust

high permeability
seawater

leaching of seawater
metal ions low permeability
from rock
high-temperature
reaction zone

heat from magma

22 Which best describes the function of 23 The author uses the simile “like a spherical
the opening sentence (“Over half a mile jigsaw puzzle” (lines 21–22) to illustrate
taller … the Pacific Ocean.”)? that

(A) It shows how unusual and (A) each plate plays a critical role.
extraordinary some ocean features (B) the earth is sphere-shaped.
are.
(C) each plate is asymmetrical.
(B) It emphasizes that Mt. Everest is not
(D) the plates fit together.
the world’s tallest mountain.
(C) It describes the geography of island
24 The movement of the earth’s tectonic
mountains in the Pacific Ocean.
plates is a function of
(D) It compares the geography of Mt.
Everest to that of Mauna Kea. (A) earthquakes and volcanoes.
(B) the moving ocean currents.
(C) new ocean floor crust.
(D) the moving molten mantle layer.

GO TO THE
NEXT PAGE
www.petersons.com
500 PART VII: Five Practice Tests

25 Which inference can you correctly make 28 The author devotes the first half of the pas-
about the discovery of deep ocean hydro- sage to an explanation of plate tectonics in
thermal vent ecosystems in 1977? order to

(A) Before the discovery, scientists (A) describe the geography of the ocean
thought that all living systems relied floor.
on photosynthesis as the basis of their (B) explain the conditions that create
food web. hydrothermal vents.
(B) Before the discovery, scientists (C) compare hydrothermal vents to
believed that deep ocean underwater volcanoes.
hydrothermal vents were caused by
(D) argue that hydrothermal vents are a
colliding plates.
unique ecosystem.
(C) After the discovery, scientists
determined that some sea life can
29 Which choice provides the best evidence
move easily between shallow waters
and deep sea environments. for the answer to the previous question?
(D) After the discovery, scientists sought (A) Lines 8–12 (“Besides being … and
ways to mine the minerals that spew seamounts.”)
from the hydrothermal vents.
(B) Lines 46–51 (“Many of … water
seeps.”)
26 Which choice provides the best evidence
(C) Lines 62–66 (“Hydrothermal
for the answer to the previous question? vents … from below.”)
(A) Lines 43–46 (“Over the … ocean (D) Lines 66–69 (“This is the engine …
scientists.”) and earthquakes.”)
(B) Lines 54–61 (“Considered one …
described ecosystems.”) 30 As used in line 100, “noxious” most nearly
(C) Lines 62–66 (“Hydrothermal means
vents … from below.”)
(A) harmful.
(D) Lines 73–79 (“Sometimes, in …
(B) annoying.
surrounding seawater.”)
(C) offensive.
27 As it is used in line 84, “billow” most (D) unusual.
nearly means
31 Based on the diagram, what eventually
(A) swell. happens to the dissolved sulfides of metals
(B) blow. that belch out of the black smoker?
(C) break.
(A) They are carried by ocean currents.
(D) spout.
(B) They are deposited on the sea bed.
(C) They rise to the surface of the sea.
(D) They drop back into the black
smoker.

Master the New SAT®


Practice Test 1 501

practice test
QUESTIONS 32–42 ARE BASED ON 45 instinct of beauty, of love for the ideal of
THE FOLLOWING PASSAGE AND which his life has no embodiment, let him
put the matter to the test. Let him take
SUPPLEMENTARY MATERIAL.
into a tenement block a handful of flowers
The passage is an excerpt from “How the Other from the fields and watch the brightened
Half Lives,” by Jacob Riis. It was published in 50 faces, the sudden abandonment of play
1890 and documented the squalid living condi- and fight that go ever hand in hand
tions in the tenements of New York City. where there is no elbow-room, the wild
The old question, what to do with the entreaty for “posies,” the eager love with
boy, assumes a new and serious phase in which the little messengers of peace
the tenements. Under the best conditions 55 are shielded, once possessed; then let
Line found there, it is not easily answered. In him change his mind. I have seen an
5 nine cases out of ten he would make an armful of daisies keep the peace of a
excellent mechanic, if trained early to block better than a policeman and his
work at a trade, for he is neither dull nor club, seen instincts awaken under their
slow, but the short-sighted despotism of 60 gentle appeal, whose very existence the
the trades unions has practically closed soil in which they grew made seem a
10 that avenue to him. Trade-schools, mockery ….
however excellent, cannot supply the Yet, as I knew, that dismal alley with
opportunity thus denied him, and at the its bare brick walls, between which
outset the boy stands condemned by his 65 no sun ever rose or set, was the world
own to low and ill-paid drudgery, held of those children. It filled their young
15 down by the hand that of all should labor lives. Probably not one of them had ever
to raise him. been out of the sight of it. They were
Home, the greatest factor of all in the too dirty, too ragged, and too generally
training of the young, means nothing to 70 disreputable, too well hidden in their slum
him but a pigeon-hole in a coop along besides, to come into line with the Fresh
20 with so many other human animals. Its Air summer boarders.
influence is scarcely of the elevating With such human instincts and
kind, if it have any. The very games cravings, forever unsatisfied, turned into
at which he takes a hand in the street 75 a haunting curse; with appetite ground
become polluting in its atmosphere. to keenest edge by a hunger that is never
25 With no steady hand to guide him, the fed, the children of the poor grow up
boy takes naturally to idle ways. Caught in joyless homes to lives of wearisome
in the street by the truant officer, or by toil that claims them at an age when the
the agents of the Children’s Societies, 80 play of their happier fellows has but just
peddling, perhaps, or begging, to help out begun. Has a yard of turf been laid and
30 the family resources, he runs the risk of a vine been coaxed to grow within their
being sent to a reformatory, where contact reach, they are banished and barred out
with vicious boys older than himself from it as from a heaven that is not for
soon develop the latent possibilities for 85 such as they. I came upon a couple of
evil that lie hidden in him. The city has youngsters in a Mulberry Street yard a
35 no Truant Home in which to keep him, while ago that were chalking on the fence
and all efforts of the children’s friends to their first lesson in “writin’.”
enforce school attendance are paralyzed And this is what they wrote: “Keeb of
by this want. The risk of the reformatory 90 te Grass.” They had it by heart, for there
is too great. What is done in the end is to was not, I verily believe, a green sod
40 let him take chances—with the chances within a quarter of a mile. Home to them
all against him. The result is the rough is an empty name.
young savage, familiar from the street.
Rough as he is, if any one doubt that this
child of common clay have in him the
GO TO THE
NEXT PAGE
www.petersons.com
502 PART VII: Five Practice Tests

32 The passage is mainly concerned with 35 In line 19, the author uses the metaphor
establishing that of the pigeon coop to establish that the
tenement
(A) there are very few job opportunities
available for poor people. (A) is as filthy as a cage for animals.
(B) the condition of the tenements (B) provides little privacy for its tenants.
condemns children to lives of misery. (C) is the only housing available for the
(C) more education is needed to help boy.
elevate the children of the poor. (D) provides no positive training or
(D) children who live in poverty naturally education.
turn to crime to support themselves.
36 As it is used in line 26, “idle” most nearly
33 As it is used in line 24, “polluting” most means
nearly means
(A) unproductive.
(A) dirty. (B) vain.
(B) dangerous. (C) immature.
(C) corrupting. (D) inactive.
(D) rowdy.
37 The author suggests in lines 34–41 that a
34 In lines 13–14, “condemned by his own” truant home
means that the boy is
(A) is no different from a reformatory.
(A) denounced because of his character. (B) does not enforce school attendance.
(B) criticized by his family members. (C) is full of vicious young boys.
(C) held back by his own community. (D) is necessary but unavailable.
(D) ridiculed by teachers in his school.

Master the New SAT®


Practice Test 1 503

practice test
38 The author suggests taking flowers to the 42 Which choice provides the best evidence
tenement child to show that for the answer to the previous question?

(A) he has the capacity to appreciate (A) Lines 81–85 (“Has a … as they.”)
exquisite things. B) Lines 85–88 (“I came … in writin’.”)
(B) he is always bored and will play with C) Lines 89–90 (“And this … te
anything. Grass.”)
(C) there are few flowers growing in the D) Lines 92–93 (“Home to … empty
tenements. name.”)
(D) the policemen patrolling the
tenements are brutal. QUESTIONS 43–52 ARE BASED ON THE
FOLLOWING TWO PASSAGES.
39 Which choice provides the best evidence
Passage 1 is a letter written by Robert Schumann
for the answer to the previous question? to Clara Wieck in 1838.
(A) Lines 41–42 (“The result … the Passage 2 is a letter from Napoleon Bonaparte
street.”) to his wife Josephine, written in 1796.
(B) Lines 44–47 (“Rough as … the test.”) Passage 1—Robert Schumann to Clara
(C) Lines 56–62 (“I have … a mockery.”) Wieck (1838)
(D) Lines 63–66 (“Yet as … those I have a hundred things to write to
children.”) you, great and small, if only I could do
it neatly, but my writing grows more and
Line more indistinct, a sign, I fear, of heart
40 The author’s photos of tenement children
5 weakness. There are terrible hours when
best support which of the following ideas your image forsakes me, when I wonder
from his argument? anxiously whether I have ordered my
life as wisely as I might, whether I had
(A) Tenement children never get to enjoy
any right to bind you to me, my angel,
childhood.
10 or can really make you as happy as I
(B) Tenement children are always hungry should wish. These doubts all arise, I
and malnourished. am inclined to think, from your father’s
(C) Tenement children are subject to attitude towards me. It is so easy to accept
police brutality. other people’s estimate of oneself. Your
(D) Tenement children learn nothing at 15 father’s behaviour makes me ask myself
home. if I am really so bad—of such humble
standing—as to invite such treatment
from anyone. Accustomed to easy victory
41 The author provides the story of the “wri-
over difficulties, to the smiles of fortune,
tin’” in the last paragraph to show that 20 and to affection, I have been spoiled by
tenement children having things made too easy for me,
(A) do not know how to spell. and now I have to face refusal, insult,
and calumny. I have read of many such
(B) are in need of good schools. things in novels, but I thought too highly
(C) have no access to green spaces. 25 of myself to imagine I could ever be the
(D) are prone to vandalism. hero of a family tragedy of the Kotzebue
sort myself. If I had ever done your father
an injury, he might well hate me; but I
cannot see why he should despise me and,
30 as you say, hate me without any reason.
But my turn will come, and I will then

GO TO THE
NEXT PAGE
www.petersons.com
504 PART VII: Five Practice Tests

show him how I love you and himself; my love, that vous, those four days make
for I will tell you, as a secret, that I really me long for my former indifference. Woe
Line love and respect your father for his many to the person responsible! May he, as
35 great and fine qualities, as no one but punishment and penalty, experience what
yourself can do. I have a natural inborn 85 my convictions and the evidence (which
devotion and reverence for him, as for is in your friend’s favour) would make
all strong characters, and it makes his me experience! Hell has no torments
antipathy for me doubly painful. Well, he great enough! Vous! Vous! Ah! How will
40 may some time declare peace, and say to things stand in two weeks? … My spirit
us, “Take each other, then.” 90 is heavy; my heart is fettered and I am
You cannot think how your letter has terrified by my fantasies. … You love me
raised and strengthened me. … You are less; but you will get over the loss. One
splendid, and I have much more reason day you will love me no longer; at least
45 to be proud of you than of me. I have tell me; then I shall know how I have
made up my mind, though, to read all 95 come to deserve this misfortune.  …
your wishes in your face. Then you will
think, even though you don’t say it, that 43 Both passages are primarily concerned
your Robert is a really good sort, that
with the subject of
50 he is entirely yours, and loves you more
than words can say. You shall indeed have (A) jealousy.
cause to think so in the happy future. I
(B) commitment.
still see you as you looked in your little
cap that last evening. I still hear you call (C) work.
55 me du. Clara, I heard nothing of what you (D) being apart.
said but that du. Don’t you remember?
Passage 2—Napoleon Bonaparte to 44 As it is used in line 39, “antipathy” most
Josephine Bonaparte (1796) nearly means
I have not spent a day without loving
you; I have not spent a night without (A) indifference.
embracing you; I have not so much as (B) mistrust.
60 drunk a single cup of tea without cursing (C) rudeness.
the pride and ambition which force me to
(D) dislike.
remain separated from the moving spirit
of my life. In the midst of my duties,
whether I am at the head of my army 45 We can infer from Passage 1 that Clara’s
65 or inspecting the camps, my beloved father’s feelings about Robert have caused
Josephine stands alone in my heart, Robert to
occupies my mind, fills my thoughts. If
I am moving away from you with the (A) dislike Clara’s father.
speed of the Rhône torrent, it is only that (B) question his and Clara’s future.
70 I may see you again more quickly. If I (C) doubt Clara’s love for him.
rise to work in the middle of the night, it
(D) question Clara’s character.
is because this may hasten by a matter of
days the arrival of my sweet love. Yet in
your letter of the 23rd and 26th Ventôse,
75 you call me vous. Vous yourself! Ah!
wretch, how could you have written this
letter? How cold it is! And then there are
those four days between the 23rd and
the 26th; what were you doing that you
80 failed to write to your husband? … Ah,

Master the New SAT®


Practice Test 1 505

practice test
46 Which choice provides the best evidence 50 We can infer that Clara’s letter to Robert
for the answer to the previous question? was different from Josephine’s letter to
Napoleon in that Clara’s letter
(A) Lines 5–13 (“There are … towards
me.”) (A) made Robert doubt himself.
(B) Lines 14–18 (“Your father’s … from (B) made Robert question her loyalty.
anyone.”) (C) left Robert feeling encouraged.
(C) Lines 31–36 (“But my … can do.”) (D) left Robert feeling confused.
(D) Lines 47–51 (“Then you … can say.”)
51 In each passage, the author recollects
47 Napoleon’s letter to Josephine suggests something their beloved said. But the ef-
that he thinks she fect of Josephine’s words (lines 73–77)
differ from the effect of Clara’s words
(A) is tired of his long absence. (lines 54–56) in that
(B) has little to occupy her time.
(A) Josephine’s words have hurt
(C) may be having an affair.
Napoleon.
(D) regrets her marriage to him.
(B) Clara’s words have offended Robert.
(C) Josephine’s words have calmed
48 Which choice provides the best evidence
Napoleon.
for the answer to the previous question?
(D) Clara’s words have humbled Robert.
(A) Lines 59–63 (“I have … my life.”)
(B) Lines 73–77 (“Yet in … this letter?”) 52 We can infer that both Robert and
(C) Lines 82–87 (“Woe to … me Napoleon
experience!”)
(A) are modest people.
(D) Lines 92–95 (“One day … this
(B) anger quickly.
misfortune.”)
(C) tend to be indecisive.
49 As it is used in line 72, “hasten” most (D) suffer from self-doubt.
nearly means

(A) stretch out.


(B) speed up.
(C) force.
(D) cause.

STOP
If you finish before time is called, you may check your work on this section only.
Do not turn to any other section.

www.petersons.com
Practice Test 1 539

answers PRACTICE TEST 1
ANSWER KEY AND EXPLANATIONS

Section 1: Reading Test

1. B 12. B 23. D 34. C 45. B


2. A 13. D 24. D 35. D 46. A
3. C 14. B 25. A 36. A 47. C
4. D 15. B 26. B 37. D 48. C
5. C 16. A 27. A 38. A 49. B
6. B 17. D 28. B 39. B 50. C
7. A 18. B 29. C 40. A 51. A
8. D 19. C 30. B 41. C 52. D
9. B 20. B 31. B 42. A
10. C 21. C 32. B 43. B
11. A 22. A 33. C 44. D

READING TEST RAW SCORE


(Number of correct answers)

1. The correct answer is (B). The narrator’s Choice (B) is misleading, as Jack’s face is
description of Jack and his bride, of their “reddened from many days in the wind and
wonder at their surroundings and happi- sun,” suggesting that he works outside but
ness at their marriage, makes choice (B) not necessarily as a farmhand or rancher.
the correct answer. Choice (A) is incorrect Choice (C) is incorrect because, while the
because though Jack seems awkward in his couple seem to shrug off the stares of other
surroundings, he does not seem aware of passengers, readers get the distinct impres-
his lower social standing compared to other sion that they are not used to being around
passengers. Though the passage provides a people of a higher social standing. Choice
great deal of insight about the people who (D) is not correct because Jack does not sug-
traveled on trains and the luxury in which gest through deeds or words that he wants
they traveled, choice (C) is incorrect because more than he has.
the primary focus is on the newly married 3. The correct answer is (C). Jack’s enthusi-
couple and their interactions. While Jack astic tone in responding to his bride’s delight
and his bride do seem self-conscious on (“Great!”) and his “brave” assurance that
the train, and while other passengers seem they are “going to go the whole thing” when
to regard them with derision, the passage she questions if they can afford the dining car
does not delve deeply into issue of class make choice (C) the correct answer. While
and class consciousness, so choice (D) is choice (A) suggests that Jack is aware of
also incorrect. his formal clothes and may want to make
2. The correct answer is (A). The couple’s a good impression, it is not clear that he
trip on the opulent train, as well as specific wants to impress his bride with his appear-
luxuries that Jack points out to his bride, ance. Therefore, choice (A) is incorrect.
makes choice (A) the correct answer. Because it involves the bride and the other

www.petersons.com
540 PART VII: Five Practice Tests

passengers’ perceptions of her, choice (B) 7. The correct answer is (A). The narra-
is also incorrect. Though (D) suggests that tor notes the other passengers’ “careless
the couple views their marriage favorably, scrutiny” and emphasizes that the bride’s
there is no indication in these lines that Jack countenance is “emotionless.” The inference
wants to impress his bride. is that the other passengers were staring
4. The correct answer is (D). In the first to such a degree that the bride’s usually
sentence, the phrase “whirling onward “placid” face flushed red with embarrass-
with such dignity of motion that a glance ment, making choice (A) correct. While
from the window seemed simply to prove the narrator notes in the following sentence
that the plains of Texas were pouring east- that the pair are “very happy,” there is no
ward” suggests that the train was moving evidence to suggest that she is blushing
very smoothly, so choice (D) is the correct from happiness, so choice (B) is not correct.
answer as “gracefulness” suggests a lack While the “careless scrutiny” likely led to
of turbulence. Choices (A) and (C) are self-consciousness, there is no evidence to
incorrect because, while they may suggest support the idea that the bride is generally
that the ride is better than expected, they lacking in self-confidence. Therefore, choice
do not describe the smooth movement that (C) is incorrect. Choice (D) is misleading,
the sentence emphasizes. Choice (B) is a because the narrator only notes that the bride
synonym of “dignity,” but it does not suit is plain, not unattractive.
the context of the sentence. 8. The correct answer is (D). In the preced-
5. The correct answer is (C). The expres- ing sentence the narrator notes that “their
sions of both Jack and his bride, as well surroundings reflected the glory of their
as the intimation that they are not wholly marriage that morning in San Antonio.”
comfortable in their attire, make choice (C) The glory of their new estate, then, refers
correct. The pair seems happy about their to their new standing as a married couple,
marriage, but there is no suggestion that they so choice (D) is correct. While “estate”
are resolute about their decision to marry or can refer to social standing, in this case the
about any other decision, so choice (A) is couple’s newly married status helps to give
incorrect. Choice (B) is a distortion because, context to the word, so choice (A) is incor-
while they seem slightly nervous, there is rect. Choice (B) is off-base, as “interests”
no evidence to suggest that they are nervous do not fit the context. While “estate” does
about the trip itself. Choice (D) is incorrect refer to property, the context of the sentence
because, although they are amazed at the does not support this definition of the word.
train’s interior, they do not seem to notice Therefore, choice (C) is incorrect.
the landscape. 9. The correct answer is (B). The narrator
6. The correct answer is (B). The couple’s notes that the couple “had small knowledge”
awkwardness and self-consciousness is of the porter’s bullying, so choice (B) is
best supported by Jack’s nervous and shy correct. There is no indication they are
countenance, choice (B). The description of being openly mocked, nor that the couple
the landscape is a distraction and does not feels catered to, so choice (A) is incorrect.
support the inference that the couple feels The narrator describes the couple’s enjoy-
self-conscious, so choice (A) is incorrect. ment and does not suggest that anyone was
Choice (C) is off-base because it suggests openly hostile to them, thus choice (C) is not
the bride is concerned about the cost, rather correct. The couple “had small knowledge”
than how the couple might appear to others. of the porter’s bullying, so choice (D) can-
Choice (D) is incorrect because the narra- not be correct because they could not have
tor’s likening the state of their marriage to become used to behavior they had small
their surroundings does not suggest that they knowledge of.
were either comfortable or uncomfortable 10. The correct answer is (C). Because the
in their surroundings. author describes the new black suit as the
cause when he says the groom’s “brick-

Master the New SAT®


Practice Test 1 541

answers PRACTICE TEST 1
colored hands were constantly performing (D) is correct. Because neither choice (A)
in a most conscious fashion,” choice (C) nor choice (B) make sense in the context
is the correct answer. Although the groom of the sentence, they are incorrect. While
is nervous in general, here the focus is on choice (C) makes sense in the context, it
how his clothes have led to certain hand does not make sense when the synonym
gestures, so choice (A) is incorrect. Choice (“attached”) in line 8 is taken into account,
(B) is off-base because although the author and so it is incorrect.
brings up the weathered condition of the 14. The correct answer is (B). The author notes
groom’s hands, he uses it as a contrast to the materials in each layer and uses phrases
the new clothes; the reader can assume like “firmness and consistency” and “closely
that the groom’s hands are clean. (D) is laid together,” making choice (B) the correct
incorrect because the groom’s happiness is answer. While the author does cite a number
not described in this part of the story, and of different plants that go into the nest, the
when he expresses happiness, it is with a way in which the hummingbird layers the
natural openness. For example, he smiles materials and fuses them to the nest is the
with delight or beams with elation. focus, so choice (A) is incorrect. Though
11. The correct answer is (A). The passage it might be assumed that such an intricate
covers nest building, feeding, and other nest takes time to build, the author does not
behaviors, so choice (A) is correct. Choice note time as a factor, so choice (C) is not
(B) is incorrect, because the passage does correct. While the author notes that the bird’s
not deal exclusively or mostly with nest saliva helps keep out moisture, there is no
building. Choice (C) is also incorrect be- mention of this characteristic as the author
cause while the author notes that the hum- describes the other layers, so choice (D) is
mingbird is interesting, it is not an argument also incorrect.
the author makes throughout the passage. 15. The correct answer is (B). The author notes
Choice (D) is also incorrect because the the female’s reaction when a person comes
author does not, for example, explain why near, which makes choice (B) correct. There
the hummingbird builds nests where it does, is no mention of how long hummingbird
but rather describes its behavior. chicks stay in the nest, so choice (A) is
12. The correct answer is (B). By noting that incorrect. Though the author mentions that
the birds arrive “about the twenty-fifth of hummingbird chirps are “not louder than
April,” the author is implying that hum- that of a small cricket or grasshopper,” it is
mingbirds are elsewhere before that. There also noted that a hummingbird’s “only note
is a certain presumption that the reader is a single chirp,” making choice (C) incor-
understands that many birds migrate for rect. The author notes that the females most
the winter, thus choice (B) is the correct likely carry “animal food” to their young,
answer. The time span the author gives so choice (D) is also incorrect.
is between the time the birds arrive and 16. The correct answer is (A). The author notes
the time they begin building their nest, so the female hummingbird’s reaction when
choice (A) is incorrect. There is no indica- a person comes near, inferring her fierce
tion that some birds do not migrate farther protection of her eggs and nest. So, choice
north of Pennsylvania, so choice (C) is not (A) is the correct answer. Choice (B) refers
a correct assumption. By referring to “the to the duration that chicks are in the nest,
Hummingbird,” the author is not referring and thus does not support the inference.
to a singular one, but rather to those species Choice (C) refers to what hummingbirds eat,
that migrate to Pennsylvania. Therefore, and it also does not support the inference.
choice (D) is also incorrect. Choice (D) is incorrect because it includes
13. The correct answer is (D). The author is details about a hummingbird’s voice and
describing how the nest is stuck to the branch does not support the idea that the female
and provides a context clue in the form of guards her young.
a synonym (“attached”) in line 8, so choice

www.petersons.com
542 PART VII: Five Practice Tests

17. The correct answer is (D). Choice (D) is 20. The correct answer is (B). The author’s
correct because the bird is putting its feath- description of the hummingbird extracting
ers in order, as supported by the word “ar- nectar from a flower serves to introduce
range.” Choice (A) is incorrect because no the idea that hummingbirds feed on flower
oil or supporting words are mentioned in the nectar (Lines 75–79), thus supporting the
passage. Choice (B) is misleading because notion that the two males are fighting over
in this instance, the author doesn’t compare a food source (the flower). So choice (B) is
the bird’s plumage to clothes. Choice (C) correct. While the author begins the passage
would be a possibility except that the author with a description of the nesting habits of fe-
describes the dexterity of the bird and uses males, nesting and mating are not discussed
the word “arrange,” which implies precision. in the second paragraph, so choice (A) is
Shuffle is a word commonly associated with incorrect. The reference to dead twigs is
clumsiness. not to any specific territory, making choice
18. The correct answer is (B). The author notes (C) incorrect. The reference to another male
that the hummingbird “poises, or suspends hummingbird provides no support for the
himself on wing, for the space of two or idea that “fruits of his victory” refers to the
three seconds.” To accomplish this, the hum- nectar of flowers, so choice (D) is incorrect.
mingbird would have to move its wings at a 21. The correct answer is (C). The name “king-
high speed, making choice (B) the correct bird” infers that the bird is large, while the
answer. While the description of the wings bee is small. The author notes that the hum-
as “only like a mist” may sound as if they mingbird “teases” the kingbird and chases
are transparent, there is no other evidence to off the bee, showing that, despite its small
support this inference. Therefore, choice (A) size, the hummingbird defends itself against
is incorrect. The sun reflecting off the wings both large and small assailants. Choice (C)
would not make them “become invisible,” so is the correct answer. The author pointedly
choice (C) is also incorrect. While the author notes how the hummingbird is successful in
praises the hummingbird, the description of both instances, making choice (A) incorrect.
the wings does not emphasize beauty, thus Choice (B) is also incorrect, as the author
choice (D) is not correct. describes the female hummingbird as stay-
19. The correct answer is (C). In lines 88–90, ing on its nest and these lines refer to the
the author notes that, “for when two males male. While choice (D) may seem plausible
meet at the same bush, or flower, a battle given how the hummingbird “teases” the
instantly takes place.” When the author kingbird, the interaction with the bee is
says the victor “returns to the same place,” slightly different in that the bee harasses
the reference must be the bush or flower, a the hummingbird.
conclusion reinforced by line 78 in which 22. The correct answer is (A). By invoking
the author describes how the hummingbird Mt. Everest, the world’s tallest mountain,
“extracts its sweets” from the flower. Thus, the author is intent on showing that there are
choice (C) is correct. Though the author equally extraordinary and exotic geographi-
notes in line 81 that the hummingbird tends cal features on the ocean floor, making choice
to alight on the “dead twigs of a tree,” there (A) the correct answer. While the author does
is no indication that this is the place the bird emphasize that Mauna Kea is actually taller,
returns to after battle, so choice (A) is incor- that isn’t the point of the opening sentence.
rect. Similarly, although the two combatants Instead, the author continues describing
are male, the author has not indicated that the many geographical features found on
the males are battling over females. Rather, the ocean floor. Therefore, choice (B) is
the battle seems to be over access to food, incorrect. The author does not describe the
so choice (B) is also incorrect. Choice (D) geography of Mauna Kea or any other island
is off-base, as the author only mentions the mountain, so choice (C) is incorrect. Because
conqueror returning “to the same place.” there is no other mention of Mt. Everest in
the passage, choice (D) is also incorrect.

Master the New SAT®


Practice Test 1 543

answers PRACTICE TEST 1
23. The correct answer is (D). In lines 26–28, our planet” and then goes on to note “other
the author points out that the way the edges previously described ecosystems” that rely
of the plates fit together determines the on sunlight and photosynthesis as the basis
geography of the ocean floor. Thus, choice of their food web. This makes choice (B) the
(D) is correct. As the author does not note correct answer. Lines 43–46 do not provide
individual plates and the role each plays, evidence because the author has not yet
choice (A) is incorrect. Comparing the earth tied the new species to the existence of an
to a jigsaw puzzle does not underscore the ecosystem that relies on chemosynthesis,
spherical shape of the earth, so choice (B) so choice (A) is incorrect. The way hydro-
is incorrect. Because the author does not thermal vents are formed does not support
describe the shape of each plate, choice (C) the inference about ecosystems that are
is also incorrect. based on photosynthesis, so choice (C) is
24. The correct answer is (D). The author incorrect. Similarly, the kinds of metals that
describes the tectonic plates as “floating spew from the vents support bacteria that
on top of the Earth’s hot flowing mantle” form the base of the chemosynthetic food
and notes that “convection currents in the web, but the existence of these metals does
molten mantle layer cause the plates to not support the inference that before 1977
slowly move,” so choice (D) is correct. scientists believed that all ecosystems were
Earthquakes and volcanoes are a result of based on photosynthesis. Thus, choice (D)
the movement of the plates, so choice (A) is also incorrect.
is incorrect. The author mentions “convec- 27. The correct answer is (A). In lines 83–84,
tion currents,” not ocean currents, causing the author describes “molecules that billow
the plates to move, thus choice (B) is also out of the seafloor.” Because of the context,
incorrect. The upward flow of magma in we know that the molecules are somehow
diverging plates causes new ocean floor emerging from the seafloor, but there is no
crust, so choice (D) is not correct. indication that it is forceful, so choice (A)
25. The correct answer is (A). In lines 53–61, seems the most reasonable answer. Answer
the author notes that the discovery forced choices (B), (C), and (D) indicate a degree
scientists to “redefine living systems on of force or type of movement that is not
our planet” and then goes on to note “other indicated in the context of the sentence.
previously described ecosystems” that 28. The correct answer is (B). At the begin-
rely on sunlight and photosynthesis as the ning of the second paragraph, the author
basis of their food web. Therefore, choice notes, “The shape of the ocean floor, its
(A) is the correct answer. It can be inferred bathymetry, is largely a result of a process
that deep ocean hydrothermal vents were called plate tectonics.” The author goes on
discovered at the same time that the ecosys- to describe how the movement of plates
tems that thrive in and around them, thus causes hydrothermal vents, so choice (B) is
choice (B) is incorrect. In lines 89–104, the correct answer. While the author does
the author describes the food web found in describe the geography of the ocean floor,
this ecosystem and notes that these animals this description is general and doesn’t tie
must be adapted to survive in the extreme into the discussion of hydrothermal vents,
environment. There is no indication that so choice (A) is incorrect. The suggestion
the animals move between environments, that hydrothermal vents are created by the
so choice (C) is incorrect. While there are same forces that create volcanoes is tangen-
many metals that spew from the vents, the tial, thus choice (C) is also incorrect. While
depth and extreme conditions imply that the author describes the ecosystems around
mining would be difficult, so choice (D) is hydrothermal vents as different from those
also incorrect. that rely on photosynthesis, the discussion of
26. The correct answer is (B). In lines 53–61, plate tectonics does not support the inference
the author notes that the discovery forced that they are unusual. Therefore, choice (D)
scientists to “redefine living systems on is not correct.

www.petersons.com
544 PART VII: Five Practice Tests

29. The correct answer is (C). The passage sage ends with the idea that “Home to them
describes the features of hydrothermal vents, is an empty name,” referring again to the
so the discussion of plate tectonics serves to tenements, so choice (B) is correct. Though
explain how hydrothermal vents are formed. much of the first paragraph is a discussion
In lines 62–66, the author explains how the of the few job opportunities, the focus is still
movement of the plates causes the forma- on the lack of opportunities in the tenements,
tion of hydrothermal vents, so choice (C) is so choice (A) is not correct. The author does
correct. In lines 8–12, the author introduces allude to education in the last paragraph, but
the geography of the ocean floor but has it is not the focus of the passage, thus choice
not yet mentioned hydrothermal vents, so (C) is also incorrect. The second paragraph
choice (A) is incorrect. In lines 46–51, the deals with delinquency, but it is within the
author notes that many species have been context of the tenements. Therefore, choice
discovered in this unique environment, but (D) is incorrect.
there is no general discussion of the forma- 33. The correct answer is (C). Both the pre-
tion of hydrothermal vents. Thus, choice ceding and following sentences discuss the
(B) is not correct. Lines 66–69 note that the idea that the tenements are not “elevating”
main causes of hydrothermal vents are also and contribute to “idle” ways, so the context
responsible for volcanoes and earthquakes, makes choice (C) the correct answer. While
but the information is parenthetical and not as polluting can mean “dirty,” the author here
extensive as in the previous lines, so choice is concerned more with the child’s character
(D) is also incorrect. development, so choice (A) is incorrect.
30. The correct answer is (B). The description There is no evidence to suggest that the
of an “extreme environment,” including games are dangerous, thus choice (B) is
darkness and extreme temperatures that also incorrect. Similarly, the context does
would kill most other forms of life, signals not suggest that the author is referring to
that choice (B) is the answer. Neither choice the games as having a noisy character, so
(A) nor choice (C) supports the idea of the choice (D) is not correct.
extremely inhospitable environment that 34. The correct answer is (C). The context of
the author is describing in the paragraph. the tenements and the rest of the sentence
Likewise, choice (D) is incorrect because (“held down by the hand that of all should
while the environment described is unusual, labor to raise him”) help make clear that the
the focus is more on the idea that it is in- author is referring to the child’s surround-
hospitable, i.e., harmful to most forms of ings, the people who make up his commu-
life that we know. nity. Thus, choice (C) is the correct answer.
31. The correct answer is (B). Based on the Because the author’s reference to “the boy”
arrows, we know that the metals go up is a reference to all boys in the tenement,
through the vent. But once they separate choice (A) cannot be correct. Choice (B) is
from the solution, the only indication we incorrect because the author’s discussion is
have of what happens to them is the label more about the corrosive effects of the entire
of “metalliferous sediment” on the sea bed. community, not of the smaller family unit.
We can infer that “metalliferous sediment” There is no indication that the children go
is sediment made up of metals, so choice to school, so choice (D) is also incorrect.
(B) is correct. There are no arrows to in- 35. The correct answer is (D). Again, the
dicate that the metals are moved by ocean author’s focus is on the tenements as spirit
currents, nor arrows to show that they rise crushing, as supported by the following
to the surface, so choices (A) and (C) are sentence, “Its influence is scarcely of the
incorrect. The arrows in the black smoker elevating kind, if it have any.” Thus, choice
only go up, making choice (D) incorrect. (D) is the correct answer. While the author
32. The correct answer is (B). The passage speaks of “so many other human animals,”
begins with a sentence that focuses on boys creating an image of packed cages, there is
who live in the tenements. Further, the pas- no suggestion that the tenements are filthy,

Master the New SAT®


Practice Test 1 545

answers PRACTICE TEST 1
only that they don’t provide an environment their tactics suggests brutality in dealing
that helps grow a boy’s character, so choice with the children. The focus, again, is on the
(A) is incorrect. Likewise, choices (B) and child’s potential, so choice (D) is incorrect.
(C) are incorrect because they do not take 39. The correct answer is (B). The lines, “if
into account the preceding sentence about anyone doubt that this child of common clay
home being “the greatest factor of all in the have in him the instinct of beauty, of love for
training of the young,” as well as the context the ideal of which his life has no embodi-
of the tenements as soul-crushing. ment,” support the idea that the tenement
36. The correct answer is (A). The author child has a capacity to appreciate beauty,
demonstrates the meaning of “idle” in the so choice (B) is the correct answer. Neither
sentence that follows the word, in which choice (A) nor choice (D) suggests the child
the author describes the boy doing things has the capacity to appreciate beauty, given
that will not help him and may lead him his (or her) surroundings. While choice (C)
into trouble. Thus, choice (A) is the cor- suggests the calming effect of flowers and
rect answer. The context of the sentence, juxtaposes this with the image of a police-
the implication that “idle” ways will lead man’s brutal tactics, it does not suggest the
to trouble does not support choice (B), capacity to appreciate the “ideal.”
“vain,” as an answer. Similarly, there is no 40. The correct answer is (A). In lines 78–81,
indication that the boy acts younger than he the author notes that tenement children
is, so choice (C) is incorrect. While “idle” lead lives of “wearisome toil” as opposed
can mean inactive, the sentence that follows to “their happier fellows” whose play “has
indicates that the author is not referring to but just begun.” The author is maintaining
inactivity, so choice (D) is also incorrect. that tenement children aren’t allowed to play
37. The correct answer is (D). The author notes and just be children, so choice (A) is the
that “The city has no Truant Home” and that correct answer. While the author notes in a
efforts to get the child to go to school “are preceding sentence that they have a “hunger
paralyzed by this want,” or this need for a that is never fed,” the author is referring
truant home, so choice (D) is the correct more to a spiritual hunger, so choice (B) is
answer. The author notes in the lines that incorrect. The author says that the children
precede lines 34–41 that a reformatory is are more likely to be moved by flowers than
full of “vicious boys,” but the author sees a by a policeman with a club, but there is no
“want” or need for a truant home, so choices discussion of police brutality. The fact that
(A) and (C) are incorrect. The suggestion the children can sleep on the street suggests
that the city needs a truant home because that the police look the other way. Therefore,
the boy won’t go to school suggests the choice (C) is also incorrect. Choice (D)
opposite of choice (B). is incorrect because, although the author
38. The correct answer is (A). The author notes says “Home is an empty name,” there is no
in lines 43–47: “Rough as he is, if any one direct correlation between that idea and the
doubt that this child of common clay have photos. The boy in the factory might have
in him the instinct of beauty, of love for the a viable home, nevertheless, he is working
ideal of which his life has no embodiment, at too young an age.
let him put the matter to the test.” Thus, 41. The correct answer is (C). In lines 83–85,
choice (A) is the correct answer. Because the author notes that tenement children
the context suggests the author is speaking “banished and barred” from “a yard of turf”
of the child’s “instinct for beauty,” choice as from “a heaven that is not for such as
(B) is incorrect. While there is a suggestion they,” and notes later that there was not “a
that no flowers grow in the tenements, choice green sod within a quarter of a mile.” Thus,
(C) is incorrect because the author’s focus choice (C) is the correct answer. While the
here is on the child and his (or her) potential, author notes that this is “their first lesson
as evidenced by the sentence cited above. in ‘writin’,” the point is more about what
Similarly, the reference to the police and they wrote, so choice (A) is not correct.

www.petersons.com
546 PART VII: Five Practice Tests

Similarly, choice (B) is incorrect, as the whether … I had any right to bind you to
author does not mention lack of schooling me, my angel” and then goes on to write in
in this paragraph. There is no support for lines 11–13, “These doubts all arise, I am
choice (D), as the focus in this paragraph inclined to think, from your father’s attitude
is on children’s play. towards me.” Thus, choice (B) is correct.
42. The correct answer is (A). In lines 81–85, Choice (A) is incorrect, as Robert notes in
the author notes that tenement children are lines 33–34, “I really love and respect your
“banished and barred” from “a yard of turf” father.” There is no indication that he doubts
as from “a heaven that is not for such as Clara’s love for him. In fact, lines 42–43
they,” supporting the idea that there are no suggest otherwise: “You cannot think how
available green spaces for them, so choice your letter has raised and strengthened me.”
(A) is the correct answer. Choice (B) does not Therefore, choice (C) is incorrect. In lines
support the idea that there are no available 43–45, Robert writes, “You are splendid, and
green spaces, as it merely recounts seeing I have much more reason to be proud of you
children write on a fence. While choice (C) than of me,” so choice (D) is also incorrect.
notes that the children tried to write “keep 46. The correct answer is (A). In lines 7–9, the
off the grass,” it does not provide a wider author notes, “I wonder anxiously whether
context for why they wrote it. Choice (D) … I had any right to bind you to me, my
is incorrect because it provides no direct angel” and then goes on to write in lines
evidence to support the idea that there are 11–13, “These doubts all arise, I am inclined
no available green spaces. to think, from your father’s attitude towards
43. The correct answer is (B). In line 50, Robert me.” Thus, choice (A) is correct. Choice
expresses his devotion to Clara by saying (B) is incorrect, as Robert doubts himself in
that he is “entirely yours,” and likewise, these lines, not his relationship with Clara.
Napoleon begins his letter by noting that “I In Lines 31–39, Robert expresses his ad-
have not spent a day without loving you; I miration for Clara’s father, sentiments that
have not spent a night without embracing do not support the inference, so choice (C)
you.” Both letters express a devotion and is incorrect. Likewise, (D) suggests a new
commitment to the relationship, so choice commitment by Robert to their relationship,
(B) is the correct answer. While Napoleon’s so choice (D) is also incorrect.
letter betrays his jealousy, Robert’s letter 47. The correct answer is (C). Napoleon
contains no note of jealousy, so choice (A) writes in lines 82–87: “Woe to the person
is incorrect. Napoleon’s letter notes his responsible! May he, as punishment and
“pride and ambition” that force him away penalty, experience what my convictions
from Josephine, a reference to his work, but and the evidence (which is in your friend’s
Robert’s letter mentions no such reference, favour) would make me experience!” thus
so choice (C) is incorrect. Likewise, choice suggesting that choice (C) is the correct
(D) is incorrect, as Robert’s letter does not answer. While Napoleon himself does
lament that the two are apart. not like being away from Josephine (lines
44. The correct answer is (D). The author 59–63), there is no indication that she has
notes in lines 28–30, “but I cannot see why grown tired of his absence, so choice (A)
he should despise me and, as you say, hate is incorrect. Choice (B) is also incorrect,
me without any reason.” Thus, choice (D) as Napoleon questions whether she is too
is correct. The synonyms “despise” and busy to write to her husband (lines 77–80).
“hate” suggest that choice (A) is incorrect, as There is nothing to suggest that Napoleon
indifference implies little feeling. Similarly, thinks Josephine questions her marriage, so
choices (B) and (C) are incorrect, as “hate” choice (D) is also incorrect.
and “despise” suggest dislike, not mistrust 48. The correct answer is (C). Napoleon
or rudeness. writes in lines 82–87: “Woe to the person
45. The correct answer is (B). In lines 6–9, responsible! May he, as punishment and
the author notes, “I wonder anxiously penalty, experience what my convictions

Master the New SAT®


Practice Test 1 547

answers PRACTICE TEST 1
and the evidence (which is in your friend’s is also incorrect. Clara’s letter to Robert
favour) would make me experience!” This only seems to have strengthened his love
makes choice (C) the correct answer. While for her, rather than made him feel confused,
Napoleon himself does not like being away so choice (D) is not correct.
from Josephine (lines 59–63), this senti- 51. The correct answer is (A). Napoleon is
ment does not support the inference that incensed and crushed that Josephine used
he thinks she’s having an affair, so choice vous (a formal way of addressing someone)
(A) is incorrect. Choice (B) is also incor- in her letter, whereas Robert is delighted
rect, as Napoleon condemns the coldness of by the memory of Clara calling him du.
her letter but does not question her fidelity Therefore, choice (A) is the correct answer,
in these lines. While Napoleon questions and choice (B) is incorrect. Likewise, choice
Josephine’s commitment in the last lines (C) is incorrect because rather than calming
of the passage, the lines do not reflect his Napoleon, the use of vous has made him
mistrust, so choice (D) is also incorrect. angry. Choice (D) is incorrect, as there is
49. The correct answer is (B). The context— no evidence that Robert feels humbled by
Napoleon’s desire to see Josephine—and the her words.
phrase “by a matter of days” suggest that 52. The correct answer is (D). In lines 13–14,
choice (B) is the correct answer. Choice (A) Robert notes, “It is so easy to accept other
is incorrect, as stretching out the time would people’s estimate of oneself.” Similarly,
delay his ability to see Josephine. “Force” Napoleon writes in lines 91–95, “You love
does not make sense when added to the me less; but you will get over the loss. One
phrase “by a matter of days,” so choice (C) day you will love me no longer; at least tell
is incorrect. Likewise, choice (D) is incor- me; then I shall know how I have come to
rect because “cause” does not suggest the deserve this misfortune ….” Choice (D)
idea that Napoleon wants to make the days then is the correct answer. Neither Robert
go by more quickly. nor Napoleon show any evidence of being
50. The correct answer is (C). In lines 42–43, modest, thus choice (A) is incorrect. There
Robert writes to Clara: “You cannot think is no evidence to support the idea that
how your letter has raised and strengthened Robert angers quickly, whereas Napoleon
me. …” whereas Napoleon writes to Jose- seems quick to anger. Thus, choice (B) is
phine in lines 75–77: “Ah! wretch, how not correct. While Robert seems indecisive,
could you have written this letter? How Napoleon does not. In fact, he seems to
cold it is!” Thus, choice (C) is the correct make up his mind about Josephine quickly,
answer. While Robert does indeed express so choice (C) is also incorrect.
doubts, those doubts seem to stem from the
attitude of Clara’s father, not Clara’s letter,
so choice (A) is incorrect. Robert does not
question Clara’s loyalty, though Napoleon
does question Josephine’s, thus choice (B)

www.petersons.com
Practice Test 2 581

practice test
SECTION 1: READING TEST
65 Minutes • 52 Questions

TURN TO SECTION 1 OF YOUR ANSWER SHEET TO ANSWER THE QUESTIONS IN THIS


SECTION.

Directions: Each passage or pair of passages below is followed by a number of questions. After
reading each passage or pair of passages, choose the best answer to each question based on what
is stated or implied in the passage or passages and in any accompanying graphics (such as a
table, chart, or graph).

QUESTIONS 1–11 ARE BASED ON getting the average American to become


THE FOLLOWING PASSAGE AND more physically active. “We knew that
there were approximately 250,000 people
SUPPLEMENTARY CHART.
30 in the United States each year dying of
“A Menace to U.S. Public Health” was authored inactivity-related diseases, but the phrase
by Rob Wilkins, a member of the National inactivity-related disease lacks pizzazz,”
Federation of Professional Trainers (NFPT). Booth said. Without a catchy name, the
The NFPT certifies personal fitness trainers to condition wasn’t getting enough attention,
understand the fundamental exercise science 35 he said. “One day while I was out
principles in order to provide safe and effective jogging, it hit me: Why not call it SeDS?”
fitness programs to individuals or small groups. Approximately two-thirds of American
The following article is taken from the NFPT’s adults are currently overweight or obese
website, http://www.nfpt.com. according to the Center for Science in the
“In the United States, even the Grim 40 Public Interest (CSPI). Due to the fact
Reaper is flabby.” – Dr. Frank W. Booth, that more than one-fourth of Americans
University of Missouri-Columbia are not physically active in their leisure
Line Being fat and physically inactive now time, obesity has doubled, and Type
5 has a name—Sedentary Death Syndrome 2 diabetes (also known as adult-onset
or “SeDS.” Approximately 2.5 million 45 diabetes) has increased tenfold. Type 2
Americans will die prematurely in the diabetes is a devastating disease that may
next ten years due to SeDS, a number lead to complications such as blindness,
greater than all alcohol, guns, motor kidney failure, heart disease, circulatory
10 vehicles, illicit drug use, and sexual problems that can result in amputation,
behavior related deaths combined. 50 and premature death.
Research has identified SeDS as the Between 1982–1994, one third of all
second largest threat to public health new cases of Type 2 diabetes were among
(heart disease remains the number one people ages 10–19. The then-Surgeon
15 cause of death for Americans) and is General of the United States recently
expected to add as much as $3 trillion to 55 observed that, “We are raising the most
healthcare costs over ten years, more than overweight youngsters in American
twice the tax cut recently passed by the history.” In 2011–2012, 8.4% of 2- to
U.S. Senate. Frank W. Booth, a professor 5-year-olds had obesity compared with
20 at the University of Missouri-Columbia, 17.7% of 6- to 11-year-olds, and 20.5% of
stated that he invented the term SeDS 60 12- to 19-year-olds.
to emphasis his point that, in the United Studies indicate that currently about
States, even the Grim Reaper is flabby. 17% of the nation’s children are obese.
Professor Booth’s goal is to make the This is not surprising, considering that
25 public and the federal government pay the average American child spends 900
more attention and spend more money on 65 hours per year in school but 1,200 hours
GO TO THE
NEXT PAGE
www.petersons.com
582 PART VII: Five Practice Tests

watching television, according to the middle-aged and older population will


TV-Turnoff Network. now affect our children and will serve
The problem is made worse by the 95 to drastically decrease their quality of
fact that fewer than 3 in 10 high school life,” said researcher Ron Gomes of the
70 students get at least 60 minutes of University of Delaware.
physical activity every day. Less than All Americans may incur a severe
half (48%) of all adults meet the 2008 decline in their health due to consistent
Physical Activity Guidelines. 100 physical inactivity. Thirty-five known
“Our bodies were designed to be conditions are exacerbated by physical
75 physically active,” said Scott Gordon of inactivity; they include arthritis pain,
East Carolina University. The trouble arrhythmias, breast cancer, colon cancer,
is that hard work, from farming to congestive heart failure, depression,
simply doing household chores without 105 gallstone disease, heart attack,
appliances, is no longer part of ordinary hypertension, obesity, osteoporosis,
80 life for most people, he said. Gordon peripheral vascular disease, respiratory
called for activity to be put back in. problems, Type 2 diabetes, sleep apnea,
“In adults, this may mean planning and stroke.
exercise into your daily routine,” he 110 Providing enjoyable experiences is
said. “However, it may be as simple as a potent strategy for increasing activity
85 taking the stairs instead of the elevator levels in youth, their attitude about
a couple of times a day.” Booth and his the value of exercise, and ultimately
supporters said a special effort must be long-term health outcome. Introducing
made to reach children, so they won’t 115 and making exercise fun for young
turn fat and weak like their parents and, children may help them develop
90 also like their parents, get sick and die commitment and a positive attitude
early. “Perhaps the greatest tragedy is that toward physical activity as they go
ailments previously associated with the through adolescence and adulthood.

Number in Thousands of New Cases of Diagnosed Diabetes Among Adults


Aged 18–79 Years, U.S. 1980–2011

Master the New SAT®


Practice Test 2 583

practice test
1 The idea the author presents in lines 16–23 4 Based on the passage, which choice best
that the cost of SeDS is twice that of a recent describes the relationship between the de-
tax cut is analogous to an automaker that sign of our bodies and the fact that obesity
has doubled?
(A) investigates a faulty brake system in
one of its models and issues a recall. (A) Humans are naturally prone to
(B) apologizes to its customers for a obesity.
faulty brake system in one of its (B) Weight gain is passed from parents to
models and assures them that next children.
year’s model has already been (C) People eat more in order to perform
redesigned. modern activities.
(C) compensates all the customers whose (D) Hard work is no longer part of most
brakes have failed by offering them a
people’s lives.
free paint job for their car.
(D) gives classes to teach its customers
5 As used in line 5, “sedentary” most nearly
how to install a fix for a faulty brake
means
system in one of its models.
(A) inactive.
2 The structure of the article is designed to (B) robust.
(C) sudden.
(A) present opinions backed up by factual
detail. (D) obese.
(B) frighten readers who are ignoring
their weight issues. 6 In lines 12–15 (“Research has … for
(C) offer testimonials from those who are Americans.”), what is the most likely rea-
most affected. son the author compares SeDS with heart
(D) focus on statistical data and how it is disease?
being interpreted. (A) To compare the symptoms of obesity
with those of heart disease
3 What effect on meaning and tone does the (B) To make readers think of the many
mention of the Grim Reaper add to the warnings against heart disease
article? (C) To demonstrate how heart disease
often leads to issues with obesity
(A) It encourages weight loss by alluding
to the Grim Reaper’s gaunt form. (D) To emphasize that poor health is not
the issue, but a fatal outcome is
(B) It links preventable health issues with
a symbol of mortality.
7 Which choice provides the best evidence
(C) It uses a symbolic figure to make a
humorous point. for the answer to the previous question?
(D) It relies on valid scientific (A) Lines 6–11 (“Approximately …
conclusions from past studies. combined”)
(B) Lines 35–36 (“One day … SeDS”)
(C) Lines 53–57 (“The then-Surgeon …
history”)
(D) Lines 115–119 (“Introducing …
adulthood”)

GO TO THE
NEXT PAGE
www.petersons.com
584 PART VII: Five Practice Tests

8 It can reasonably be inferred from the pas- Line Indians, exclusive of those in Alaska. The
sage and the chart that steady increases in 5 names of the different tribes and bands
new cases of Type 2 diabetes began around as entered in the statistical table, so the
Indian Office Reports, number nearly
(A) 1980. three hundred.
(B) 1988. There is not among these three
10 hundred bands of Indians one which has
(C) 1998.
not suffered cruelly at the hands either of
(D) 2009. the Government or of white settlers. The
poorer, the more insignificant, the more
9 As used in line 32 , “pizzazz” most nearly helpless the band, the more certain the
15 cruelty and outrage to which they have
means
been subjected. This is especially true
(A) research. of the bands on the Pacific slope. These
Indians found themselves all of a sudden
(B) oomph.
surrounded by and caught up in the great
(C) seriousness. 20 influx of gold-seeking settlers, as helpless
(D) action. creatures on a shore are caught up in a
tidal wave. There was not time for the
10 The passage most strongly suggests which Government to make treaties; not even
time for communities to make laws. The
of the following? 25 tale of the wrongs, the oppressions, the
(A) A catchy name will motivate people murders of the Pacific-slope Indians in
to lose weight. the last thirty years would be a volume by
itself, and is too monstrous to be believed.
(B) Moving the body is essential to It makes little difference, however,
health. 30 where one opens the record of the history
(C) Type 2 diabetes is another form of of the Indians; every page and every year
obesity. has its dark stain. The story of one tribe is
(D) Children can have good habits the story of all, varied only by differences
despite poor role models. of time and place; but neither time nor
35 place makes any difference in the main
facts. Colorado is as greedy and unjust in
11 Which choice provides the best evidence
1880 as was Georgia in 1830, and Ohio in
for the answer to the previous question? 1795; and the United States Government
breaks promises now as deftly as then,
(A) Lines 24–28 (“Professor … active”)
40 and with an added ingenuity from long
(B) Lines 40–45 (“Due to … tenfold”) practice.
(C) Lines 45–50 (“Type 2 … death”) One of its strongest supports in so
(D) Lines 91–97 (“Perhaps … Delaware”) doing is the wide-spread sentiment
among the people of dislike to the Indian,
45 of impatience with his presence as a
QUESTIONS 12–22 ARE BASED ON THE
“barrier to civilization” and distrust of
FOLLOWING PASSAGE. it as a possible danger. The old tales
This passage is excerpted from Helen Hunt Jack- of the frontier life, with its horrors of
son’s A Century of Dishonor, published in 1881. Indian warfare, have gradually, by two
In 1879, Jackson became a Native American 50 or three generations’ telling, produced
rights activist after witnessing a speech by Ponca in the average mind something like an
chief Standing Bear. hereditary instinct of questioning and
There are within the limits of the unreasoning aversion which it is almost
United States between two hundred impossible to dislodge or soften.
and fifty and three hundred thousand

Master the New SAT®


Practice Test 2 585

practice test
55 President after president has appointed 12 The author’s description of government
commission after commission to inquire inquiries into the handling of Native
into and report upon Indian affairs, and to American affairs in lines 55–69 suggests
make suggestions as to the best methods that the author
of managing them. The reports are filled
60 with eloquent statements of wrongs (A) believes the government recognizes
done to the Indians, of perfidies on the the need to be fairer in its dealings
part of the Government; they counsel, with the Native Americans.
as earnestly as words can, a trial of the (B) is doubtful that the government
simple and unperplexing expedients of is inquiring into Native American
65 telling truth, keeping promises, making affairs with the intention of making
fair bargains, dealing justly in all ways substantive changes.
and all things. These reports are bound up
(C) supposes that presidents have been
with the Government’s Annual Reports,
more critical of the government’s
and that is the end of them.
dealings with the Native Americans
70 The history of the Government
than the commissions.
connections with the Indians is a
shameful record of broken treaties and (D) acknowledges that the government
unfulfilled promises. The history of the understands the problem but is not
border white man’s connection with the equipped to determine a viable
75 Indians is a sickening record of murder, solution.
outrage, robbery, and wrongs committed
by the former, as the rule, and occasional 13 The author refers to different states and
savage outbreaks and unspeakably different times (lines 29–41) as a way of
barbarous deeds of retaliation by the
80 latter, as the exception. (A) citing specific abuses that were
Taught by the Government that they spread over a century.
had rights entitled to respect, when those (B) pointing out which states had the
rights have been assailed by the rapacity worst records of abuse.
of the white man, the arm which should
(C) showing that these abuses no longer
85 have been raised to protect them has ever
occur in America.
been ready to sustain the aggressor.
The testimony of some of the highest (D) defining where abuses in the Pacific
military officers of the United States is slope area occurred.
on record to the effect that, in our Indian
90 wars, almost without exception, the first 14 Which choice provides the best evidence
aggressions have been made by the white for the answer to the previous question?
man …. Every crime committed by a
white man against an Indian is concealed (A) Lines 9–12 (“There is not … white
and palliated. Every offense committed settlers.”)
95 by an Indian against a white man is borne (B) Lines 24–28 (“The tale … believed”)
on the wings of the post or the telegraph
(C) Lines 29–32 (“It … stain”)
to the remotest corner of the land, clothed
with all the horrors which the reality or (D) Lines 70–73 (“The history …
imagination can throw around it. Against promises”)
100 such influences as these are the people of
the United States need to be warned.

GO TO THE
NEXT PAGE
www.petersons.com
586 PART VII: Five Practice Tests

15 What explanation does the author give 19 Which of the following summaries of the
for the abuse of Native Americans as de- last paragraph is the most accurate?
scribed in lines 22–28?
(A) The generals of the U.S. Army
(A) The author blames old tales of Native suggest that they had to be aggressive
Americans attacks on the frontier. to keep Native Americans from
(B) The author cites a record of broken defeating them and that sometimes
treaties and abuse of the laws. there were crimes committed against
Native Americans. Both sides spread
(C) The author says that the settlers
their interpretation of events across
surrounded Native Americans.
the nation.
(D) The author describes the Pacific-
(B) In court hearings, soldiers discussed
slope area as lawless and chaotic. how the white man often took the
fight to Native Americans in order to
16 In what year did serious abuses of Native move them off the land and that there
Americans occur in Georgia? were occasions when this resulted in
savage behavior by both parties.
(A) 1795 (C) Proof that the white man was the
(B) 1830 aggressor in almost every conflict
(C) 1855 comes from the U.S. Army itself
and the offenses of white men are
(D) 1880 disguised while the few offenses
of Native Americans are widely
17 The author chose a text structure exaggerated.
designed to (D) The history of the conflicts between
Native Americans and white men is
(A) follow the historical chronology of one of gross injustice and extreme
how settlers in the United States dealt crimes against the tribes most of the
with Native Americans over time. time, while horrible crimes against
(B) dispel some of the common white people are generally few and
misconceptions of the dealings far between.
between the United States and Native
Americans.
20 The central theme of the passage is that
(C) categorize causes and effects in
explaining the US government’s (A) Native Americans have been
treatment of Native Americans. victimized by the U.S. government.
(D) present opposing viewpoints as (B) what happened to one tribe happened
to why the United States has had to all the tribes eventually.
conflicts with Native Americans. (C) the U.S. government has done little
to help Native Americans.
18 As used in line 94, “palliated” most nearly (D) Native Americans and white settlers
means had conflicts with each other in the
past.
(A) intensified.
(B) confused.
(C) eased.
(D) excused.

Master the New SAT®


Practice Test 2 587

practice test
21 Which choice provides the best evidence separated from their families at a
for the answer to the previous question? moment’s notice.
20 In 1802, the first slaves to inhabit
(A) Lines 12–16 (“The poorer … Arlington arrived with their owner,
subjected”) George Washington Parke Custis. The
(B) Lines 29–32 (“It makes … stain”) grandson of Martha Washington and
adopted grandson of George Washington,
(C) Lines 32–36 (“The story … facts”)
25 Custis had grown up at Mount Vernon,
(D) Lines 73–80 (“The history … as had many of his slaves. Upon Martha
exception”) Washington’s death, Custis inherited
her slaves and purchased others who
22 As used in line 79, “barbarous” most belonged to his mother. In all, Custis
30 owned nearly 200 slaves and as many
nearly means with
as 63 lived and worked at Arlington.
(A) silent determination. The others worked on his other two
plantations near Richmond, Virginia.
(B) steadfast revenge.
Once at Arlington, the slaves
(C) calculated antagonism. 35 constructed log cabins for their homes
(D) wild brutality. and began work on the main house.
Using the red clay soil from the property
QUESTIONS 23–32 ARE BASED ON and shells from the Potomac river, they
made the bricks and stucco for the walls
THE FOLLOWING PASSAGE AND
40 and exterior of the house. The slaves
SUPPLEMENTARY MATERIAL. also harvested timber from the Arlington
Robert E. Lee and his family lived on a plantation forest, which was used for the interior
estate in Arlington, Virginia, up until 1861. When flooring and supports. The slaves were
Civil War broke out, he and his family departed responsible for keeping up the house and
for safer quarters. Lee became the commander of 45 laboring on the plantation, working to
the Rebel field forces in 1862. His former home is harvest corn and wheat, which was sold at
now a National Park site. The full-length text of market in Washington.
the following passage, provided by the National Custis saw his daughter marry Lt.
Park Service, can be found at http://www.nps. Robert E. Lee at Arlington in 1831.
gov/arho/learn/historyculture/slavery.htm. 50 Robert and Mary Anna came to call
Arlington home and Custis was a
Slavery at Arlington prominent figure in the lives of the seven
From its earliest days, Arlington House Lee children. In his later years, Custis did
was home not only to the Custis and Lee not stray far from Arlington. He made his
families who occupied the mansion, but 55 will in 1855, and he increasingly relied
Line to dozens of slaves who lived and labored
on his son-in-law, Col. Lee, to handle his
5 on the estate.
tangled business affairs. Until his death,
For nearly sixty years, Arlington Custis retained his old bedchamber in the
functioned as a complex society made north wing of the mansion, where he died
up of owners and slaves, whites and 60 after a short illness on October 10, 1857.
blacks. To some observers, on the surface, Some slaves had very close
10 Arlington appeared as a harmonious
relationships with the family members,
community in which owner and slave though these relationships were governed
often lived and worked side by side. Yet by the racial hierarchy that existed
an invisible gulf separated the two, as 65 between slaves and slaveholders. Mr.
slaves were the legal property of their Custis relied heavily on his carriage
15 owners. The enslaved possessed no
driver, Daniel Dotson, and Mrs. Lee had
rights, could not enter into legally binding a personal relationship with the head
contracts, and could be permanently housekeeper, Selina Gray. As Mary’s
GO TO THE
NEXT PAGE
www.petersons.com
588 PART VII: Five Practice Tests

70 arthritis increasingly restricted her to her by the Custises at the time of her
activities through the years, she depended emancipation around 1826.
on Selina for assistance. As evidence While such allowances may have
of their close bond, Mrs. Lee entrusted 95 improved the quality of life for the
Selina with the keys to the plantation at Arlington slaves, most black men and
75 the time of the Lees’ evacuation in May women on the estate remained legally in
1861. bondage until the Civil War. In his will,
There is evidence that some slaves at Custis stipulated that all the Arlington
Arlington had opportunities not widely 100 slaves should be freed upon his death
afforded to slaves elsewhere. Mrs. Custis, if the estate was found to be in good
80 a devout Episcopalian, tutored slaves in financial standing or within five years
basic reading and writing so that they otherwise. When Custis died in 1857,
could read the Bible. Mrs. Lee and her Robert E. Lee—the executor of the
daughters continued this practice even 105 estate—determined that the slave labor
though Virginia law had prohibited the was necessary to improve Arlington’s
85 education of slaves by the 1840s. Mrs. financial status. The Arlington slaves
Custis also persuaded her husband to free found Lee to be a more stringent
several women and children. taskmaster than his predecessor. Eleven
Some of these emancipated slaves 110 slaves were “hired out” while others were
settled on the Arlington estate, including sent to the other estates. In accordance
90 Maria Carter Syphax who lived with her with Custis’s instructions, Lee officially
husband on a seventeen-acre plot given freed the slaves on December 29, 1862.

The room at the east end on the lower level housed the summer kitchen, with cooks’ quarters above. The center room
at the lower level was a washroom, with the washerwoman’s quarters above. The rooms at the west end housed various
domestic slaves, including the coachmen, gardener, and housekeeper.

Master the New SAT®


Practice Test 2 589

practice test
23 What is the most likely purpose of the 27 Which of the following statements is true
passage? based on the graphic and the passage.

(A) To inform people about the evils of (A) The slaves lived in one enormous
slavery house built just for slaves.
(B) To persuade people that slavery was (B) The slaves’ quarters were adequate
not so terrible for the needs of the slaves.
(C) To describe the history of Arlington (C) The slaves lived in very tight
House quarters.
(D) To illustrate how slaves lived before (D) The slave quarters were located next
the Civil War to the main house of the plantation.

24 Why did Mrs. Custis teach her slaves to 28 How was Robert E. Lee related to George
read? Washington?

(A) So they could teach other slaves and (A) Lee married the daughter of
become self-sufficient Washington’s grandson.
(B) So they could read their contracts (B) Lee married George Washington’s
with their owners granddaughter.
(C) So they could teach her children to (C) Lee’s son married Martha
read Washington’s granddaughter.
(D) So they could read the Bible (D) Lee’s father was Martha
Washington’s nephew.
25 What did George Washington Parke Cus-
tis inherit from his grandmother Martha 29 What evidence in the passage suggests that
Washington? Lee was more of a practical man than an
idealist?
(A) Land and slaves
(B) Slaves (A) Lines 53–57 (“Custis did not …
(C) A house with land and slaves business affairs”)
(D) Three plantations (B) Lines 79–85 (“Mrs. Custis … by the
1840’s”)
26 How was the life of Selina Gray different (C) Lines 103–107 (“When Custis …
financial status”)
from that of other slaves?
(D) Lines 112–113 (“Lee officially …
(A) She didn’t have to work as hard. December 29, 1862”)
(B) She was trusted by Mrs. Lee.
(C) She took care of the Lee children.
(D) She ran the whole plantation.

GO TO THE
NEXT PAGE
www.petersons.com
590 PART VII: Five Practice Tests

30 Which of the following best illustrates that over the past century and is projected
slaves were considered property in the era Line to rise another 2 to 11.5°F over the
described in the passage? 5 next hundred years. Small changes in
the average temperature of the planet
(A) Lines 6–9 (“Arlington functioned … can translate to large and potentially
and blacks”) dangerous shifts in climate and weather.
(B) Lines 27–29 (“Custis inherited … his The evidence is clear. Rising global
mother”) 10 temperatures have been accompanied by
changes in weather and climate. Many
(C) Lines 82–85 (“Mrs. Lee … the
places have seen changes in rainfall,
1840s”)
resulting in more floods, droughts, or
(D) Lines 85–87 (“Mrs. Custis … and intense rain, as well as more frequent and
children”) 15 severe heat waves. The planet’s oceans
and glaciers have also experienced some
31 How does the author use the phrase “invis- big changes—oceans are warming and
ible gulf” (line 13)? becoming more acidic, ice caps are
melting, and sea levels are rising. As
(A) As a figure of speech—related to 20 these and other changes become more
differences in stature pronounced in the coming decades, they
(B) As a martime definition—related to a will likely present challenges to our
hidden body of water society and our environment.
Humans are largely responsible for
(C) As a geographical reference—related
25 recent climate change. Over the past
to a ravine or abyss
century, human activities have released
(D) As an architectural description— large amounts of carbon dioxide and other
related to building placement greenhouse gases into the atmosphere.
The majority of greenhouse gases come
32 In the context of the passage, what is the 30 from burning fossil fuels to produce
energy, although deforestation, industrial
best definition of the word “afforded” (line
processes, and some agricultural practices
79)?
also emit gases into the atmosphere.
(A) Spared or given up without risk Greenhouse gases act like a blanket
35 around Earth, trapping energy in the
(B) Had sufficient money to pay for
atmosphere and causing it to warm. This
(C) Provided or supplied phenomenon is called the greenhouse
(D) Purchased in exchange for effect and is natural and necessary to
support life on Earth. However, the
QUESTIONS 33–42 ARE BASED ON THE 40 buildup of greenhouse gases can change
Earth’s climate and result in dangerous
FOLLOWING PASSAGES.
effects to human health and welfare and
Passage 1 is excerpted from the U.S. Environ- to ecosystems.
mental Protection Agency website. Passage 2 is Our lives are connected to the
excerpted from the article “Science Has Spoken: 45 climate. Human societies have adapted
Global Warming Is a Myth” by Oregon Institute to the relatively stable climate we have
of Science and Medicine chemists, Arthur B. enjoyed since the last ice age, which
Robinson and Zachary W. Robinson. This article ended several thousand years ago. A
was published in the Wall Street Journal in 1997. warming climate will bring changes that
50 can affect our water supplies, agriculture,
Passage 1
power and transportation systems, the
Climate change is happening natural environment, and even our own
Our Earth is warming. Earth’s health and safety.
average temperature has risen by 1.4°F

Master the New SAT®


Practice Test 2 591

practice test
Some changes to the climate are useful, for now computer climate models
55 unavoidable. Carbon dioxide can stay in 105 are very unreliable.
the atmosphere for nearly a century, so So we needn’t worry about human use
Earth will continue to warm in the coming of hydrocarbons warming the Earth. We
decades. The warmer it gets, the greater also needn’t worry about environmental
the risk for more severe changes to the calamities, even if the current, natural
60 climate and Earth’s system. Although it’s 110 warming trend continues: After all the
difficult to predict the exact impacts of Earth has been much warmer during the
climate change, what’s clear is that the past 3,000 years without ill effects.
climate we are accustomed to is no longer But we should worry about the effects
a reliable guide for what to expect in the of the hydrocarbon rationing being
65 future. 115 proposed at Kyoto. Hydrocarbon use has
We can reduce the risks we will face major environmental benefits. A great
from climate change. By making choices deal of research has shown that increases
that reduce greenhouse gas pollution in atmospheric carbon dioxide accelerate
and preparing for the changes that are the growth rates of plants and also permit
70 already underway, we can reduce risks 120 plants to grow in drier regions. Animal
from climate change. Our decisions today life, which depends upon plants, also
will shape the world our children and increases.
grandchildren will live in.
Passage 2 33 Upon which concepts do both passages
[The global warming] hypothesis fully agree?
75 predicts that global temperatures will
rise significantly, indeed catastrophically, (A) That global warming has been proven
if atmospheric carbon dioxide rises. by evidence
Most of the increase in atmospheric (B) That an increase in overall
carbon dioxide has occurred during temperature is manageable
80 the past 50 years, and the increase has (C) That levels of atmospheric carbon
continued during the past 20 years. Yet dioxide have increased
there has been no significant increase
(D) That usual weather patterns have
in atmospheric temperature during
those 50 years, and during the 20 years been affected
85 with the highest carbon dioxide levels,
temperatures have decreased. 34 Which choice provides the best evidence
In science, the ultimate test is the for the answer to the previous question?
process of experiment. If a hypothesis
fails the experimental test, it must be (A) Lines 5–8 (“Small changes … and
90 discarded. Therefore, the scientific weather.”)
method requires that the global warming (B) Lines 39–43 (“However, the … to
hypothesis be rejected. ecosystems.”)
Why, then, is there continuing
(C) Lines 78–81 (“Most of the … 20
scientific interest in “global warming”?
years.”)
95 There is a field of inquiry in which
scientists are using computers to try (D) Lines 113–115 (“But we should … at
to predict the weather—even global Kyoto.”)
weather over very long periods. But
global weather is so complicated that
100 current data and computer methods are
insufficient to make such predictions.
Although it is reasonable to hope that
these methods will eventually become

GO TO THE
NEXT PAGE
www.petersons.com
592 PART VII: Five Practice Tests

35 Which point of view characterizes both 39 In both passages, the authors present in-
passages? formation by

(A) One of scientists taking neutral (A) listing a sequence of events that
positions begins in the past and continues into
(B) One of fanatics defending a cause the future.
(C) One of humans concerned for global (B) discussing the causes of a situation
well-being and the resulting effects or lack of
effects.
(D) One of debaters directly addressing
readers as “you” (C) comparing two different approaches
to a problem and determining which
will be most effective.
36 In making their arguments, the authors of
(D) defining the problems the world faces
both passages fail to
and then offering solutions to them.
(A) provide sources of proof for their
claims. 40 The essential difference between the argu-
(B) account for recent advances in the ments the two sets of authors present is
field. whether or not
(C) acknowledge dissenting opinions.
(A) atmospheric hydrogen should be
(D) provide definitions for their controlled.
terminology.
(B) climate and weather can be modified
by humans.
37 The word choice of the first passage (po-
(C) atmospheric hydrogen poses a threat
tentially dangerous, severe, challenges, to human life.
dangerous effects) and the second passage
(D) the production of hydrocarbons is a
(no significant, rejected, insufficient, un-
natural result of human activity.
reliable) differ in that

(A) the first is employing colorful 41 Which choice provides the best evidence
language to describe, and the second for the answer to the previous question?
is relying on simpler language to
define. (A) Lines 29–30 (“The majority … into
(B) Passage 1 uses scientific terms and the atmosphere.”)
Passage 2 uses layperson’s terms. (B) Lines 55–58 (“Carbon dioxide …
(C) the first passage attempts to reassure coming decades”)
readers, and the second passage tries (C) Lines 74–77 (“The global … dioxide
to motivate them. rises”)
(D) Passage 1 is sounding a warning, and (D) Lines 116–120 (“A great … drier
Passage 2 is negating any cause for regions”)
alarm.
42 As used in line 7, “translate” most nearly
38 As used in line 21, “pronounced” most means
nearly means
(A) comprehend.
(A) articulated. (B) expand.
(B) announced. (C) transform.
(C) inconspicuous. (D) explain.
(D) noticeable.

Master the New SAT®


Practice Test 2 593

practice test
QUESTIONS 43–52 ARE BASED ON THE Their faces were interesting—of the
FOLLOWING PASSAGE. dry, shrewd, quick-witted New England
45 type, and thin hair twisted neatly back
The following is an excerpt from a short story, out of the way. Mrs. Crowe could look
“Miss Tempy’s Watchers,” by Sarah Orne Jewett, vague and benignant, and Miss Binson
a novelist and short-story writer who lived from was, to quote her neighbors, a little too
1849–1909. In the story, two women watch over sharp-set, but the world knew that she
their deceased friend on the evening before her 50 had need to be, with the load she must
funeral and share their memories of her. carry supporting an inefficient widowed
The time of year was April; the sister and six unpromising and unwilling
place was a small farming town in New nieces and nephews. The eldest boy was
Hampshire, remote from any railroad. at last placed with a good man to learn
Line One by one the lights had been blown out 55 the mason’s trade. Sarah Ann Binson,
5 in the scattered houses near Miss Tempy for all her sharp, anxious aspect never
Dent’s, but as her neighbors took a last defended herself, when her sister whined
look out of doors, their eyes turned with and fretted.
instinctive curiosity toward the old house She was told every week of her life
where a lamp burned steadily. They gave 60 that the poor children would never have
10 a little sigh. “Poor Miss Tempy!” said had to lift a finger if their father had lived,
more than one bereft acquaintance; for and yet she had kept her steadfast way
the good woman lay dead in her north with the little farm, and patiently taught
chamber, and the lamp was a watcher’s the young people many useful things for
light. The funeral was set for the next day 65 which, as everybody said, they would
15 at one o’clock. live to thank her. However pleasureless
The watchers were two of her oldest her life appeared to outward view, it was
friends. Mrs. Crowe and Sarah Ann brimful of pleasure to herself.
Binson. They were sitting in the kitchen Mrs. Crowe, on the contrary, was
because it seemed less awesome than 70 well-to-do, her husband being a rich
20 the unused best room, and they beguiled farmer and an easy-going man. She
the long hours by steady conversation. was a stingy woman, but for all of that
One would think that neither topics nor she looked kindly; and when she gave
opinions would hold out, at that rate, all away anything, or lifted a finger to help
through the long spring night, but there 75 anybody, it was thought a great piece of
25 was a certain degree of excitement just beneficence, and a compliment, indeed,
then, and the two women had risen to which the recipient accepted with twice
an unusual level of expressiveness and as much gratitude as double the gift that
confidence. Each had already told the came from a poorer and more generous
other more than one fact that she had 80 acquaintance. Everybody liked to be on
30 determined to keep secret; they were good terms with Mrs. Crowe. Socially,
again and again tempted into statements she stood much higher than Sarah Ann
that either would have found impossible Binson.
by daylight. Mrs. Crowe was knitting a
blue yarn stocking for her husband; the
35 foot was already so long that it seemed as
if she must have forgotten to narrow it at
the proper time. Mrs. Crowe knew exactly
what she was about, however; she was
of a much cooler disposition than Sister
40 Binson, who made futile attempts at some
sewing, only to drop her work into her lap
whenever the talk was most engaging.

GO TO THE
NEXT PAGE
www.petersons.com
594 PART VII: Five Practice Tests

43 The two women are in Miss Tempy Dent’s 46 Even though these two women are very
house because they are different, the author shows that they are
getting closer by
(A) waiting to tell other friends and
family that she has died. (A) having them sit in the kitchen instead
(B) staying with the body so it isn’t alone of the best room.
until it is buried. (B) having Mrs. Crowe give Sarah Ann
(C) conducting a funeral service something nice.
following Miss Tempy’s requests. (C) explaining that Sarah Ann was
(D) visiting Miss Tempy Dent, but she actually very happy.
has died. (D) explaining they were telling each
other secrets.
44 The theme of this excerpt can best be
described as 47 Which choice provides the best evidence
for the answer to the previous question?
(A) old friends often grow closer when
they lose a friend. (A) Lines 22–28 (“One would … and
(B) the bonds of friendship remain strong confidence.”)
even in death. (B) Lines 46–53 (“Mrs. Crowe … nieces
(C) trying to stay awake all night isn’t and nephews.”)
hard for dear friends. (C) Lines 59–66 (“She was … thank her.”)
(D) the living carry on the traditions of (D) Lines 69–71 (“Mrs. Crowe … easy-
the dead. going man.”)

45 In lines 33–42, the author uses the activi- 48 Which choice provides the best summary
ties of knitting and sewing to show that of what happened between Mrs. Crowe
and Sarah Ann Binson in Miss Tempy’s
(A) the two women are so lost in kitchen?
conversation that they neglect their
work. (A) Both women found it hard to fill up
(B) there are a lot of things that need to the long hours.
be done by morning. (B) Both women had become excited by
(C) the women are so upset by the death the intimacy they shared.
that they find it hard to work. (C) Mrs. Crowe did not reveal as much
(D) the two elderly women are getting as Sarah Ann Binson.
absent-minded and weak. (D) Sarah Ann Binson revealed her love
of pleasure and freedom.

Master the New SAT®


Practice Test 2 595

practice test
49 Although they were both friends to Miss 51 As used in line 49, “sharp-set” most nearly
Tempy Dent, the relationship between means
Mrs. Crowe and Sarah Ann Binson could
be described as not (A) hungry.
(B) restless.
(A) all that close, but enjoying the same
(C) willy-nilly.
pleasures.
(B) the same age, but close enough to (D) close-fisted.
understand.
(C) united in purpose, but learning to 52 As used in line 76, “beneficence” most
work together. nearly means
(D) from the same social status, but (A) furniture.
intimate.
(B) succor.
(C) magnanimity.
50 Which choice provides the best evidence
for the answer to the previous question? (D) scrupulousness.

(A) Lines 1–3 (“The time of … any


railroad.”)
(B) Lines 28–33 (“Each had … by
daylight.”)
(C) Lines 37–42 (“Mrs. Crowe … most
engaging.”)
(D) Lines 81–83 (“Socially … Sarah Ann
Binson.”)

STOP
If you finish before time is called, you may check your work on this section only.
Do not turn to any other section.

www.petersons.com
Practice Test 2 631

answers PRACTICE TEST 2
ANSWER KEY AND EXPLANATIONS
Section 1: Reading Test

1. C 12. B 23. C 34. B 45. A


2. A 13. A 24. D 35. C 46. D
3. B 14. D 25. B 36. A 47. A
4. D 15. D 26. B 37. D 48. B
5. A 16. B 27. C 38. D 49. D
6. D 17. B 28. A 39. B 50. B
7. A 18. D 29. C 40. C 51. A
8. C 19. C 30. B 41. C 52. C
9. B 20. A 31. A 42. C
10. B 21. D 32. C 43. B
11. B 22. D 33. C 44. A

READING TEST RAW SCORE


(Number of correct answers)

1. The correct answer is (C). An automaker illustrating statistics about diabetes. Choice
providing a free paint job to customers with (B) is incorrect because the information is
cars that have failing breaks offers generous not presented in a sensational manner; it
but irrelevant compensation for an error is concerned with facts and opinions from
that could have life or death consequences. experts. Those affected are not quoted, so
Similarly, a tax cut is a pleasant bonus, but choice (C) is also incorrect. Choice (D) fo-
in the author’s opinion, if the seriousness of cuses only on statistics, and the article does
SeDS were addressed instead, the savings not discuss how the statistics are interpreted,
for the U.S. treasury would be enormous, so it is incorrect as well.
and the benefits to U.S. citizens would be 3. The correct answer is (B). The Grim Reaper
incalculable. Choice (A) is incorrect because is a traditional image of the inevitability of
the automaker is taking appropriate steps death, and the article suggests that diseases
to remedy its error, as well as allocating its caused by obesity hasten death. Choice (A)
funds responsibly, which is the opposite of is incorrect because in this article, the Grim
what the author sees happening in the U.S. Reaper is referred to as “flabby.” Choice
Choices (B) and (D) are incorrect because (C) is partially correct because the idea of a
they show acknowledgement of the problem flabby Grim Reaper is ironic; however, the
but offer nonsensical solutions. article is about a larger attempt to educate
2. The correct answer is (A). The article the public about the causes of a health cri-
presents the opinions of Frank Booth first, sis. Choice (D) is also incorrect, confusing
accompanied by factual details. These are statistical details with the fantastical image
followed by quotes from Scott Gordon and of death as embodied by the Grim Reaper.
Ron Gomes, along with other facts and charts

www.petersons.com
632 PART VII: Five Practice Tests

4. The correct answer is (D). The article says the number peaked in 2009. Choice (A) is
that the human body has evolved over time the start of the chart, and although it shows
to do hard work like farming and household an increasing trend overall, there are many
chores, but machines now do much of that decreases as well. Choice (D) is the high-
work, which leads to obesity. Choice (A) est number of new cases reported, but it is
focuses on obesity without recognizing the not the beginning of continuing increases
causes. Choice (B) confuses the fact that because the number of cases falls off the
children do inherit different body types, but following year.
inactivity is not an inherited trait. Choice (C) 9. The correct answer is (B). Booth wanted
is in direct contradiction to the point made to come up with a catchy name, something
in the article, which is that people are now that would get people’s attention. Choice
more sedentary than in previous generations. (A) makes sense within the context of the
5. The correct answer is (A). Frank Booth sentence, but it doesn’t convey the idea of
wanted to emphasize that a lack of exercise a catchy name. Choice (C) suggests that
or activity leads to obesity and even death, “inactivity-related disease” is too frivolous,
so he chose “sedentary” as a synonym for with the author saying it’s not going to
“inactive” to add pizzazz to a condition that catch the public’s attention. Choice (D) ties
needs greater attention. Choice (B) is actu- the idea of “inactivity-related” to a lack of
ally the opposite meaning, referring to health “action,” but it doesn’t match other ideas
and energy. Choice (C) echoes another health in the sentence.
issue: Sudden Death Syndrome, which 10. The correct answer is (B). The author of-
refers to children who die mysteriously as fers activity or exercise as a solution to the
infants. The process the article discusses is rise in obesity. Although this connection is
not sudden or restricted to children. Choice not explained in detail, it forms the basis of
(D) describes the result of inactivity, but the article. Choice (A) is incorrect because
does not define it. the author sees a catchy name as simply a
6. The correct answer is (D). The author links way to make people aware of the problem.
a well-known killer—heart disease—to a Choice (C) is incorrect because although the
health concern that is usually considered article connects the rise in Type 2 diabetes
less critical: obesity. Choice (A) focuses with obesity, it is only one health issue the
on similar symptoms but is incorrect; the article discusses. Choice (D) is also incor-
author is comparing the outcomes, not the rect because of the narrowness of its focus.
symptoms. Choice (B) doesn’t take into The article brings up the importance of
account one half of the comparison, so it is children developing better habits than their
incorrect. Choice (C) is incorrect because parents, but is not the overarching theme of
the connection is reversed. Obesity leads the passage.
to heart disease. 11. The correct answer is (B). Choice (B)
7. The correct answer is (A). Choice (A) establishes a clear link between the number
emphasizes that SeDs is a bigger killer of Americans who are not physically active
than the sum of several causes that readers and the doubling of the obesity rate. Choice
know well. Choice (B) is incorrect because (A) mentions the fact that one person wants
it refers to Booth having a revelation about more money to be allocated to helping
the name of the disease. Choice (C) is a Americans be more active, but doesn’t link
statement from the then-Surgeon General obesity to fitness. Choice (C) describes
about the disease, but it does not relate the the complications of Type 2 diabetes but
answer to the previous question. Choice (D) doesn’t refer to obesity. Choice (D) explains
is incorrect because it talks about one way the connection between children and ail-
to prevent obesity in children. ments associated with obesity but doesn’t
8. The correct answer is (C). In 1998, new indicate how essential it is for all human
cases of Type 2 diabetes began to increase beings to be physically active.
and grow continuously every year, until

Master the New SAT®


Practice Test 2 633

answers PRACTICE TEST 2
12. The correct answer is choice (B). Although not the specific abuse of Native Americans.
the government professes that wrongs done Choice (B) is incorrect because there were
to the Native Americans should be corrected, no treaties or laws to break. Choice (C) is
the author’s use of the phrase “simple and partially correct because it describes Na-
unperplexing expedients” suggests that the tive Americans as being outnumbered, but
author believes the government’s intentions it doesn’t explain the fundamental dangers
are not sincere. For this reason, choice (A) of the time and place.
is incorrect. Choice (C) is also incorrect, 16. The correct answer is (B). The article spe-
as the author does not make a distinction cifically refers to abuse in Georgia in 1830.
between the presidents and the commissions Choice (A) is associated with Ohio; choice
in their approach to Native American affairs. (C) is associated with the Pacific-slope and
Choice (D) is incorrect because the author the Gold Rush of 1849; and choice (D) is
does not suggest that the government is associated with Colorado.
unable to determine a viable solution, only
17. The correct answer is (B). Choice (B) best
that the government seems uninterested or
describes the purpose of the majority of the
unwilling to do so.
paragraphs of the passage. The author notes
13. The correct answer is (A). The author is what others have thought or said about the
making two points. First, she is citing spe- shared history of these groups and makes the
cific places and times where some of the point that the United States has consistently
worst abuses occurred, and second, is de- mistreated Native Americans. Choice (A) is
scribing “countless” abuses over the course incorrect because the author is interested in
of a century. Choice (B) is incorrect because, building an argument rather than establish-
although specific states are mentioned, that is ing a chronological account. Choice (C) is
not the overall thrust of the passage. Choice incorrect because, while the author touches
(C) implies that because the dates cited are on some causes and effects, they are second-
not recent, the problems have stopped, yet ary to the argument. Choice (D) is inaccurate
the second half of the sentence explains that because the author is presenting only one
the government “breaks promises now as viewpoint, not opposing viewpoints.
deftly as then.”
18. The correct answer is (D). The author re-
14. The correct answer is (D). Lines 32–33 peatedly refers to concealing or explaining
make the point that “every page and every away the offenses of white settlers against
year has its dark stain.” This phrase sup- Native Americans. Choice (A) has the op-
ports the author’s assertion that abuse was posite meaning of “palliated.” Although
widespread throughout the century. Choice concealment sometimes takes the form of
(A) makes the point that there were many confusion, based on the context, choice (B)
tribes that were abused, but it does not refer- is incorrect because the sentences that follow
ence the time period in which these abuses the sentence contrast how offenses against
took place. Choice (B) stresses the abuse white settlers are spread with the way of-
specific to the Pacific-slope Indians but fenses against Native Americans are spread.
not the scope of the overall abuse. Choice “Palliated” can be used to refer to easing
(D) describes the whole scope of the abuse medical symptoms, but that meaning is not
but only with regard to broken treaties and applicable here, so choice (C) is incorrect.
unfulfilled promises.
19. The correct answer is (C). Choice (C)
15. The correct answer is (D). The correct is the best summary because it accurately
answer is Choice (D) because in lines states that the army admits they were the
22–24 (There … laws), the author talks aggressor. The few times Native Ameri-
about an area that was settled so quickly cans did something horrible, the events
that there wasn’t a chance to create laws. were played up as worse than they were,
Settlers could do whatever they wanted while the army’s crimes were played down.
to the tribes, unregulated by any treaty. Choice (A) gets some of the facts wrong and
Choice (A) explains general prejudices but makes assumptions that aren’t in the original

www.petersons.com
634 PART VII: Five Practice Tests

paragraph. Choice (B) does the same and ship. There is no language that suggests the
interprets “testimony” as literally referring author wished to persuade readers, choice
to a courtroom. Choice (D) is too simplistic (B). Although the author makes it clear
and leaves out much of the pertinent infor- how disadvantaged slaves were, the pas-
mation of the paragraph. sage does not focus on the evils of slavery,
20. The correct answer is A. Choice (A) choice (A). Although the passage includes
stresses the inequality in the way Native some information about the lives of slaves,
Americans were treated and any part they choice (D), its overall focus is the history
had in the conflict between the settlers tak- of the plantation.
ing their land and their attacking settlers. 24. The correct answer is (D). The text explains
Choice (B) is only one idea that is part of the that Mrs. Custis taught the slaves to read the
larger, more comprehensive theme. Choice Bible (lines 79–82) because she was a very
(C) may be true, but it is not really covered religious woman. There is no indication that
by the passage. Choice (D) is incorrect she wanted them to be able to teach the skills
because the passage clearly states that it is to anyone else, choices (A) and (C), or read
a continuing problem, not one that existed any other material, choice (B).
only in the past. 25. The correct answer is (B). The text states
21. The correct answer is (D) Choice (D) that Custis inherited slaves. It does not say
makes a sweeping statement of the situation how he acquired the land for his plantations,
and points out how one-sided the conflict so we cannot assume that it was granted to
between Native Americans and white set- him by Martha Washington, choice (A). The
tlers has been. Choice (A) focuses on the passage describes how Custis went about
fact that weaker tribes suffered more than building the house, so it’s safe to say that he
others, not that all tribes were victims. did not inherit a house with land and slaves,
Choice (B) focuses more on a timeline of choice (C). The text states that Custis had
abuse rather than who is to blame. Choice three plantations in all, but it does not say
(C) points out that all tribes were mistreated how he acquired them, choice (D).
in a like manner but doesn’t place the blame 26. The correct answer is (B). The text tells us
on white settlers. that Selina was given the keys to the planta-
22. The correct answer is (D). Choice (D) tion, which implies great trust in the days of
suggests the wildness of cruelty that white slavery. As head housekeeper, she may also
settlers perceived in the retaliation by Native have cared for the Lee children, choice (C),
Americans, which further spread fear and but that information is not in the passage.
hatred of the tribes and helped to publicize Being head of housekeeping is not the same
the need to remove them. Choice (A) mis- as running the whole plantation, choice (D);
interprets “unspeakable,” which refers to while housekeeping of a large mansion is
something so outrageous it shouldn’t be said hard work, the text doesn’t compare it to
out loud, instead of the nature of the acts the fieldwork of other slaves, choice (A).
themselves. Choice (B) references the idea 27. The correct answer is (C). From the blue-
of retaliation but does not characterize the print and the caption, it is clear that there were
acts correctly. Choice (C) refers to some of a lot of slaves housed in a small space, even
the anger and aggression Native Americans if an exact number is not supplied. It is one
felt in striking back but suggests careful house, but it’s hardly large, especially given
planning and strategy, an interpretation that the number of people living there, choice
is not supported by the passage. (A), and so it was not adequate, choice (B).
23. The correct answer is (C). The main theme It is not possible to tell from the drawing
in the passage describes the people who where the main house is located in relation
lived at Arlington House—slaves and own- to these slave quarters, choice (D).
ers—and includes information about how 28. The correct answer is (A). Robert E. Lee
the house was constructed and by whom as was related to George Washington through
well as the history of the plantation’s owner- marriage. Paragraph 5 explains the rela-

Master the New SAT®


Practice Test 2 635

answers PRACTICE TEST 2
tionship: Custis’s daughter, Mary Anna, of the tutoring being given in exchange for
married Robert E. Lee, and Lee came to something else as in choice (D).
Arlington to live with the Custis family. 33. The correct answer is (C). Both passages
Custis was the adopted grandson of Mar- agree that the amount of carbon dioxide in
tha and George Washington, which made our atmosphere has increased. However,
his daughter—Mary Anna—Washington’s choice (A) is disputed by the second pas-
great granddaughter. sage, and choice (B) is disputed by the first
29. The correct answer is (C). The first sen- passage. Choice (D) is refuted by the second
tence in the last paragraph states that the passage as being too complicated to prove
slaves would only be freed if the estate were or predict.
in good financial standing. The slaves were 34. The correct answer is (B). Only choice
not freed immediately because Lee deter- (B) contains the admission by the author of
mined that their labor was still needed to get the second passage that atmospheric carbon
his father-in-law’s financial affairs in order. dioxide is increasing, which the first pas-
The fact that Custis relied on Lee, choice sage also states. Choice (A) discusses some
(A), shows that Custis believed Lee was a of the effects of carbon dioxide levels but
good financial manager, but it doesn’t show does not confirm their increase. Choice (B)
that it is true. Lee freed the slaves, choice discusses the changes to the atmosphere
(D), because it was part of Custis’s wishes. without specifically mentioning increased
The reader does not learn whether or not Lee hydrogen levels. Choice (D) directly coun-
agreed with those wishes. While this is true ters any attempts to limit carbon dioxide but
that Mrs. Custis taught slaves how to read, doesn’t support the fact there are increases.
choice (B), the passage does not report how
35. The correct answer is (C). Both passages
Lee felt about this act.
address the reader as “we” or “our,” indicat-
30. The correct answer is (B). Custis could ing a shared interest among humans. Each
inherit slaves because they were legally passage is trying to persuade the reader to one
equivalent to any other property—a good viewpoint, thus making choice (A) incorrect.
that could be bought and sold. Choice (B) Choice (B) is too strong a description since
illustrates this point. Simply being able to both passages try to convince with logic
free a slave (D) doesn’t illustrate the con- rather than pure emotionalism. Choice (D)
cept of property ownership; it illustrates the is incorrect because there are no second-
concept of physical freedom. Choice (A) person or “you” statements in the passages.
is a general description of the complexity
36. The correct answer is (A). Neither pas-
of the relationships on the plantation, but
sage provides sources for the information,
doesn’t show how slaves were property to
observations, and conclusions that they
be bought and sold.
are advancing. Choice (B) ignores some of
31. The correct answer is (A). The “invisible the computer simulation that is discussed.
gulf” the author mentions is one of separa- Choice (C) is incorrect because at least one of
tion between slaves and owners in terms of the passages mentions dissenting opinions.
privilege and human rights. While “gulf” Choice (D) is only partially correct, as some
can describe a body of water, as in choice terms are defined at least through context.
(B); a chasm, as in choice (C); or a physical
37. The correct answer is (D). The purpose of
gap, as in choice (D), here the author’s use
the first passage is to convince readers that
is metaphorical.
global warming is a danger, while the second
32. The correct answer is (C). In this context, passage is attempting to refute any cause for
“afford” does not refer to money as in choice concern. Choice (A) misses the point that
(B), but to things provided to others. Since both passages use persuasive terms. Neither
slaves were property, and treated as such, few is limited to description or definition. Choice
owners thought about teaching them to read (B) is not applicable because both passages
or write. There is no risk implied in this use use a variety of terms. Choice (C) reverses
of the word, choice (A), or any suggestion the purpose of the two passages.

www.petersons.com
636 PART VII: Five Practice Tests

38. The correct answer is (D). The sentence re- the way in which “translate” is used here. The
fers to symptoms of climate change becom- effects the authors mention could include
ing more recognizable. “Pronounced” can expansion, but that is too narrow a defini-
refer to the way something is said; however, tion for the context, which makes choice
that meaning doesn’t fit the context, choice (B) incorrect. Choice (D) refers to helping
(A). Choice (C) has the opposite meaning someone understand something, which does
from the meaning of “pronounced” as used not fit the context of the sentence.
in the passage. “Pronounced” can refer to a 43. The correct answer is (B). The two women
declaration or announcement, choice (B), but are fulfilling a tradition common to many
that meaning also does not fit the context. cultures, in which the living stay with the
39. The correct answer is (B). Although the body of a friend or family member so it
authors of these passages disagree, they are doesn’t have to be alone. The tradition partly
focused on the causes and effects of certain grew out of the possibility that the deceased
problems, choice (B), not solutions, choice might revive, as sometimes happened in the
(D). Choice (A) suggests a timeline of events. days before doctors were readily available.
Although both authors reference past and Choice (A) is incorrect; it is clear that word
future events, information is not presented has already spread that Miss Tempy died.
sequentially. Choice (C) is something the Choice (C) has no supporting evidence in
reader might unconsciously do, but the the story. Choice (D) is a presumption that
authors don’t take this approach. is also unsupported by details in the story.
40. The correct answer is (C). The first pas- 44. The correct answer is (A). While perform-
sage focuses only on the negative effects ing a duty to a friend, two of her oldest,
of atmospheric hydrogen, and the second dearest friends draw closer to each other.
passage states that atmospheric carbon Choice (B) is incorrect because the focus
dioxide does not have a negative effect. of the story is the relationship between the
Choice (A) is about human control, which two remaining friends. Choice (C) may be
is only a minor element of each argument. true but is incidental. Choice (D) may also
Choice (B) focuses on the weather and be true but is not central to the story.
climate argument, which is also a lesser 45. The correct answer is (A). Mrs. Crowe
issue. Choice (D) references hydrocarbon forgets to pay attention to her knitting be-
production, which is only one part of the cause she’s so enjoying her talk with Sarah
larger argument. Ann Binson. Sarah Ann Binson, on the
41. The correct answer is (C). The author of other hand, abandons her sewing completely
the second passage is quoting those he or when she’s preoccupied. Choice (B) is a
she disagrees with, such as the author of the presumption unsupported by the details of
first passage. Choice (A) only highlights the the story. Choice (C) is contradicted by the
sources of greenhouse gases and does not way the women are portrayed in the story.
encapsulate the major differences between Although both women are elderly, the author
these arguments. Choice (B) focuses on portrays them as energetic and intelligent,
the long-term effects of carbon dioxide albeit somewhat eccentric.
but doesn’t summarize the two competing 46. The correct answer is (D). Choice (D)
arguments. Choice (D) focuses only on shows that under this special circumstance,
beneficial effects of increased atmospheric the women are bonding. Choice (A) is
carbon dioxide. a minor detail that doesn’t reflect their
42. The correct answer is (C). Choice (C) cor- friendship. Choice (B) does not happen in
rectly fits with the idea of changes becoming the story, although it is mentioned as a trait
shifts and suggests that the effects of climate of Mrs. Crowe’s. Choice (C) is part of the
change may be unpredictable. Language author’s description of Sarah Ann, but it
translation, choice (A), is the process of doesn’t indicate closeness to Mrs. Crowe.
converting words and meaning from one 47. The correct answer is (A). Choice (A)
language into another; however, that is not best shows that, despite her burdens, Sarah

Master the New SAT®


Practice Test 2 637

answers PRACTICE TEST 2
Ann Binson could be excited, expressive, but this detail isn’t mentioned. Choice (C)
and confident. Choice (B) merely describes is contradicted by the fact that they work
Sarah Ann’s situation and doesn’t provide well together.
insight into other aspects of her life. Choice 50. The correct answer is (B). The sharing
(C) spells out what she had done for her of secrets is evidence that the two women
relatives, but it doesn’t tell us much about are becoming good friends despite their
her joy. Choice (D) provides details about different positions in life. Choice (A) only
Mrs. Crowe but not about Sarah Ann Binson. discusses sharing memories, which even
48. The correct answer is (B). During the long distant acquaintances might do. Choice (C)
night together talking, both women reveal describes their dispositions but doesn’t give
secrets and say things they would never have much insight into their friendships. Choice
said under normal circumstances. Choice (A) (D) only describes Mrs. Crowe and doesn’t
is a supposition the author makes, presuming explain their friendship.
they wouldn’t have enough to talk about all 51. The correct answer is (A). Sarah Ann
night long, but that doesn’t prove to be the would have to be eager and hungry to carry
case. Choice (C) reflects the characterization the load of her family as discussed in the
of Mrs. Crowe as “knowing exactly what she rest of the sentence. Although Sarah Ann
was about” and “a cooler disposition,” yet may be restless, the focus of the sentence is
she is so involved in her conversation that on how much responsibility she shoulders
she doesn’t notice she is knitting a sock that for her relatives, which makes choice (B)
is too long. Choice (D) reflects the author’s incorrect. Choice (C) is not a term usually
comment that her life is filled with pleasure, applied to a person. Choice (D) applies more
but she also says that, to outward appear- closely to Mrs. Crowe as she is described
ances, Sarah Ann Binson was burdened by later in the passage.
caring for her family.
52. The correct answer is (C). Giving some-
49. The correct answer is (D). Choice (D) is thing away is being generous or magnani-
correct because despite Mrs. Crowe’s higher mous. Choice (A) is a confusion with a piece
social standing, the women easily fill a long of furniture. Choice (B) is a confusion with
night with intimate talk. Choice (A) may lifting a finger or helping someone. Choice
be true, but the reader couldn’t know this. (D) is a confusion with thoroughness being
Choice (B) assumes they are the same age, an act of kindness.

www.petersons.com
Practice Test 3 673

practice test
SECTION 1: READING TEST
65 Minutes • 52 Questions

TURN TO SECTION 1 OF YOUR ANSWER SHEET TO ANSWER THE QUESTIONS IN THIS


SECTION.

Directions: Each passage (or pair of passages) below is followed by a number of multiple-choice
questions. After reading each passage, select the best answer to each question based on what is
stated or implied in the passage or passages and in any supplementary material, such as a table,
graph, chart, or photograph.

QUESTIONS 1–11 ARE BASED ON THE 30 the surface of the fault holds the rocks
FOLLOWING PASSAGE. together so they do not slip immediately
when pushed sideways. Eventually
The U.S. Geological Survey (USGS) is a enough stress builds up and the rocks
government agency whose goal is to provide slip suddenly, releasing energy in waves
reliable scientific information about the 35 that travel through the rock to cause the
Earth, including minimizing loss from natural shaking that we feel during an earthquake.
disasters. This excerpt is from the organiza- Earthquakes typically originate
tion’s website. For the full passage, please visit several to tens of miles below the
http://earthquake.usgs.gov/learn. surface of the Earth. It takes decades
Earthquakes, Megaquakes, and the Movies 40 to centuries to build up enough stress
Throughout the history of Hollywood, to make a large earthquake, and the
disaster films have been sure-fire winners fault may be tens to hundreds of
for moviemakers …. With amazing special miles long. People cannot prevent
Line effects, it’s easy to get caught up in the earthquakes from happening or stop
5 fantasy disaster epic. What makes a great 45 them once they’ve started—giant nuclear
science fantasy film often bears no relation explosions at shallow depths, like those
to real facts or the hazards people truly face. in some movies, won’t actually stop an
The U.S. Geological Survey is the earthquake.
lead federal agency responsible for It’s well known that California, the
10 researching, monitoring and forecasting 50 Pacific Northwest, and Alaska all have
geologic hazards such as earthquakes, frequent earthquakes, some of which
volcanoes and landslides …. Let’s start are quite damaging. Some areas of the
with some science-based information on country are more at risk than others,
earthquakes. but, in fact, 42 of the 50 states could
15 Earthquakes are naturally occurring 55 experience damaging ground shaking
events outside the powers of humans from an earthquake in 50 years (which is
to create or stop. An earthquake is the typical lifetime of a building), and 16
caused by a sudden slip on a fault, much states have a relatively high likelihood of
like what happens when you snap your experiencing damaging ground shaking.
20 fingers. Before the snap, you push your 60 The two most important variables
fingers together and sideways. Because affecting earthquake damage are the
you are pushing them together, friction intensity of ground shaking and the
keeps them from slipping. When you quality of the engineering of structures
apply enough stress to overcome this in the region. The level of shaking
25 friction, your fingers move suddenly, 65 is controlled by the proximity of the
releasing energy. The same “stick-slip” earthquake source to the affected region
process goes on in the earth. Stresses and the types of rocks that seismic waves
in the Earth’s outer layer push the sides pass through en route (particularly those
of the fault together. The friction across at or near the ground surface). Generally,
GO TO THE
NEXT PAGE
www.petersons.com
674 PART VII: Five Practice Tests

70 the bigger and closer the earthquake, 1 Which of the following best describes the
the stronger the shaking. But there author’s purpose in writing this article?
have been large earthquakes with very
little damage because they caused little (A) To counter the myths about
shaking or because the buildings were earthquakes driven by fictional films
75 built to withstand that shaking. In other (B) To explain to people the causes and
cases, moderate earthquakes have caused effects of earthquakes
significant damage because the shaking
(C) To show how Hollywood distorts
was locally amplified, or because the
science
structures were poorly engineered.
80 The idea of a “Mega-Quake”—an (D) To give people advice about what to
earthquake of magnitude 10 or larger—is do if an earthquake strikes
very unlikely. Earthquake magnitude is
based in part on the length of faults— 2 What is the main cause of an earthquake?
the longer the fault, the larger the
85 earthquake. The simple truth is that (A) A fault in the Earth’s crust
there are no known faults capable of (B) Friction caused by stresses built up in
generating a magnitude 10 or larger rocks
“mega-quake.” …
(C) Stresses in the Earth’s outer layer
Then there’s this business of
90 California falling off into the ocean. (D) Energy waves through the rocks in
NOT TRUE! The ocean is not a the Earth’s crust
great hole into which California can
fall, but is itself land at a somewhat 3 Who is the target audience of the article?
lower elevation with water above it.
95 It’s impossible that California will be (A) The general public
swept out to sea. Instead, southwestern (B) Scientists
California is moving slowly (2 inches
(C) Filmmakers
per year) towards Alaska. 15 million
years (and many earthquakes) from now, (D) Science teachers
100 Los Angeles and San Francisco will be
next-door neighbors. 4 How does the article counter the claim that
Another popular cinematic and literary in the future, part of California may fall off
device is a fault that opens during an into the ocean?
earthquake to swallow up an inconvenient
105 character. But the ground moves parallel (A) It states that it would be a disaster.
to a fault during an earthquake, not away (B) It states that sea level rise will
from it. If the fault could open, there prevent it.
would be no friction. Without friction,
(C) It explains the similarity between
there would be no earthquake. Shallow
California and Alaska.
110 crevasses can form during earthquake-
induced landslides, lateral spreads, or (D) It explains how the ocean is just land
other types of ground failures. Faults, covered by water.
however, do not gape open during an
earthquake.
115 So when you see the next big disaster
\ film, rest assured that movies are just
entertainment. Enjoy them! And then go
learn about the real-world science behind
disasters, and if you live in an area where
120 hazards exist, take the suggested steps to
protect you and your family.

Master the New SAT®


Practice Test 3 675

practice test
5 Which choice provides the best evidence 9 In line 78, “amplified” most nearly means
for the answer to the previous question?
(A) lifted.
(A) Lines 91–94 (“The ocean … above (B) supplemented.
it.”)
(C) intensified.
(B) Lines 95–96 (“It’s impossible … out
(D) augured.
to sea.”)
(C) Lines 96–98 (“Instead,
10 How does the use of the phrase “inconve-
southwestern … towards Alaska.”)
nient character”(lines 104–105) affect the
(D) Lines 98–101 (“15 million years …
tone of the passage?
neighbors.”)
(A) It reveals a negative attitude about
6 As used in line 24, “stress” most nearly unscientific data.
means (B) It illustrates a mocking tone toward
how the storylines are written.
(A) anxiety.
(C) It reveals a scholarly attitude about
(B) weight. science.
(C) pressure. (D) It communicates a warning about
(D) emphasis. inaccurate scientific information.

7 Based only on information in the article, 11 How does the author refute the idea that an
what is the most likely reason that Haiti earthquake could cause the earth to open
experienced such extensive damage in the up and swallow people and things?
earthquake of 2010?
(A) By pointing out that the idea of the
(A) The earthquake source was very near Earth opening up is portrayed in
the affected region and the buildings movies
were poorly constructed. (B) By noting that there are no known
(B) Haiti has had earthquakes many faults capable of producing a “mega
times before, and they were all quake”
destructive. (C) By explaining that the ground
(C) The people are poor and were moves parallel to a fault during an
unprepared for the earthquake of earthquake
2010. (D) By suggesting that subsequent
(D) The faults were deep and numerous landslides can cause crevasses to
across the country. open up

8 Which choice provides the best evidence


for the answer to the previous question?

(A) Lines 17–29 (“An earthquake is …


the fault together.”)
(B) Lines 37–43 (“Earthquakes
typically … miles long.”)
(C) Lines 60–64 (“The two most
important … in the region.”)
(D) Lines 105–108 (“But the ground …
be no friction”)
GO TO THE
NEXT PAGE
www.petersons.com
676 PART VII: Five Practice Tests

QUESTIONS 12–22 ARE BASED ON THE supposition that your American governors
FOLLOWING TWO PASSAGES. shall be honest that all the good qualities
of this government are founded; but its
After the Constitution was drafted, it had to be defective and imperfect construction puts
ratified by at least nine of the thirteen the states. 40 it in their power to perpetrate the worst
The following two passages illustrate the debate of mischiefs should they be bad men;
over ratification. Passage 1 is from a speech made and, sir, would not all the world, blame
on June 5, 1788 by Patrick Henry, the governor our distracted folly in resting our rights
of Virginia, at the state’s convention, called upon the contingency of our rulers being
specifically to ratify the Constitution. Passage 45 good or bad? Show me that age and
2 is from an essay written by James Madison, country where the rights and liberties of
which first appeared in a New York newspaper the people were placed on the sole chance
on June 6, 1788, and later became part of what of their rulers being good men without
is now known as the Federalist Papers. a consequent loss of liberty! I say that
Passage 1 50 the loss of that dearest privilege has ever
If you make the citizens of this followed, with absolute certainty, every
country agree to become the subjects such mad attempt.
of one great consolidated empire of Passage 2
Line America, your government will not have
In order to lay a due foundation for
5 sufficient energy to keep them together.
that separate and distinct exercise of the
Such a government is incompatible 55 different powers of government, which to
with the genius of republicanism. There a certain extent is admitted on all hands
will be no checks, no real balances, in to be essential to the preservation of
this government. What can avail your liberty; it is evident that each department
10 specious, imaginary balances, your rope-
should have a will of its own; and
dancing, chain-rattling ridiculous ideal 60 consequently should be so constituted that
checks and contrivances? But, sir, “we are the members of each should have as little
not feared by foreigners; we do not make agency as possible in the appointment
nations tremble.” Would this constitute of the members of the others …. It is
15 happiness or secure liberty? I trust, sir,
equally evident that the members of each
our political hemisphere will ever direct 65 department should be as little dependent
their operations to the security of those as possible on those of the others for the
objects. emoluments annexed to their offices.
This Constitution is said to have Were the executive magistrate, or the
20 beautiful features; but when I come to
judges, not independent of the legislature
examine these features, sir, they appear 70 in this particular, their independence in
to me horribly frightful. Among other every other would be merely nominal.
deformities, it has an awful squinting; But the great security against a gradual
it squints toward monarchy, and does concentration of the several powers in
25 not this raise indignation in the breast
the same department, consists in giving
of every true American? Your president 75 to those who administer each department
may easily become king. Your Senate the necessary constitutional means and
is so imperfectly constructed that your personal motives to resist encroachments
dearest rights may be sacrificed to what of the others. The provision for defense
30 may be a small minority; and a very
must in this, as in all other cases, be made
small minority may continue for ever 80 commensurate to the danger of attack.
unchangeably this government, altho Ambition must be made to counteract
horridly defective. Where are your checks ambition. The interest of the man must be
in this government? Your strongholds will connected with the constitutional rights
35 be in the hands of your enemies. It is on a
of the place. It may be a reflection on

Master the New SAT®


Practice Test 3 677

practice test
85 human nature, that such devices should 12 Which of the following best represents the
be necessary to control the abuses of differences in point of view of the authors
government. But what is government of the two passages?
itself, but the greatest of all reflections
on human nature? If men were angels, no (A) Henry was concerned about the
90 government would be necessary. If angels balance of power and Madison was
were to govern men, neither external nor concerned about concentration of
internal controls on government would be wealth.
necessary. In framing a government which (B) Henry worried about too much power
is to be administered by men over men, in the hands of the government
95 the great difficulty lies in this: you must and Madison worried about too
first enable the government to control the much power in any one branch of
governed; and in the next place oblige government.
it to control itself. A dependence on the
(C) Henry was focused on states’ rights
people is, no doubt, the primary control
and Madison was focused on adding
100 on the government; but experience has
the Bill of Rights to the Constitution.
taught mankind the necessity of auxiliary
precautions. (D) Henry was afraid of a return to
monarchy and Madison was afraid of
government corruption.

13 Which represents the best summary of


Patrick Henry’s objection to the drafted
Constitution?

(A) Its structure is unstable and will lead


to loss of liberty.
(B) It leaves too much power in the
hands of the people.
(C) It does not centralize power enough.
(D) It makes government dependent on
people who are flawed.

GO TO THE
NEXT PAGE
www.petersons.com
678 PART VII: Five Practice Tests

14 Examine the illustration that shows Patrick Henry delivering his most famous speech; the one
in which he declared, “Give me liberty or give me death!” Choose the option that best explains
how the artist’s portrayal applies to Henry’s speech about the Constitution.

(A) The artist shows Henry as an important (C) The artist shows Henry standing on
man with a message, surrounded by his feet making dramatic gestures and
many people who react strongly his speaking with passion. In the speech
speech. Henry delivers his speech to about the Constitution, Henry uses
the state Assembly as an important dramatic phrases and figures of speech
figure—governor of Virginia. to try to persuade his listeners.
(B) The artist shows Henry as a calm, (D) The artist shows Henry as an extremist
persuasive speaker, using logic and so carried away by his own emotions
reasoning to persuade his listeners. that people think he’s a madman. This
Henry uses order and reasoning in view contrasts sharply with Henry’s
his speech about the Constitution. He speech, in which he slowly builds his
asks his listeners logical questions to argument, appealing to the listener’s
persuade them that he is right. sense of patriotism.

Master the New SAT®


Practice Test 3 679

practice test
15 What was Madison’s strongest counter- 17 What do these two statements show about
argument to those who were concerned how their authors viewed human nature?
about a strong central government?
Henry, lines 45–49: “Show me that age and
(A) Men are inherently flawed. country where the rights and liberties of
the people were placed on the sole chance
(B) No one is perfect.
of their rulers being good men without a
(C) Government would ultimately be consequent loss of liberty!”
controlled by the people.
Madison, lines 90–93: “If angels were
(D) So long as the powers are separated,
to govern men, neither external nor in-
power will not be concentrated.
ternal controls on government would be
necessary.”
16 Which choice provides the best evidence
for the answer to the previous question? (A) Henry believed humans are naturally
dishonest and self-interested and
(A) Lines 72–78 (“But the great … of the Madison believed that humans are
others.”) basically honest and good, but not
(B) Lines 81–82 (“Ambition must … perfect.
counteract ambition.”) (B) Henry believed past rulers were
(C) Lines 89–90 (“If men were … would generally good men who lost their
be necessary.”) way; Madison believed humans tend
to be dishonest.
(D) Lines 98–102 (“A dependence on …
auxiliary precautions.”) (C) Henry didn’t trust ordinary men to
be rulers and Madison believed all
people could be trusted to rule.
(D) Henry believed that government
is unnecessary for a free people;
Madison believed that government
needs to be regulated.

18 Who is Patrick Henry quoting in lines


12–14 But, sir, “we are not feared by for-
eigners; we do not make nations tremble”?

(A) An omniscient narrator


(B) The framers of the Constitution
(C) The general public
(D) The people of Virginia

19 What kind of government does Henry


think is best?

(A) Monarchy
(B) Republic
(C) Autocracy
(D) Plutocracy

GO TO THE
NEXT PAGE
www.petersons.com
680 PART VII: Five Practice Tests

20 Which choice provides the best evidence This species common name is the
for the answer to the previous question? “pocket shark,” though those in the
10 field of classifying animals refer to it
(A) Lines 3–4 (“one great … of by its scientific name Mollisquama sp.,
America.”) according to a new study published in
(B) Line 7 (“the genius … the international journal of taxonomy
republicanism.”) Zootaxa. While it is small enough to,
15 yes, fit in your pocket, it’s dubbed
(C) Line 24 (“it squints … monarchy”)
“pocket” because of the distinctive orifice
(D) Lines 26–27 (“Your president … above the pectoral fin—one of many
become king.”) physiological features scientists hope to
better understand.
21 As it is used in line 58, “department” most 20 “The pocket shark we found was
nearly means only 5 and a half inches long, and was
a recently born male,” said Mark Grace
(A) an executive in the government. of NOAA Fisheries’ Pascagoula, Miss.,
(B) an office in the government. Laboratory, lead author of the new
25 study, who noted the shark displayed an
(C) a level of government.
unhealed umbilical scar. “Discovering
(D) a branch of government. him has us thinking about where mom
and dad may be, and how they got to the
22 As used in line 80, “commensurate” most Gulf. The only other known specimen
nearly means 30 was found very far away, off Peru, 36
years ago.”
(A) proportional. Interestingly, the specimen Grace
(B) provisional. discovered wasn’t found it the ocean, per
se, but rather in the holdings of NOAA’s
(C) dependent.
35 lab in Pascagoula. It was collected in
(D) relevant. the deep sea about 190 miles offshore
Louisiana during a 2010 mission by the
QUESTIONS 23–32 ARE BASED ON NOAA Ship Pisces to study sperm whale
THE FOLLOWING PASSAGE AND feeding. Grace, who was part of that
SUPPLEMENTARY MATERIAL. 40 mission after the rare shark was collected,
and upon uncovering the sample at the lab
This article is excerpted from the National years later, recruited Tulane University
Oceanic and Atmospheric Administration researchers Michael Doosey and Henry
Fisheries website (NOAA). The public agency Bart, and NOAA Ocean Service genetics
provides science news and scientific findings 45 expert Gavin Naylor, to give the specimen
related to the Earth and the Earth’s atmosphere. an up-close examination.
This article describes the finding of a tiny, rare A tissue sample was collected, and
shark. For the full passage, please visit www. by tapping into the robust specimen
nmfs.noaa.gov/stories. collection of Tulane University’s
NOAA and Tulane researchers identify 50 Biodiversity Research Institute, scientists
second possible specimen ever found. were able to place the specimen into
A very small and rare species of shark the genus Mollisquama. Further genetic
is swimming its way through scientific analysis from Naylor indicates that
literature. But don’t worry, the chances of pocket sharks are closely related to the
Line this inches-long vertebrate biting through 55 kitefin and cookie cutter species, fellow
5 your swimsuit is extremely slim, because members of the shark family Dalatiidae.
if you ever spotted one, you’d be the third Like other Dalatiidae shark species it is
person to ever do so. possible that pocket sharks when hungry
may remove an oval plug of flesh from

Master the New SAT®


Practice Test 3 681

practice test
60 their prey (various marine mammals, 25 Why was the finding reported in a journal
large fishes, and squid). of taxonomy?
The specimen is part of the Royal
D. Suttkus Fish Collection at Tulane (A) Because scientists wanted to test the
University’s Biodiversity Research DNA of this shark
65 Institute in Belle Chasse, La., and it is (B) Because the scientific community
hoped that further study of the specimen was excited about a new species
will lead to many new discoveries.
(C) Because the journal specializes in
Already, the specimen—when compared
marine animals
to the 1979 specimen taxonomic
70 description—is found to have a series of (D) Because scientists wanted to report
glands along the abdomen not previously such a rare species and how they
noted. Partners at the Smithsonian classified it
National Museum of Natural History in
Washington, D.C., and American Natural 26 Which choice provides the best evidence
75 History Museum in New York City have for the answer to the previous question?
also contributed to the study of this shark.
“This record of such an unusual and (A) Lines 25–26 (“the shark … umbilical
extremely rare fish is exciting, but it’s scar.”)
also an important reminder that we still (B) Lines 50–56 (“scientists were …
80 have much to learn about the species that Dalatiidae.”)
inhabit our oceans,” Grace added.
(C) Lines 66–67 (“the specimen … new
discoveries.”)
23 What does the article illustrate about how
(D) Lines 68–72 (“Already, the …
scientific information is gathered? previously noted.”)
(A) Sometimes information can be lost
for years but may be useful when 27 What information did scientists need to
found and shared. determine the species of the shark?
(B) Scientists conduct many different
(A) The length of the shark
kinds of tests on animals to get
information about them. (B) The “pocket” feature that made it
unique
(C) All scientific research is recorded in
journals. (C) Tissue samples to provide genetic
information
(D) Scientists in different locations often
share their findings. (D) Its position in the food chain

24 Why were scientists surprised at finding


the shark?

(A) Scientists didn’t know about it


before.
(B) It was discovered by accident in a
lab.
(C) It was thought to be extinct.
(D) Scientists thought it lived in colder
waters.

GO TO THE
NEXT PAGE
www.petersons.com
682 PART VII: Five Practice Tests

Credit: Dr. Mark Grace, Zootaxa 3948 (3): 587–600

28 Based on the diagram above, where is the 30 Which choice provides the best evidence
shark’s pocket located? for the answer to the previous question?

(A) Above the pectoral fin (A) Lines 32–35 (“Interestingly … in


(B) Between the eye and the pectoral fin Pascagoula”)
(C) Above the eye on both sides of the (B) Lines 65–67 (“it is hoped that … new
shark discoveries”)
(D) In the spiracle (C) Lines 52–55 (“Further genetic …
cutter species”)
29 Why were scientists excited about the find? (D) Lines 72–76 (“Partners at the … of
this shark”)
(A) They expect to learn more about the
habitat of the pocket shark. 31 Based on its use in line 48, “robust” most
(B) They will be able to learn more about nearly means
what they eat.
(A) typical.
(C) They can figure out how they are
related to other sharks. (B) healthy.
(D) They expect to be able to discover (C) varied.
many new things about sharks. (D) distinguished.

32 As it is used in line 59, “plug” most nearly


means

(A) lump.
(B) a protrusion.
(C) obstruction.
(D) scar.
Master the New SAT®
Practice Test 3 683

practice test
QUESTIONS 33–42 ARE BASED ON THE 45 as it meanders along for no given reason;
FOLLOWING PASSAGE. and yet when a stream acts like that it
ought to be required to explain itself.
The following passage is excerpted from Mark Cooper narrowed it to less than twenty to
Twain’s essay, “Fenimore Cooper’s Literary accommodate some Indians. He bends a
Offenses” (1895). James Fenimore Cooper 50 “sapling” to the form of an arch over this
(1789–1851) was a popular writer whose works narrow passage, and conceals six Indians in
were published in the first half of the 19th century. its foliage. They are “laying” for a settler’s
[James Fenimore] Cooper’s art has scow or ark, which is coming up the stream
some defects. In one place in Deerslayer, on its way to the lake. Its rate of progress
and in the restricted space of two-thirds 55 cannot be more than a mile an hour.
Line of a page, Cooper has scored 114 offences The ark is one hundred and forty feet
5 against literary art out of a possible 115. long; the dwelling is ninety feet long. The
…In his little box of stage properties idea of the Indians is to drop softly and
he kept six or eight cunning devices. A secretly from the arched sapling as the ark
favorite one was to make a moccasined 60 creeps along under it. It will take the ark
person tread in the tracks of the a minute and a half to pass under. It will
10 moccasined enemy, and thus hide his take the ninety-foot dwelling a minute to
own trail. Cooper wore out barrels and pass under. Now, then, what did the six
barrels of moccasins in working that trick. Indians do? It would take you thirty years
Another stage-property that he pulled 65 to guess. Their chief warily watched the
out of his box pretty frequently was his canal-boat as it squeezed along under
15 broken twig. Every time a Cooper person him, and when he had got his calculations
is in peril, and absolute silence is worth fined down to exactly the right shade, as
four dollars a minute, he is sure to step he judged, he let go and dropped. And
on a dry twig. There may be a hundred 70 missed the house! He missed the house
handier things to step on, but … Cooper and landed in the stern of the scow. It
20 requires him to turn out and find a dry was not much of a fall, yet it knocked
twig; and if he can’t do it, go and borrow him silly. He lay there unconscious. If the
one. In fact, the Leather Stocking Series house had been ninety-seven feet long he
ought to have been called the Broken 75 would have made the trip. The fault was
Twig Series. Cooper’s, not his. There still remained in
25 If Cooper had any real knowledge of the roost five Indians.
Nature’s ways of doing things, he had a The boat has passed under and is now
most delicate art in concealing the fact. For out of their reach. Let me explain what
instance: one of his acute Indian experts, 80 the five did—you would not be able to
Chingachgook (pronounced Chicago, I reason it out for yourself. No. 1 jumped
30 think), has lost the trail of a person he is for the boat, but fell in the water astern
tracking through the forest. Neither you of it. Then No. 2 jumped for the boat, but
nor I could ever have guessed out the fell in the water still farther astern of it.
way to find it. It was very different with 85 Then No. 3 jumped for the boat and fell a
Chicago. He turned a running stream out of good way astern of it. Then No. 4 jumped
35 its course, and there, in the slush in its old for the boat and fell in the water away
bed, were that person’s moccasin-tracks. astern. Then even No. 5 made a jump for
The current did not wash them away, as it the boat—for he was a Cooper Indian ….
would have done in all other like cases— 90 I may be mistaken, but it does seem
no, even the eternal laws of Nature have to me that Deerslayer is not a work of art
40 to vacate when Cooper wants to put up a in any sense; it does seem to me that it is
delicate job of woodcraft on the reader. destitute of every detail that goes to the
…In the Deerslayer tale, Cooper has making of a work of art; in truth, it seems
a stream which is fifty feet wide where it 95 to me that Deerslayer is just simply a
flows out of a lake; it narrows to twenty literary delirium tremens.
GO TO THE
NEXT PAGE
www.petersons.com
684 PART VII: Five Practice Tests

33 What is the most likely reason that Twain 37 Why did Twain include a pronunciation
wrote this essay? guide to the Native American Indian name
Chingachgook (line 29)?
(A) To point out flaws in Cooper’s works
(B) To make fun of a writer he didn’t like (A) To make it easier for people to read
his essay
(C) To criticize Cooper’s treatment of
Native Americans (B) To mock Cooper’s plot
(D) To show the importance of literary (C) To show respect for the Native
criticism American tribe
(D) To illustrate that Cooper’s work was
34 Which of the following words best de- about actual Native American peoples
scribe the tone of Twain’s critique?
38 Overall, what is the basic criticism Twain
(A) Arrogant had of Cooper’s work?
(B) Scholarly
(A) His books are too long and
(C) Cheery
complicated.
(D) Humorous
(B) His work has no artistic or literary
value.
35 Which choice provides the best evidence
(C) His plots are unrealistic and not
for the answer to the previous question? believable.
(A) Lines 4–5 (“Cooper has … possible (D) His stories are boring.
115.”)
(B) Lines 11–12 (“Cooper wore … that 39 Which choice provides the best evidence
trick”) for the answer to the previous question?
(C) Line 89 (“for he … Cooper Indian”)
(A) Lines 4–5 (“Cooper has … possible
(D) Lines 92–94 (“it does seem … work 115.”)
of art”)
(B) Lines 48–49 (“Cooper narrowed …
some Indians.”)
36 As used in line 6, “stage properties” most
(C) Lines 79–81 (“Let me … for
nearly means yourself.”)
(A) literary tricks. (D) Lines 88–89 (“Then even … Cooper
Indian.”)
(B) dramatic license.
(C) stage rules.
40 According to Twain’s criticism in the pas-
(D) scripts for a drama.
sage, how did Cooper characterize Native
Americans?

(A) They were athletic and smart.


(B) They were simple-minded and inept.
(C) They were awkward and sneaky.
(D) They were sneaky and agile.

Master the New SAT®


Practice Test 3 685

practice test
41 In line 50, Twain most likely puts “sapling” continually used them in a conscious,
in quotation marks to emphasize that theatrical sort of way, peculiarly offensive
25 in a boy. The pupils were abnormally
(A) the word is not generally used large, as though he was addicted to
anymore. belladonna, but there was a glassy glitter
(B) Cooper uses the word excessively in about them which that drug does not
his works. produce.
30 When questioned by the Principal as
(C) the description of the tree does not
to why he was there Paul stated, politely
sound like a sapling.
enough, that he wanted to come back
(D) sapling is a Latin word used to to school. This was a lie, but Paul was
describe certain kinds of trees. quite accustomed to lying; found it,
35 indeed, indispensable for overcoming
42 As used in line 93, “destitute” most nearly friction. His teachers were asked to state
means their respective charges against him,
which they did with such a rancor and
(A) impoverished. aggrievedness as evinced that this was not
(B) full. 40 a usual case. Disorder and impertinence
were among the offenses named, yet
(C) void.
each of his instructors felt that it was
(D) capable. scarcely possible to put into words the
cause of the trouble, which lay in a sort of
QUESTIONS 43–52 ARE BASED ON THE 45 hysterically defiant manner of the boy’s;
FOLLOWING PASSAGE. in the contempt which they all knew he
felt for them, and which he seemingly
Pulitzer prize-winning writer Willa Cather made not the least effort to conceal. Once,
worked as a reporter and also wrote several when he had been making a synopsis
novels and short stories. This excerpt is from one 50 of a paragraph at the blackboard, his
of her more popular short stories, written in 1905. English teacher had stepped to his side
Paul’s Case: A Study in Temperament and attempted to guide his hand. Paul
It was Paul’s afternoon to appear had started back with a shudder and
before the faculty of the Pittsburgh thrust his hands violently behind him.
High School to account for his various 55 The astonished woman could scarcely
Line misdemeanors. He had been suspended have been more hurt and embarrassed
5 a week ago, and his father had called at had he struck at her. The insult was so
the Principal’s office and confessed his involuntary and definitely personal as to
perplexity about his son. Paul entered be unforgettable. In one way and another
the faculty room suave and smiling. His 60 he had made all of his teachers, men
clothes were a trifle outgrown, and the tan and women alike, conscious of the same
10 velvet on the collar of his open overcoat feeling of physical aversion. In one class
was frayed and worn; but for all that there he habitually sat with his hand shading his
was something of the dandy in him, and eyes; in another he always looked out the
he wore an opal pin in his neatly knotted 65 window during the recitation; in another
black four-in-hand, and a red carnation in he made a running commentary on the
15 his buttonhole. This latter adornment the lecture, with humorous intention.
faculty somehow felt was not properly His teachers felt this afternoon that
significant of the contrite spirit befitting a his whole attitude was symbolized by
boy under the ban of suspension. 70 his shrug and his flippantly red carnation
Paul was tall for his age and very flower, and they fell upon him without
20 thin, with high, cramped shoulders and a mercy, his English teacher leading the
narrow chest. His eyes were remarkable pack. He stood through it smiling, his
for a certain hysterical brilliancy, and he pale lips parted over his white teeth. (His
GO TO THE
NEXT PAGE
www.petersons.com
686 PART VII: Five Practice Tests

75 lips were constantly twitching, and he 46 We can infer from the passage that the feel-
had a habit of raising his eyebrows that ings of Paul’s teachers toward him may be
was contemptuous and irritating to the described as mostly
last degree.) Older boys than Paul had
broken down and shed tears under that (A) frustration and anger.
80 baptism of fire, but his set smile did not (B) sadness and confusion.
once desert him, and his only sign of
(C) scorn and disdain.
discomfort was the nervous trembling of
the fingers that toyed with the buttons of (D) hope and tenderness.
his overcoat, and an occasional jerking of
85 the other hand that held his hat. Paul was 47 Which choice provides the best evidence
always smiling, always glancing about for the answer to the previous question?
him, seeming to feel that people might
be watching him and trying to detect (A) Lines 4–7 (“He had been … about his
something. This conscious expression, son.”)
90 since it was as far as possible from boyish (B) Lines 15–18 (“This latter … ban of
mirthfulness, was usually attributed to suspension.”)
insolence or “smartness.”
(C) Lines 36–40 (“His teachers … not a
usual case.”)
43 Based on the passage, what is the best way
(D) Lines 59–62 (“In one way …
to describe the story? physical aversion.”)
(A) It’s a character study.
48 From whose perspective or viewpoint is
(B) It’s an account of a real-life character.
the story told?
(C) It’s a psychological story with a
complicated plot. (A) Paul
(D) It’s a story that emphasizes the (B) The narrator
setting.
(C) A psychologist
(D) The school principal
44 What was the reason that Paul was asked
to go to the principal’s office?
49 How does Cather show a connection be-
(A) To explain to the teachers why he tween Paul’s feelings and his actions?
wanted to return to school
(A) She describes how subtle signals
(B) To explain to the faculty why he had
reflect Paul’s mood or disposition.
been misbehaving
(B) She describes his reactions compared
(C) To explain to the principal why he
to how others would react in similar
was late for class
circumstances.
(D) To explain to his parents why he had
(C) She gives details about his physical
been suspended
appearance and that of the teachers.
(D) She provides details about his
45 Why did Paul wear a red carnation?
behavior and the way it is interpreted
by others.
(A) To show respect for the faculty
(B) To show remorse
(C) To make himself appear wealthy
(D) To defy the faculty

Master the New SAT®


Practice Test 3 687

practice test
50 Which choice provides the best evidence 51 Based on the passage, which meaning of
for the answer to the previous question? the word “temperament” best fits the story?

(A) Lines 78–85 (“He stood … white (A) Complexion


teeth.”) (B) Adjustment
(B) Lines 74–78 (“His lips were … last (C) Mood
degree.”)
(D) Personality
(C) Lines 78–85 (“Older boys … his
hat.”)
52 As in line 12, “dandy” most nearly means
(D) Lines 89–92 (“This conscious … or
someone who
‘smartness.’”)
(A) is first-rate in his class.
(B) dresses with elegance and care.
(C) is carefree.
(D) is brilliant.

STOP
If you finish before time is called, you may check your work on this section only.
Do not turn to any other section.

www.petersons.com
Practice Test 3 723

answers Practice Test 3


ANSWER KEY AND EXPLANATIONS

Section 1: Reading Test

1. A 12. B 23. D 34. D 45. D


2. B 13. D 24. B 35. B 46. A
3. A 14. C 25. D 36. A 47. C
4. D 15. D 26. B 37. B 48. B
5. A 16. A 27. C 38. C 49. A
6. C 17. A 28. A 39. B 50. C
7. A 18. B 29. D 40. C 51. D
8. C 19. B 30. B 41. C 52. B
9. C 20. B 31. C 42. C
10. B 21. D 32. A 43. A
11. C 22. A 33. B 44. B

READING TEST RAW SCORE


(Number of correct answers)

1. The correct answer is (A). The overall main earthquakes is friction, as stated in choice
points of the article are all meant to dispel (B). Therefore, choices (A), (C), and (D)
the myths surrounding earthquakes: how are incorrect.
and why they occur. The article provides 3. The correct answer is (A). The article is
supporting details about the science of intended to separate fact from fiction for
earthquakes, which also counter the Holly- those who might be misinformed. Although
wood myths. Thus, choice (A) is the correct there are a lot of scientific facts in the pas-
answer. While the author does explain the sage, the tone is casual and almost scolding
causes and effects of earthquakes, it is to in its refutation of the myths surrounding
dispel the myths created by Hollywood, thus earthquakes. Choice (C) is incorrect because
choice (B) is incorrect. Choice (C) is incor- filmmakers would not be particularly inter-
rect, as the passage applies to earthquakes ested. Choices (B) and (D) are incorrect as
only and not to science in general. Choice both science teachers and scientists would
(D) is incorrect as this topic is not covered already know these facts.
in the passage.
4. The correct answer is (D). The article
2. The correct answer is (B). The text proves counters the myths by pointing out in lines
that “eventually enough stress builds up and 91–94 that the ocean isn’t a hole into which
the rocks slip suddenly, releasing energy in a land mass can fall, as the ocean is itself
waves that travel through the rock to cause just land at a lower elevation. Thus, choice
the shaking that we feel during an earth- (D) is the correct answer. Choice (A) is
quake” (lines 32–36). Although all of the incorrect because it accepts the myth as
factors listed in choices (A), (C), and (D) fact rather than countering it. Choice (B)
are contributing causes, the main cause of is incorrect because while the author talks

www.petersons.com
724 PART VII: Five Practice Tests

about the land mass under the ocean as be- tion the affected region of an earthquake.
ing at a lower sea level than California, it Similarly, choices (A) and (D) both contain
is only in explaining the geography of the lines that regard the attributes and causes
region. Choice (C) is incorrect because the of an earthquake, but fail to address why a
author does not explain that California and region is affected. Therefore, choices (A)
Alaska are similar but rather that southern and (D) are incorrect.
California is moving slowly toward Alaska. 9. The correct answer is (C). The context, that
5. The correct answer is (A). The author notes moderate earthquakes have caused signifi-
in lines 91–96 that California cannot break cant damage because the shaking was locally
off, as it is part of the land that makes up the amplified, suggests that amplified means
ocean floor, only at a higher elevation. Thus, made stronger, so choice (C) is the correct
choice (A) is the correct answer. Choice answer. Choice (A) is incorrect, as shaking
(B) is incorrect, as it is not an explanation would not be lifted. Choice (B) is incorrect,
as much as a refutation. Choices (C) and as “supplemented” or “added to” does not
(D) are incorrect, as they merely point out make sense in the context of an earthquake.
another feature of the geology of California; Choice (C) is incorrect because whether or
they do not clarify why California will not not an earthquake was predicted would have
be swept out into the ocean. no bearing on the shaking that is felt locally.
6. The correct answer is (C). In the context of 10. The correct answer is (B). The phrase
this passage, “stress” is pressure that builds reveals that disaster movies are made
up in rocks. Only in human interactions does more dramatic for effect at the expense of
“stress” describe the pressure that builds up scientific accuracy. Using the word “incon-
in people, causing worry and/or anxiety. venient” mocks the story; it implies that the
Stress does not imply the idea of weight in earthquake is used as an easy way to write a
the context of this passage, making choice character out of the story. Choices (A), (C),
(C) incorrect. It also does not mean “em- and (D) are incorrect because the use of the
phasis,” so choice (D) is incorrect. referenced phrase does not directly relate to
7. The correct answer is (A). The passage scientific information within the passages,
makes it clear that the closer a region is to as stated in these choices.
the earthquake source, the more that region 11. The correct answer is (C). The passage
will be affected. The most likely reason explains that friction is required for an
for devastation in Haiti (or any place) is earthquake (lines 108–109), making the
that these variables were in place: lack of idea of a gaping fault impossible, as a fault
buildings that could withstand the shaking line could only open up without friction.
from an earthquake and an earthquake that Choice (A) is incorrect because although
was probably very strong. The combination the concept is portrayed in movies, which
can be deadly. Choices (B) and (C) may be use the image, that fact does not refute the
partially true, but neither choice completely myth. Choices (B) and (D) are both incorrect
accounts for why Haiti experienced such because they are true statements that do not
extensive damage. Therefore, choices (B) refute the premise.
and (C) are both incorrect. Choice (D) is 12. The correct answer is (B). Patrick Henry
incorrect because the passage does not sup- lashes out against concentrated power (Pas-
port this statement. sage 1, lines 1–5). Madison, on the other
8. The correct answer is (C). Lines 64–69 hand, explains that power is essential, but
clearly explain that the effects of an earth- separating the power of the government into
quake are dependent on the proximity of the different departments (branches) will act as
earthquake source to the affected region as a check on that power, ensuring that it can-
well as the structure of the buildings that not get out of control (Passage 2, sentence
are shaken by the earthquake. Choice (B) 1). Choices (A), (C), and (D) are incorrect
is incorrect because these lines discuss the because the statements made in them do not
creation of an earthquake, but do not men- reflect the content of the passages.

Master the New SAT®


Practice Test 3 725

answers Practice Test 3


13. The correct answer is (D). Henry addresses would not have too much power. Choice (C)
the issue of concentrating power and says would be detrimental to use as a counterar-
that it rests “on a supposition that your gument, and is therefore incorrect. Choices
American governors shall be honest” (lines (B) and (D) are both incorrect because they
35–37), which, in his view, is a dangerous do not pertain to the counterargument.
and unnecessarily risky assumption to make. 17. The correct answer is (A). Henry’s state-
Choice (A) is incorrect because it is never ment shows a skepticism about human nature
implied in the passage that the structure of and the ability of humans to resist corrup-
the Constitution is unstable. Choices (B) tion. He says that never in human history
and (C) are both incorrect because neither has there been a benevolent government
correlate with the views of Henry and would that automatically grants its people rights;
therefore not be an objection of his. governments only grant the people rights
14. The correct answer is (C). The artist por- and liberties if their power is threatened.
trays Henry as a passionate speaker. This This statement shows Henry’s belief that
mirrors the dramatic phases Henry uses in human beings are inherently dishonest
his speech, such as calling the features of and self-concerned. Madison, on the other
the Constitution “horribly frightful” and the hand, believes that although humans are not
government “horridly defective.” Choice perfect beings, with the proper controls, it is
(A) is incorrect because the reader has no possible for humans to govern themselves
way of knowing how people react to Henry’s without resorting to tyranny. Choices (B),
speech. The artist does not portray Henry (C), and (D) are all incorrect because they
as calm; instead Henry appears impas- contain information that is not substantiated
sioned, making choice (B) incorrect. The in the passage.
artist shows Henry as impassioned, but not 18. The correct answer is (B). Henry is address-
necessarily an extremist. Some spectators ing the Virginia Convention and attempting
look excited; some look angry or sullen, but to persuade them to vote against ratification
there is no indication Henry is perceived (introduction). In this part of his speech,
as a madman. Henry does appeal to the Henry is responding to the idea of checks
listener’s patriotism, but his argument is and balances in the government, calling
not built slowly. The structure is logical, them “imaginary.” He refers rhetorically to
but the pacing is quick and the tone urgent. what the opposition (i.e., the framers) are
Therefore, choice (D) is also incorrect. saying about how such a government would
15. The correct answer is (D). Madison argues work with balanced branches and a central-
that a government whose powers are sepa- ized government. Henry also discusses the
rated by design will act as controls, each “ridiculous ideal checks and contrivances”
branch on the other, and avoid concentration and how the opposition’s idea of checks
of power. Choices (A) and (B) are incorrect and balances would not work. Choice (A) is
because they both relate an idea that would incorrect because Henry is giving a speech
be detrimental to use in order to address and would therefore not be referring to a
concerns about a strong central govern- narrator. Choices (C) and (D) are incorrect
ment. Choice (C) is incorrect because it is because, in the speech, Henry is addressing
not a point made by Madison anywhere in the opposition and not all of the general
Passage 2. public or the people of Virginia are in the
16. The correct answer is (A). Lines 73–80 opposition.
describe how Madison believes the govern- 19. The correct answer is (B). In line 7 (“the
ment should be divided into branches that genius of republicanism”), Henry praises
would check and balance each other. This a republican form of government (i.e., one
would serve to counteract the idea that the that has elected representatives and is not run
government is too centralized because the by a monarchy) Henry indicates that such a
branches would allow for the power to be centralized government, as outlined in the
divided and would make it so that one branch Constitution, cannot work with the goals of

www.petersons.com
726 PART VII: Five Practice Tests

forming a republican government. Choices because while it is true, it does not explain
(A), (C), and (D) are incorrect because Henry why the scientists were surprised. Choices
is adamant about his opposition towards (C) and (D) are incorrect because neither is
anything similar to a monarchial govern- suggested in the passage.
ment, autocracy and plutocracy included. 25. The correct answer is (D). Scientific jour-
20. The correct answer is (B). Line 7 (“the nals report news relevant to the field of the
genius of republicanism”) clearly states journal’s specialty. Taxonomy is a specific
that Henry believes that the ideal form of field that studies the classification of animals
government is republicanism where all the and plants, which is shown by the interest
people are represented. Choices (C) and (D) in determining the genus and species of
are similar because they both have to do with the pocket shark. Choices (A) and (B) are
the United States becoming a monarchy. incorrect because, although they could be
Therefore, these choices are both incorrect. true, they do not explain why the finding was
Choice (A) is incorrect because it does not reported in the journal of taxonomy. Choice
have to do with Henry’s preference in forms (C) is incorrect because the question notes
of government. that the finding was reported in a journal of
21. The correct answer is (D). Madison’s refer- taxonomy, not a journal that specializes in
ence to “department” refers to the branches marine animals.
of the government that are outlined in the 26. The correct answer is (B). Choice (B) de-
Constitution (executive, legislative, judi- scribes how scientists were able to classify
cial), which is apparent from the context. the specimen and genetically analyze the
Choice (A), (B), and (C) are incorrect be- specimen. This explains why the finding
cause they do not make sense in the context was published in the journal of taxonomy,
of the paragraph. which is the classification of species. Choice
22. The correct answer is (A). In the context (A) and choice (C) do not relate to the rea-
of the sentence, Madison is referring to how sons for why the finding was published in
the checks on one department (or branch) the journal of taxonomy and are therefore
would be in turn proportional to the checks incorrect. Although choice (D) relates to
on another department. One department taxonomy, it does not explain where the
would hold another in check to maintain a finding was published. Therefore, choice
balance of power so that power would not (D) is also incorrect.
get concentrated in one branch. Choices 27. The correct answer is (C). Paragraph 5
(B), (C), and (D) do not make sense in the explains that tissue samples were analyzed
context and are therefore incorrect. to get genetic information that could be
23. The correct answer is (D). The article compared to other specimens. Using this
mentions multiple scientists and institutions data, scientists could then determine how
who all worked on identifying the shark and to classify the pocket shark. Choices (A),
sharing their information so that it could (B), and (D) could all help to determine the
be correctly catalogued and understood. species of the shark, but they do not provide
The overall conclusion about methodology, the exact information that scientists need to
therefore, shows that science can be a col- determine the species of the shark.
laborative endeavor. Choice (A) is incorrect 28. The correct answer is (A). In Part A of the
because it is not relevant to the passage. diagram, there is a lighter spot above the fin,
Choice (B) is incorrect because it contains though it is not labeled there. But on part
information that is not fully supported by C, the pocket gland is labeled and is shown
the passage. Choice (C) is also incorrect, as in the same place and on the light spot in
it is not suggested in the passage. part A. Based on this, it can be inferred that
24. The correct answer is (B). Lines 32–35 this is the place where the pocket is located.
show that the scientists discovered the Choices (B), (C), and (D) are incorrect be-
shark by accident when it was found in cause neither the image nor the text provides
a controlled lab. Choice (A) is incorrect

Master the New SAT®


Practice Test 3 727

answers Practice Test 3


evidence to suggest that the pockets are sarcasm and satire, to show why it cannot be
located in any of these locations. considered any kind of literary art, making
29. The correct answer is (D). Lines 66–67 choice (A) incorrect. Choice (C) is incorrect
explain that scientists “hoped that further because only a portion of the passage deals
study of the specimen will lead to many new with the Native Americans. Although Twain
discoveries.” The specifics aren’t discussed does criticize Cooper based on his work, he
because scientists cannot anticipate what never shows the importance of literary criti-
they will discover, but it can be assumed cism in the passage. For this reason, choice
that it will be related to sharks. Choices (A), (D) is also incorrect.
(B), and (C) are incorrect because they all 34. The correct answer is (D). For the entirety
relate directly to the pocket shark; scientists of the passage, Twain mocks Cooper’s story,
hope to discover new information about but he does so in a humorous way. For ex-
sharks in general. ample, in the second to the last paragraph
30. The correct answer is (B). Lines 65–67 (lines 79–81), he describes the action of the
explain how the discovery of the pocket novel, saying: “Let me explain what the five
shark could lead to further discoveries in did—you would not be able to reason it out for
the future, a prospect that would excite yourself.” This is sarcasm making mockery
scientists. Choice (A), (C), and (D) are all of the action of the characters in the book.
incorrect because they involve informa- It is light-hearted and expresses incredulity,
tion about the pocket shark that, although thus setting a humorous tone. Choice (A) is
probably exciting to some, is very specific incorrect because arrogance implies an exag-
and does not provide a broad explanation gerated sense of self-importance that is not
of why the find may be exciting in general, expressed in the passage. The voice of the
as choice (B) does. passage is somewhat informal and, as a result,
not scholarly. This makes choice (B) incor-
31. The correct answer is (C). “Robust” in
rect. The tone of the passage is humorous;
this context describes a collection that has
however, it is not cheery and does not suggest
variety. A specimen is a sample used for
optimism, making choice (C) incorrect.
testing, so it makes sense for the lab to have
a large, substantial collection of marine 35. The correct answer is (B). Lines 11–12
animal samples for testing. Choices (A), illustrate the hyperbole that Twain uses
(B), and (D) are incorrect because it cannot to humorously describe how Cooper took
be inferred, based on the passage, that the liberty with his description of the techniques
“robust specimen collection” is how these Native Americans used to track footprints
choices describe it. to find and/or attack enemy tribes. In this
line, Twain’s description of wearing out
32. The correct answer is (A). The author re-
the moccasins in the process of following
fers to pocket sharks, which, when hungry,
a trail pokes fun at the erroneous nature of
take an “oval plug of flesh” from prey, so
Cooper’s characterization. Choices (A),
“plug” is this context means lump or piece.
(C), and (D) are all critiques, but they do
Choice (B) is incorrect because there is no
not express humor.
indication that the flesh that the shark bites
off actually protrudes from the prey’s body. 36. The correct answer is (A). Twain refers to
Choice (C) is incorrect because “obstruction Cooper’s use of various literary devices as
of flesh” does not make sense. Choice (D) “stage properties,” as a way to mock Coo-
is also incorrect because it, too, makes no per. The “barrels and barrels of moccasins”
sense in the context of the sentence. is a satire on the moccasins referred to by
Cooper’s character who is tracking a Native
33. The correct answer is (B). In spite of the
American through footprints made by the
humor, Twain makes it clear that he does not
moccasins. The “stage properties” Twain
approve of either the story, which he finds
refers to do not have anything to do with
unrealistic and boring, or the style, which
drama or theater, making choices (B), (C),
he finds repetitive. While he does point out
and (D) incorrect.
the flaws in the work, he does so by using

www.petersons.com
728 PART VII: Five Practice Tests

37. The correct answer is (B). The pronun- miscalculate the timing). Choices (A), (B),
ciation guide is another way of mocking and (D) describe the Native Americans as
Cooper’s book. Here, Twain describes the more intelligent and competent than Cooper
plot and throws in, as an aside, the pronun- is credited with describing them.
ciation key to indicate how far Cooper veers 41. The correct answer is (C). Twain notes
from reality, so much so that the Native that Cooper “conceals six Indians in its (the
Americans he describes might as well be sapling’s) foliage” (lines 51–52). A sapling
called Chicago. Choices (A) and (C) are is a young, immature tree, so it is illogical
incorrect because Twain is writing satirically to assume that six people could be hidden in
and does not actually intend for the reader the foliage of a sapling. Thus, choice (C) is
to pronounce the Native American’s name the correct answer. Choice (A) is incorrect,
correctly. As the work is satire, Twain only as it is untrue. Choice (B) is incorrect, as
intends to criticize Cooper, and illustrating the audience would not necessarily know
that Cooper’s work was about actual Na- that this is why he put it in quotation marks.
tive Americans would achieve this goal. Choice (D) is incorrect, as it is untrue.
Therefore, choice (D) is incorrect.
42. The correct answer is (C). In the context
38. The correct answer is (C). The biggest of the sentence, “destitute” means that the
and most frequent criticism Twain makes work is void of the variables that make a
in the passage is to show how unrealistic story artful. Although the word can mean
the plot and characters are. For example, in impoverished, that meaning does not fit
describing the ark (paragraph 5), he explains this context, making choice (A) incorrect.
how the timing is off for the entire scene, Choices (B) and (D) are both incorrect be-
making its occurrence totally unreasonable. cause they do not make sense in this context.
And here he jokes and ridicules the outcome:
43. The correct answer is (A). The overall
“He missed the house and landed in the stern
descriptions are of the character Paul. The
of the scow. It was not much of a fall, yet it
passage describes how he looks, his actions
knocked him silly.” (Lines 70–73) showing
and reactions, and his emotions. Through
how absurd the story line is. Choices (A),
the use of a narrator telling the story, it also
(B), and (D) are not substantiated by the
describes how others react to him, giving
passage and are therefore incorrect.
the reader an outside perspective of the
39. The correct answer is (B). Lines 48–49 refer character. Thus, choice (A) is the correct
to Cooper’s changes in the width and flow answer. Choice (B) is incorrect because the
of a river, which Twain deems unrealistic passage description notes that it is from a
and therefore defective. Twain makes the short story and therefore fiction. Choice (C)
case that Cooper’s description of the natural is incorrect because there is no plot in the
world is mistaken and at odds with reality passage. Choice (D) is incorrect because it
and therefore defective. Choice (A) is merely is the character of Paul—not any one set-
a hyperbolic description of Twain’s list of ting—that is emphasized.
defects, and choice (C) illustrates Twain’s
44. The correct answer is (B). The author notes
style of satire. Therefore, both choices (A)
in lines 1–4 that it “was Paul’s afternoon to
and (C) are incorrect. Choice (D) does not
appear before the faculty of the Pittsburgh
show how the jump was unrealistic or how in
High School to account for his various mis-
itself it mischaracterizes the way something
demeanors.” Thus, choice (B) is the correct
would happen, making choice (D) incorrect.
answer. Choice (A) is incorrect because
40. The correct answer is (C). Twain criti- while the author notes that Paul does tell the
cizes Cooper’s characterization of the Na- principal that he wants to return to school,
tive Americans as dim-witted, inept, and this was not the reason why he was asked
deceitful. The Native Americans bumble to appear before the faculty. Choice (C) is
through every attempt they make to attack incorrect because tardiness is not mentioned
the settlers (for example, they can’t jump as one of Paul’s infractions. Choice (D) is
onto a very slow-moving boat, because they incorrect because while he father calls the

Master the New SAT®


Practice Test 3 729

answers Practice Test 3


principal after Paul’s suspension confessing allows for an objective look at an adolescent
that he is perplexed by his son’s behavior, by observing his actions, reactions, and ap-
there is no mention that Paul’s parents are pearance. Choice (B) is the correct answer.
in the principal’s office at this time. Choices (A) and (D) are incorrect because
45. The correct answer is (D). The carnation is both Paul and the principal are referred to
obvious on his coat lapel, and it is a contrast in the third person. Choice (C) is incorrect
to the way he is dressed in shabby clothes, because a psychologist is not noted in the
as if he didn’t care. And yet the carnation introduction or the text.
adds a note of frivolity and mockery to the 49. The correct answer is (A). Paul’s appear-
seriousness of the event. It is a way of subtly ance is described in great detail and his outer
communicating that he will do as he pleases, appearance is often betrayed by behavior that
in defiance of the wishes of the faculty, thus gives away his true feelings. For example, he
choice (D) is the correct answer. Choice smiles but his fingers tremble and play with
(A) is incorrect, as Paul does not wish to the buttons on his coat (lines 80–84). Choices
show respect to the faculty. Choice (B) is (B), (C), and (D) are incorrect because they
incorrect because Paul does not intend to do not describe Paul’s actions and feelings;
show remorse. In fact, he intends to show rather, they describe contrasts with others
defiance. Choice (C) is incorrect because and their appearance and reactions.
if Paul wanted to appear wealthy, he would 50. The correct answer is (C). Cather notes
not wear shabby clothes. in her description of Paul playing with
46. The correct answer is (A). The author notes the buttons on his coat that it was the only
in lines 38–40 that his teachers stated their outward sign of his discomfort, thus in lines
charges against him with “such a rancor 82–85, she connects his outer actions to his
and aggrievedness as evinced that this was inner feelings, allowing the reader inside
not a usual case.” Thus, choice (A) is the Paul’s emotional state. Choice (C) then is
correct answer. Choice (B) is incorrect be- the correct answer. Choices (A) and (B) are
cause while the teachers may not understand incorrect, as the lines do not interpret the
Paul’s behavior, they are too offended by it meaning of Paul’s facial expression. Choice
to be sad or confused. Choice (C) is incor- (D) is incorrect, as the interpretation noted
rect because “scorn and disdain” imply that here is made by others and not necessarily
the teachers have made a value judgment, reflective of Paul’s feelings.
determining that Paul is despicable and un- 51. The correct answer is (D). The word as used
worthy. Instead, their actions suggest anger. in the title suggests that the story is focused
Choice (D) is incorrect because nothing in on the personality makeup of the protago-
the text suggests such feelings. nist. Thus, choice (D) is the correct answer.
47. The correct answer is (C). In lines 38–40, Choice (A) is incorrect because the narrator
the author notes that Paul’s teachers stated does not describe Paul’s complexion. Choice
their charges with “rancor and aggrieved- (B) is incorrect, as the word does not make
ness,” words that suggest anger and frustra- sense in the context. Choice (C) is incorrect,
tion. Thus, choice (C) is the correct answer. because the story is a description of a boy’s
Choice (A) is incorrect because these lines behavior over a period of time, and mood
refer to Paul’s father’s feelings. Choice (B) suggests one particular moment or feeling.
is incorrect because while it suggests that the 52. The correct answer is (B). A dandy is a per-
teachers did not feel that Paul was contrite, son who is meticulous in the way he dresses
it does not support the idea that they were and takes extreme care in his appearance.
angry and frustrated. Choice (D) is incorrect The paragraph in which the word is used is in
because these lines explain how the teachers the context of Paul’s clothing, and “dandy”
interpreted Paul’s feelings toward them. suits the characterization of Paul, so choice
48. The correct answer is (B). An unnamed (B) is the correct answer. Choices (A), (C),
narrator describes Paul, using great detail to and (D) are incorrect because the context is
show how others perceived him. The narrator a description of Paul’s dress.

www.petersons.com
Practice Test 4 765

practice test
SECTION 1: READING TEST
65 Minutes • 52 Questions

TURN TO SECTION 1 OF YOUR ANSWER SHEET TO ANSWER THE QUESTIONS IN THIS


SECTION.

Directions: Each passage (or pair of passages) below is followed by a number of multiple-choice
questions. After reading each passage, select the best answer to each question based on what is
stated or implied in the passage or passages and in any supplementary material, such as a table,
graph, chart, or photograph.

QUESTIONS 1–10 ARE BASED ON 25 regimented our whole people temporally


THE FOLLOWING PASSAGE AND into a socialistic state. However justified
in war time, if continued in peace-time
SUPPLEMENTARY MATERIAL.
it would destroy not only our American
From his humble beginnings, Herbert Hoover system but with it our progress and
made his fortune in the mining industry. He 30 freedom as well.
earned his reputation as a humanitarian and When the war closed, the most
skilled administrator during and after World vital of issues both in our own country
War I and later served as Secretary of Com- and around the world was whether
merce under both Presidents Harding and government should continue their
Coolidge. In 1928, he was nominated for pres- 35 wartime ownership and operation of
ident by the Republican Party. The following many [instruments] of production and
is an excerpt from a speech he gave at the distribution. We were challenged with
end of his campaign against the Democratic a … choice between the American system
nominee, New York Governor Alfred E. Smith. of rugged individualism and a European
During one hundred and fifty years 40 philosophy of diametrically opposed
we have builded up a form of self doctrines, doctrines of paternalism and
government and a social system which is state socialism. The acceptance of these
Line peculiarly our own. It differs essentially ideas would have meant the destruction of
5 from all others in the world. It is the self-government through centralization …
American system. … It is founded upon 45 [and] the undermining of the individual
the conception that only through ordered initiative and enterprise through which
liberty, freedom and equal opportunity our people have grown to unparalleled
to the individual will his initiative and greatness. …
10 enterprise spur on the march of progress. I would like to state to you the effect
And in our insistence upon equality of 50 that … [an interference] of government
opportunity has our system advanced \ in business would have upon our system
beyond all the world. of self-government and our economic
During [World War I] we necessarily system. That effect would reach to the
15 turned to the government to solve daily life of every man and woman. It
every difficult economic problem. The 55 would impair the very basis of liberty and
government having absorbed every freedom. …
energy of our people for war, there was Let us first see the effect on self-
no other solution. For the preservation of government. When the Federal
20 the state the Federal Government became Government undertakes to go into
a centralized despotism which undertook 60 commercial business it must at once set
unprecedented responsibilities, assumed up the organization and administration
autocratic powers, and took over the of that business, and it immediately finds
business of citizens. To a large degree, we itself in a labyrinth. … Commercial
GO TO THE
NEXT PAGE
www.petersons.com
766 PART VII: Five Practice Tests

business requires a concentration of Nor do I wish to be misinterpreted as


65 responsibility. Our government to succeed believing that the United States is a free-
in business would need to become in 95 for-all and devil-take-the-hindmost. The
effect a despotism. There at once begins very essence of equality of opportunity
the destruction of self-government …. and of American individualism is that
Liberalism is a force truly of the there shall be no domination by any group
70 spirit, a force proceeding from the deep or [monopoly] in this republic. … It is no
realization that economic freedom 100 system of laissez faire. …
cannot be sacrificed if political freedom I have witnessed not only at home but
is to be preserved. [An expansion of the abroad the many failures of government
government’s role in the business world] in business. I have seen its tyrannies,
75 would cramp and cripple the mental and its injustices, its destructions of self-
spiritual energies of our people. It would 105 government, its undermining of the very
extinguish equality and opportunity. It instincts which carry our people forward
would dry up the spirit of liberty and to progress. I have witnessed the lack of
progress. … For a hundred and fifty years advance, the lowered standards of living,
80 liberalism has found its true spirit in the the depressed spirits of people working
American system, not in the European 110 under such a system. …
systems. And what has been the result of the
I do not wish to be misunderstood. … American system? Our country has
I am defining general policy. … I become the land of opportunity to those
85 have already stated that where the born without inheritance, not merely
government is engaged in public 115 because of the wealth of its resources
works for purposes of flood control, of and industry but because of this freedom
navigation, of irrigation, of scientific of initiative and enterprise. Russia has
research or national defense … it will natural resources equal to ours. … But
90 at times necessarily produce power or she has not had the blessings of one
commodities as a by-product. 120 hundred and fifty years of our form of
government and our social system.

This map shows the electoral votes for each state in the presidential election
of 1928, which Herbert Hoover won in a landslide.

Master the New SAT®


Practice Test 4 767

practice test
1 Based on the excerpt, which state- 5 How did Hoover view European govern-
ment is true of Hoover’s ideas about ments?
government?
(A) Tyrannical/despotic
(A) Socialism is justified during wartime, (B) Democratic/republican
but not at other times.
(C) Socialistic/paternalistic
(B) Government cannot solve economic
(D) Autocratic/monarchical
problems.
(C) A strong government stifles
6 Based on the map, what is a landslide?
individual liberty.
(D) A democratic government is always (A) A popular vote
superior to other governments.
(B) A close contest
(C) A hard-won election
2 Why did Hoover believe it was neces-
(D) A decisive victory
sary to strengthen the federal govern-
ment during wartime?
7 Which of the following lines in the text
(A) The United States was fighting explain how the U.S. changed course
despotism and therefore had to during WWI?
become despotic.
(B) Only a socialistic government can (A) Lines 1–4 (“During one hundred …
function during wartime. our own.”)
(C) Most resources went toward the war (B) Lines 14–16 (“During [World War
effort, draining them for other uses in I] … economic problem.”)
the economy. (C) Lines 53–54 (“That effect … and
(D) Businesses acting by themselves woman.”)
could interfere with the war effort. (D) Lines 89–91 (“it will at times … a
by-product.”)
3 How does Hoover’s speech change in
lines 83–110 (“I do not wish … under 8 How does Hoover indicate that he be-
such a system.”)? lieves the American form of government
is superior to others?
(A) He changes his attitude.
(B) He shows he wants to be understood. (A) Lines 6–10 (“It is founded … of
progress.”)
(C) He shows that his ideas are moderate.
(B) Lines 69–73 (“Liberalism is … be
(D) He clarifies his position by providing
preserved.”)
details about his philosophy.
(C) Lines 99–100 (“It is no … laissez
faire.”)
4 What is the most likely reason that
(D) Lines 117–118 (“Our country … of
Hoover includes lines 83–110 in his
opportunity”)
speech?

(A) He wanted to appeal to more voters. 9 As used in line 63, “labyrinth” most
(B) He wanted to preempt criticism. nearly means
(C) He wanted to soften his position.
(A) maze.
(D) He wanted to provide context for his
(B) cave.
views.
(C) bind.
(D) landmine.
GO TO THE
NEXT PAGE
www.petersons.com
768 PART VII: Five Practice Tests

10 As used in line 25, “regimented” most me to work out some great good to
nearly means 20 others …
I allude to this here because I have
(A) organized. often felt that much that is in that book
(B) militarized. (“Uncle Tom”) had its root in the awful
scenes and bitter sorrows of that summer.
(C) bullied.
25 It has left now, I trust, no trace on my
(D) established. mind except a deep compassion for the
sorrowful, especially for mothers who are
QUESTIONS 11–21 ARE BASED ON THE separated from their children ….
FOLLOWING TWO PASSAGES AND I am now writing a work which will
SUPPLEMENTARY MATERIAL. 30 contain, perhaps, an equal amount of
matter with Uncle Tom’s Cabin. It will
Passage 1 is an excerpt from a letter written contain all the facts and documents upon
by Harriet Beecher Stowe, the abolitionist and which that story was founded, and an
author of the best-selling novel Uncle Tom’s immense body of facts, reports of trial,
Cabin (1852). The book describes the horrors 35 legal documents, and testimony of people
of slavery and is said to have helped promote now living South, which will more than
the abolitionists’ cause. The recipient of the confirm every statement in Uncle Tom’s
letter, Mrs. Follen, a fellow abolitionist, was Cabin. I must confess that till I began the
also a poet, editor, and novelist. Passage 2 examination of facts in order to write this
is an excerpt from the memoir of Frederick 40 book, much as I thought I knew before,
Douglass, an active abolitionist who had I had not begun to measure the depth of
escaped from slavery in 1838. After gaining the abyss. The law records of courts and
his freedom, Douglass published Narrative of judicial proceedings are so incredible
the Life of Frederick Douglass, An American as to fill me with amazement whenever
Slave, Written by Himself (1845). The passage 45 I think of them. It seems to me that the
describes how he planned his escape. book cannot but be felt, and, coming upon
Passage 1 the sensibility awaked by the other, do
something. I suffer exquisitely in writing
Harriet Beecher Stowe, from a letter to these things. It may be truly said that I
Mrs. Follen (1853) 50 suffer with my heart’s blood. Many times
I had two little curly-headed twin in writing Uncle Tom’s Cabin I thought
daughters to begin with, and my stock my heart would fail utterly, but I prayed
in this line was gradually increased, till I earnestly that God would help me till I
Line have been the mother of seven children, got through, and still I am pressed beyond
5 the most beautiful and the most loved 55 measure and above strength ….
of whom lies buried near my Cincinnati
residence. It was at his dying bed and at Passage 2—Recollection of Frederick
his grave that I learned what a poor slave Douglass
mother may feel when her child is torn … It is impossible for me to
10 away from her. In those depths of sorrow describe my feelings as the time of my
which seemed to me immeasurable, it was contemplated start grew near. I had
my only prayer to God that such anguish a number of warm-hearted friends in
might not be suffered in vain. There 60 Baltimore,—friends that I loved almost as
were circumstances about his death of I did my life,—and the thought of being
15 such peculiar bitterness, of what seemed separated from them forever was painful
almost cruel suffering, that I felt that I beyond expression. It is my opinion that
could never be consoled for it unless this thousands would escape from slavery,
crushing of my own heart might enable 65 who now remain, but for the strong
cords of affection that bind them to their

Master the New SAT®


Practice Test 4 769

practice test
friends. The thought of leaving my friends punishment, and being placed beyond the
was decidedly the most painful thought means of escape. It required no very vivid
with which I had to contend. The love of imagination to depict the most frightful
70 them was my tender point, and shook my scenes through which I would have to
decision more than all things else. Besides 85 pass, in case I failed. The wretchedness of
the pain of separation, the dread and slavery, and the blessedness of freedom,
apprehension of a failure exceeded what I were perpetually before me. It was life
had experienced at my first attempt. The and death with me. But I remained firm
75 appalling defeat I then sustained returned and according to my resolution, on the
to torment me. I felt assured that, if I 90 third day of September, 1838, I left my
failed in this attempt, my case would be chains, and succeeded in reaching New
a hopeless one—it would seal my fate as York without the slightest interruption of
a slave forever. I could not hope to get any kind.
80 off with anything less than the severest

This map shows population and density of slaves in 1860.


(Credit: Historical Map & Chart Collection, Office of Coast Survey, NOAA)

11 What was the common thread of the 12 Why would Stowe be sympathetic to
experiences that stimulated the writing Douglass’ experience?
of Stowe and Douglass?
(A) She was appalled at the idea of
(A) Loss of close friends slavery.
(B) Death in the family (B) She had lost a young child.
(C) Witness to cruelty of slavery (C) She knew freed slaves who told her
(D) Permanent loss of loved ones their stories.
(D) She had family members who had
been enslaved.

GO TO THE
NEXT PAGE
www.petersons.com
770 PART VII: Five Practice Tests

13 What is the most likely reason that 17 What do the passages suggest about the
Stowe wrote Uncle Tom’s Cabin? abolition movement in the mid-nine-
teenth century?
(A) She wanted to earn her own money.
(B) She wanted to document her own (A) It was made up of people who
experience with slavery. opposed slavery based on reading
about it.
(C) She wanted to show others why
slavery should be abolished. (B) There were many people writing
to expose the evils of slavery in an
(D) She wanted to express her concerns
effort to abolish it.
about freed slaves in the North.
(C) People in the North were angry that
slavery was legal.
14 Why did Stowe want to write another
(D) The conflict over slavery was heating
book after the success of Uncle Tom’s
up.
Cabin?

(A) She wanted to show that her 18 As used in line 48, “exquisitely” most
characters were real even though the nearly means
work is a novel.
(B) She wanted to confirm that Uncle (A) beautifully.
Tom’s Cabin was based on the truth. (B) exhaustively.
(C) She needed to prove her point to sell (C) delightfully.
more books. (D) intensely.
(D) She discovered that slavery was
worse than she had thought. 19 As used in line 69, “contend” most
nearly means
15 What did Douglass think prevented more
slaves from running away? (A) struggle.
(B) assert.
(A) They wouldn’t be able to get jobs in
(C) debate.
the North and would be homeless.
(D) challenge.
(B) They would get caught and severely
punished.
20 What evidence suggests that Stowe’s
(C) They were too afraid of the difficult
and dangerous journey. abolitionist activism was a way to me-
morialize her lost child?
(D) They were afraid that they would
never see their loved ones again. (A) Lines 7–9 (“It was at … mother may
feel”)
16 Why might Douglass have given his nar- (B) Lines 16–20 (“that I felt … good to
rative its title? others”)
(C) Lines 22–24 (“that book … that
(A) He was almost illiterate and liked the
summer”)
way it sounded.
(D) Lines 49–50 (“It may be … heart’s
(B) He wanted people to know that he
blood”)
actually was the author.
(C) He wanted to make sure no one else
could take his identity.
(D) He wanted to promote the abolitionist
cause.

Master the New SAT®


Practice Test 4 771

practice test
21 Why does Stowe believe she needs to do occupied by those orbiters. For safety,
research for her next book? NASA also monitors positions of ESA’s
and India’s orbiters, which both fly
(A) Lines 25–27 (“It has left … the elongated orbits.
sorrowful”) …
(B) Lines 40–42 (“much as I … the 35 Traffic management at Mars is much
abyss.”) less complex than in Earth orbit, where
more than 1,000 active orbiters plus
(C) Lines 42–43 (“The law records … so
additional pieces of inactive hardware
incredible”)
add to hazards. As Mars exploration
(D) Lines 50–52 (“Many times … fail 40 intensifies, though, and will continue to
utterly”) do so with future missions, precautions
are increasing. The new process was
QUESTIONS 22–31 ARE BASED ON established to manage this growth as new
THE FOLLOWING PASSAGE AND members are added to the Mars orbital
SUPPLEMENTARY MATERIAL. 45 community in years to come.
All five active Mars orbiters use the
The following passage is a news item pub- communication and tracking services of
lished on May 4, 2015, from the Jet Pro- NASA’s Deep Space Network, which
pulsion Laboratory at the California Institute is managed at JPL [Jet Propulsion
of Technology. The lab is part of NASA and is 50 Laboratory]. This brings trajectory
dedicated to the robotic exploration of space. information together, and engineers
Traffic Around Mars Gets Busy can run computer projections of future
NASA has beefed up a process of trajectories out to a few weeks ahead for
traffic monitoring, communication, and comparisons.
maneuver planning to ensure that Mars 55 “It’s a monitoring function to
Line orbiters do not approach each other too anticipate when traffic will get heavy,”
5 closely. said Joseph Guinn, manager of JPL’s
Last year’s addition of two new Mission Design and Navigation Section.
spacecraft orbiting Mars brought the “When two spacecraft are predicted to
census of active Mars orbiters to five, the 60 come too close to one another, we give
most ever. NASA’s Mars Atmosphere and people a heads-up in advance so the
10 Volatile Evolution (MAVEN) and India’s project teams can start coordinating about
Mars Orbiter Mission joined the 2003 whether any maneuvers are needed.”
Mars Express from ESA (the European The amount of uncertainty in the
Space Agency) and two from NASA: 65 predicted location of a Mars orbiter
the 2001 Mars Odyssey and the 2006 a few days ahead is more than a mile
15 Mars Reconnaissance Orbiter (MRO). (more than two kilometers). Calculating
The newly enhanced collision-avoidance projections for weeks ahead multiplies
process also tracks the approximate the uncertainty to dozens of miles, or
location of NASA’s Mars Global 70 kilometers. In most cases when a collision
Surveyor, a 1997 orbiter that is no longer cannot be ruled out from projections
20 working. two weeks ahead, improved precision
It’s not just the total number that in the forecasting as the date gets closer
matters, but also the types of orbits will rule out a collision with no need for
missions use for achieving their science 75 avoidance action. Mission teams for the
goals. MAVEN, which reached Mars relevant orbiters are notified in advance
25 on Sept. 21, 2014, studies the upper when projections indicate a collision is
atmosphere. It flies an elongated orbit, possible, even if the possibility will likely
sometimes farther from Mars than disappear in subsequent projections.
NASA’s other orbiters and sometimes 80 This situation occurred on New Year’s
closer to Mars, so it crosses altitudes weekend, 2015.
30 GO TO THE
NEXT PAGE
www.petersons.com
772 PART VII: Five Practice Tests

On Jan. 3, automated monitoring spacecraft unless projections a day or two


determined that two weeks later, MAVEN 105 ahead showed probability of a hazardous
and MRO could come within about two conjunction. The amount of uncertainty
85 miles (three kilometers) of each other, about each spacecraft’s exact location
with large uncertainties remaining in the varies, so the proximity considered unsafe
exact passing distance. Although that was also varies. For some situations, a day-
a Saturday, automatic messages went out 110 ahead projection of two craft coming
to the teams operating the orbiters. within about 100 yards (100 meters) of
90 “In this case, before the timeline got each other could trigger a maneuver.
short enough to need to plan an avoidance The new formal collision-avoidance
maneuver, the uncertainties shrank, process for Mars is part of NASA’s
and that ruled out the chance of the two 115 Multi-Mission Automated Deep-Space
spacecraft coming too near each other,” Conjunction Assessment Process. A side
95 Guinn said. This is expected to be the benefit of it is that information about
usual pattern, with the advance warning when two orbiters will be near each
kicking off higher-level monitoring and other—though safely apart—could be
initial discussions about options. 120 used for planning coordinated science
If preparations for an avoidance observations. The pair could look at
100 maneuver were called for, spacecraft some part of Mars or its atmosphere
commands would be written, tested, from essentially the same point of view
and approved for readiness, but such simultaneously with complementary
commands would not be sent to a 125 instruments.

This graphic depicts the relative shapes and distances from Mars for five active orbiter missions plus the planet’s two
natural satellites. It illustrates the potential for intersections of the spacecraft orbits.

Master the New SAT®


Practice Test 4 773

practice test
22 Which of the following is the best sum- 26 Which are the natural satellites of Mars?
mary of passage?
(A) MRO and Phobos
(A) Gathering information from space (B) Phobos an MAVEN
requires careful and complex
(C) Deimos and Odyssey
monitoring of spacecraft satellites.
(D) Phobos and Deimos
(B) The multiple satellites orbiting
around Mars could collide even
though they are not traveling in the 27 Based on the graphic, which satellite is
same orbits. least likely to have collision problem?
(C) Planning for space satellites around
(A) Phobos, because it has a more
other planets takes skilled teamwork.
elongated orbit
(D) Monitoring traffic around the Earth
(B) MOM, because its orbit extends
is more complex than monitoring the
farthest from Mars
traffic around Mars.
(C) Deimos, because it only intersects
with one other satellite
23 Which countries are involved in moni-
(D) MAVEN, because its path is
toring of the Mars satellites?
controlled by NASA engineers
(A) The United States, Russia, and India
(B) Europe and the United States 28 Which lines in the passage explain why
(C) India, Europe, and the United States scientists must constantly monitor the
satellites’ orbits?
(D) The United States, China, and Russia
(A) Lines 24–26 (“MAVEN, which …
24 According to information in the passage, upper atmosphere”)
what happened on New Year’s weekend (B) Lines 26–29 (“It flies … to Mars”)
2015? (C) Lines 35–36 (“Traffic
management … Earth orbit”)
(A) Data indicated that there could be a
collision. (D) Lines 46–48 (“All five … Space
Network”)
(B) Data gave scientists new information
about the orbit of the satellites.
29 What evidence in the passage supports
(C) The collision alert turned out to be a
false alarm. the idea that the distances between satel-
lites are related to the degree of danger
(D) Two satellites almost collided.
they pose?

25 Based on information given in the pas- (A) Lines 75–78 (“Mission teams … is
sage, which of the following career fields possible”)
would the workers at the Jet Propulsion (B) Lines 83–85 (“MAVEN and … each
Lab most likely be qualified for? other”)
(C) Lines 106–109 (“The amount … also
(A) Disaster preparedness planner
varies”)
(B) Meteorologist
(D) Lines 113–116 (“The new formal …
(C) Airplane pilot Assessment Process.”)
(D) Road construction worker

GO TO THE
NEXT PAGE
www.petersons.com
774 PART VII: Five Practice Tests

30 As used in line 106, “conjunction” most Now, native potato cultivation starts
nearly means 20 at around 4,000 meters. In just 30 years,
challenges associated with a warming
(A) connection between the satellites. climate have pushed potato cultivation up
(B) divergence between orbits. by 200 meters. The speed of this change in
planting zones due to a warming climate is
(C) linking of Mars and a satellite.
25 unprecedented as it is pushing the farmers
(D) alignment of two satellites. to the top of the mountain, beyond which
there is no more soil or land.
31 As used in line 52, “projections” most In addition to moving to higher
nearly means elevation for potato cultivation,
30 Quechua farmers in the Potato Park are
(A) use of trajectory information. also responding to this challenge by
(B) reference to future trajectories. stewarding over 1,440 cultivars of native
potato. These include their own varieties
(C) comparisons to other trajectories.
plus cultivars that different entities have
(D) engineers can make comparisons. 35 provided to the Park, 410 of which have
come from CIP.
QUESTIONS 32–42 ARE BASED ON THE The five communities that make up
FOLLOWING PASSAGE. the Potato Park are also working with
CIP scientists in the characterization of
Potatoes were first cultivated in Peru by the 40 potato diversity, monitoring changes
Incas between 8000 and 5000 BCE. Today, in potato varieties used over time and
potatoes make up the fourth largest food testing of varieties in different parts of the
crop in the world. In a ground-breaking landscape, a combined territory of over
agreement between the International Potato 9,000 hectares.
Center (known by its Spanish acronym CIP), 45 Planting a diversity of potatoes
the nonprofit organization ANDES, and the provides a vital safeguard against crop
Association of the Potato Park communities, failure–if disaster strikes, the farmers will
scientists are helping Peruvian farmers test always have food. This strategy to reduce
and monitor many varieties of potatoes. One risk comes from their ancestors.
of their goals is to ensure continued diversity 50 The agreement with CIP has brought
in potatoes—an action that could prevent back varieties which had been collected
disasters such as the Irish Potato Famine. from the communities in the 1960s
As world leaders gather in Lima to but had since been lost. The resulting
negotiate a new global climate deal at the landscape-based gene bank is actively
UN Climate conference, this innovative, 55 managed by the five Potato Park
Line inclusive work shows the importance
communities.
5 of new kinds of partnerships between
It provides a critical source of climate-
scientists and farmers for adaptation to resilient crops for adaptation, both locally
climate change. and globally. Although gene banks
In the Peruvian highlands near Cusco, 60 conserve many food crops, they cannot
climate change has already impacted safeguard them all, and their collections
10 farmers in a fundamental way. Rising
are no longer evolving in response to
temperatures are correlated with increased climatic changes or accessible to farmers.
pests and diseases, making it difficult to Alejandro Argumedo, Director of
grow potatoes, their staple food. 65 Programs of the Peruvian NGO ANDES
The effects of these temperature explained: “The landmark agreement
15 changes are very pronounced in the Potato
between CIP and the Potato Park for
Park, a valley outside of Cusco, where repatriation and monitoring of native
just 30 years ago, cultivation of native potatoes represents a fundamental shift
potato was routinely done at 3,800 meters. 70 in approach. Rather than only collecting

Master the New SAT®


Practice Test 4 775

practice test
crops from farmers, scientists have also 33 How are potato farmers trying to combat
given farmers crops from their gene climate change?
bank in return. The disease-free seeds
and scientific knowledge gained have (A) They will be attending the UN
75 boosted food security, and the new summit on climate change.
varieties have enhanced income, enabling (B) They are forming new partnerships
the communities to develop novel food with scientists.
products.”
(C) They are planting more diverse seeds
Head of the CIP Genetic Resources-
and plants.
80 Genebank David Ellis said: “Through
the agreement, first signed in 2004, (D) They are experimenting with a
CIP is increasing its understanding of different kind of staple food.
how climate change is affecting potato
diversity and agro-ecosystems, and 34 What kind of climate is best suited for
85 through collaborative and mutually potatoes?
beneficial research with the farmers, it
has continued to enhance knowledge, (A) Tropical
adaptation to climate change and capacity (B) Cool
development for sustaining potato
(C) Warm
90 production and traditional knowledge.
In Asia and Africa, hardy local (D) Cold
landraces and livestock breeds are also
proving a vital resource in the struggle to 35 How does plant diversity help reduce
cope with more extreme weather such as crop failure?
95 droughts.
Krystyna Swiderska, Principal (A) If one variety of plant fails, there are
Researcher at IIED [International Institute likely to be others that succeed.
for Environment and Development] said: (B) Different communities of farmers can
“From China to Kenya, farmers have share their seeds to make sure at least
100 improved both resilience and productivity some seeds germinate.
by crossing resilient landraces with high-
(C) Some plants act as guards against
yielding modern varieties.”
others because they are more
resistant to insects.
32 What is the relationship between plant
(D) Different kinds of plants enable
diversity and climate change? scientists to experiment with plants
and save them in gene banks.
(A) Plant diversity can provide some
protection from crop failure due to
climate change.
(B) Increasing plant diversity with
ancient varieties of potatoes may
be more resilient to the higher
temperatures associated with climate
change.
(C) Some varieties of potatoes are more
adaptable to climate change than
others.
(D) The potato gene bank tries to ensure
plant diversity by saving seeds of all
varieties from extinction.

GO TO THE
NEXT PAGE
www.petersons.com
776 PART VII: Five Practice Tests

36 Which of the following is the best sum- 39 What evidence in the passage illustrates
mary of the passage? the effects of climate change in Peru?

(A) New farming techniques are helping (A) Lines 10–13 (“Rising
Peruvian farmers adapt to a changing temperatures … staple food”)
climate. (B) Lines 45–47 (“Planting a … crop
(B) Peruvian farmers are part of an failure”).
international group trying to fight the (C) Lines 59–63 (“Although gene …
effects of climate change. climatic changes”)
(C) Ancestral growing techniques (D) Lines 85–86 (“CIP is … the
combined with new scientific farmers”)
methods are proving to be useful
in combating the effects of climate
40 What evidence does the text provide to
change on Peruvian farmers.
show that potatoes were eaten by people
(D) Extreme weather conditions are
thousands of years ago?
proving difficult challenges for
farmers all over the world. (A) Lines 33–35 (“These include … the
Park”)
37 What are the implications of climate (B) Lines 38–41 (“Potato Park … over
change on world hunger? time”)
(C) Lines 48–49 (“This strategy …their
(A) It won’t be affected.
ancestors”)
(B) It may enable the development of
(D) Lines 50–53 (“The agreement …
new crops to feed the world.
been lost”)
(C) With the right scientific intervention,
it could alleviate hunger.
41 As used in line 32, “stewarding” most
(D) It may get worse.
nearly means

38 Why would the Peruvian farmers want (A) managing.


other nations to advocate for the UN (B) controlling.
summit to take steps to mandate that (C) regulating.
countries act to alleviate climate change?
(D) developing.
(A) Peru is a small country and doesn’t
have much power. 42 As used in line 15, “pronounced” most
(B) Climate change is directly affecting nearly means
their livelihoods and their ability to
feed their population. (A) articulated.
(C) Peru won’t be attending the (B) official.
conference. (C) well-known.
(D) Because of its high altitude, climate (D) noticeable.
change has a larger impact on
Peruvian farmers.

Master the New SAT®


Practice Test 4 777

practice test
QUESTIONS 43–52 ARE BASED ON THE the famous Trojan war, the theme of the
FOLLOWING PASSAGE. greatest poems of antiquity, those of
Homer and Virgil.
In his leisure time, Thomas Bulfinch (1796– 45 Menelaus called upon his brother
1867) enjoyed writing condensed summaries chieftains of Greece to fulfill their pledge,
of classical literature. Written for the general and join him in his efforts to recover his
reader, “to popularize mythology and extend wife. They generally came forward, but
the enjoyment of elegant literature,” his three Ulysses, who had married Penelope, and
volumes of classics were published under the 50 was very happy in his wife and child,
title Bullfinch’s Mythology. This passage is an had no disposition to embark in such a
excerpt from chapter XXVII, The Trojan War. troublesome affair. He therefore hung
Minerva was the goddess of wisdom, back and Palamedes was sent to urge
but on one occasion she did a very foolish him. When Palamedes arrived at Ithaca,
thing; she entered into competition with 55 Ulysses pretended to be mad. He yoked
Line Juno and Venus for the prize of beauty. It an ass and an ox together to the plough
5 happened thus: At the nuptials of Peleus and began to sow salt. Palamedes, to try
and Thetis all the gods were invited him, placed the infant Telemachus before
with the exception of Eris, or Discord. the plough, whereupon the father turned
Enraged at her exclusion, the goddess 60 the plough aside, showing plainly that he
threw a golden apple among the guests, was no madman, and after that could no
10 with the inscription, “For the fairest.” longer refuse to fulfill his promise. Being
Thereupon Juno, Venus, and Minerva now himself gained for the undertaking,
each claimed the apple. Jupiter, not he lent his aid to bring in other reluctant
willing to decide in so delicate a matter, 65 chiefs, especially Achilles. This hero was
sent the goddesses to Mount Ida, where the son of that Thetis at whose marriage
15 the beautiful shepherd Paris was tending the apple of Discord had been thrown
his flocks, and to him was committed among the goddesses. Thetis was herself
the decision. The goddesses accordingly one of the immortals, a sea-nymph, and
appeared before him. Juno promised him 70 knowing that her son was fated to perish
power and riches, Minerva glory and before Troy if he went on the expedition,
20 renown in war, and Venus the fairest of she endeavoured to prevent his going.
women for his wife, each attempting to She sent him away to the court of King
bias his decision in her own favor. Paris Lycomedes, and induced him to conceal
decided in favour of Venus and gave 75 himself in the disguise of a maiden
her the golden apple, thus making the among the daughters of the king. Ulysses,
25 two other goddesses his enemies. Under hearing he was there, went disguised as
the protection of Venus, Paris sailed to a merchant to the palace and offered for
Greece, and was hospitably received by sale female ornaments, among which
Menelaus, king of Sparta. Now Helen, the 80 he had placed some arms. While the
wife of Menelaus, was the very woman king’s daughters were engrossed with the
30 whom Venus had destined for Paris, the other contents of the merchant’s pack,
fairest of her sex. She had been sought as Achilles handled the weapons and thereby
a bride by numerous suitors, and before betrayed himself to the keen eye of
her decision was made known, they all, 85 Ulysses, who found no great difficulty in
at the suggestion of Ulysses, one of their persuading him to disregard his mother’s
35 number, took an oath that they would prudent counsels and join his countrymen
defend her from all injury and avenge her in the war.
cause if necessary. She chose Menelaus,
and was living with him happily when
Paris became their guest. Paris, aided by
40 Venus, persuaded her to elope with him,
and carried her to Troy, whence arose
GO TO THE
NEXT PAGE
www.petersons.com
778 PART VII: Five Practice Tests

43 Which of the following is a theme used 47 Based on the myth, what started the
in this myth? Trojan War?

(A) Humans use deception to achieve (A) A fight over an apple


goals. (B) A fight for the hand of Helen
(B) The gods have special powers of to (C) A test of Ulysses’ loyalty
control events.
(D) The marriage of Peleus and Thetis
(C) The bonds of marriage tie people’s
loyalties to one another.
48 What is the relationship between
(D) War is a dangerous undertaking that
Telemachus and Ulysses?
threaten human life.
(A) Telemachus was Ulysses’ son.
44 Which of the following could character- (B) Telemachus was a god and Ulysses
ize Ulysses’ decision not to participate in human.
the war? (C) Telemachus was a prince and Ulysses
was king of Sparta.
(A) Rational
(D) Telemachus was an old man and
(B) Wise
Ulysses was his son.
(C) Hypocritical
(D) Practical 49 Which of the following lines from the
text explain why the Spartans go to war
45 Why was the golden apple referred to as with the Trojans?
the “Apple of Discord”?
(A) Lines 33–37 (“they all … if
(A) It was sought after by Eris. necessary.”)
(B) It started a war. (B) Lines 54–55 (“When Palamedes …
(C) It was rotten. be mad.”)
(D) It was claimed by several goddesses. (C) Lines 73–76 (“She sent … the
king.”).
46 Why did Thetis try to prevent Achilles (D) Lines 83–88 (“Achilles handled …
the war.”)
from going to Troy?

(A) She thought his presence might start 50 Which of the following lines from the
a war. text illustrate powers ascribed to the
(B) She didn’t want him to marry Helen. gods and goddesses in the myth?
(C) She wanted him to ask King
(A) Lines 8–9 (“the goddess … the
Lycomedes for his help.
guests”)
(D) She didn’t want her son to be killed
(B) Lines 25–27 (“Under the … to
in battle.
Greece”)
(C) Lines 68–69 (“Thetis was … the
immortals”)
(D) Lines 77–78 (“[Ulysses] went
disguised as … the palace”)

Master the New SAT®


Practice Test 4 779

practice test
51 As used in line 87, “prudent counsels” 52 As used in line 5, “nuptials” most nearly
most nearly mean means

(A) cautious plans. (A) wedding ceremony.


(B) wise advise. (B) beauty contest.
(C) legal documents. (C) spring ritual.
(D) sensible recommendations. (D) religious rite.

STOP
If you finish before time is called, you may check your work on this section only.
Do not turn to any other section.

www.petersons.com
Practice Test 4 817

answers Practice Test 4


ANSWER KEY AND EXPLANATIONS

Section 1: Reading Test

1. A 12. B 23. C 34. B 45. B


2. C 13. C 24. A 35. A 46. D
3. D 14. B 25. A 36. C 47. B
4. B 15. D 26. D 37. D 48. A
5. C 16. B 27. C 38. B 49. A
6. D 17. B 28. B 39. A 50. B
7. B 18. D 29. C 40. C 51. B
8. D 19. A 30. D 41. A 52. A
9. A 20. D 31. B 42. D
10. A 21. B 32. A 43. A
11. D 22. B 33. C 44. C

READING TEST RAW SCORE


(Number of correct answers)

1. The correct answer is (A). Hoover held that 4. The correct answer is (B). These lines
economic success is only possible if people show that Hoover not only was addressing
have economic freedom (lines 8–10) but he his immediate audience, he was communi-
justifies the stronger centralized govern- cating his ideas to the country as a whole.
ment, which he characterizes as socialist, He was arguing for a democratic form of
bordering on despotic, as necessary during government that was free from govern-
wartime (lines 14–15). ment interference, which he believed was
2. The correct answer is (C). Hoover thought the best kind of government. However, he
that the best way to have a strong economy acknowledges possible criticisms and ad-
was to allow individual freedom, but that dresses them head on by explaining that is
during wars, it was necessary to allocate not taking extreme positions and that his
resources to the war effort. Only the federal statements are generalities.
government is able to control resources so 5. The correct answer is (C). Hoover describes
that they could be put to best use depending the philosophy of European governments as
on need for the war. being made of “diametrically opposed”
3. The correct answer is (D). In these two doctrines—paternalism and socialism—that
paragraphs, Hoover explains that his gen- would undermine self-government and
eral statements are not to be interpreted individual initiative and enterprise.
as extreme positions, and in doing so, he 6. The correct answer is (D). The map shows
provides some details to clarify his position. the electoral votes being wildly in Hoover’s
His attitude remains the same. He does want favor, with him winning all but a handful of
to be understood, but that does not indicate states, mostly in the South. A landslide is
a change in the overall speech.

www.petersons.com
818 PART VII: Five Practice Tests

something won by a wide margin and thus 13. The correct answer is (C). As an abolition-
a decisive victory. ist, Stowe worked with others in the move-
7. The correct answer is (B). Hoover describes ment to try to get the practice outlawed. One
his strong beliefs about government and of the methods she used to accomplish this
economic liberty but tempers it by showing goal was to write about slavery in an effort
that the country needs to be flexible so that to persuade people to oppose it. Uncle Tom’s
when necessary, the government can change Cabin, although a work of fiction, told stories
sufficiently to control resources in order to about slavery that made people more aware
respond to a crisis, as in wartime. of its horrors.
8. The correct answer is (D). Hoover’s argu- 14. The correct answer is (B). In lines 29–38,
ment is that progress can only be achieved Stowe suggests that people were prone to
when people are free to pursue their own doubt the factual basis for Uncle Tom’s
economic interests without interference. Cabin. She describes the lengths she goes
However, none of the other statements to in order to research the facts about the
indicate a superiority to other forms of gov- slave experience and how she was even more
ernance. In this statement he says that the horrified than she expected. So she says that
result of American government policies has in her next book, she will document the facts
enabled the country to surpass other nations. as they were used in the novel to prove the
validity of the book.
9. The correct answer is (A). A labyrinth is a
maze or a puzzle, which in the context of the 15. The correct answer is (D). Douglass la-
sentence shows that Hoover is referring to ments about the loss of loved ones when he
the problems that will ensue if government thinks about his plan to escape. It states that
gets into business. The result, he says, will he believes this deep loss is a major reason
be a tangled mess of bureaucracy rather than that more slaves don’t attempt to flee. All
a smooth operation. of the other reasons in choices (A), (B), and
(C) are also true, but choice (D) is the main
10. The correct answer is (A). In the context
reason Douglass cites.
of this sentence, the word regiment refers to
the organization of people or government. 16. The correct answer is (B). The addition of
Although the context is military, the regi- “An American Slave, Written by Himself”
men here refers to the civilian population. reinforces the idea that it was unusual for a
slave to write anything, no less a complete
11. The correct answer is (D). Both passages
book. We can infer from the inclusion of this
describe some kind of loss that is associated
information in the title that Douglass wanted
with slavery. Stowe recalls how she was
to make sure no one thought someone else
deeply affected by the loss of her young
had written the work.
child, and she relates this experience to that
of slaves who are often separated from their 17. The correct answer is (B). The passages
families. Douglass describes the anxiety are but two examples of how abolitionists
around losing one’s loved ones when escap- were drawing attention to the plight of the
ing from slavery, a permanent loss since once slaves. Stowe discusses how she planned to
escaped, there would be no way to return to write more, and the introductory material
visit or to keep in contact. describes Uncle Tom’s Cabin as a best-seller,
indicating that it was widely read. The fact
12. The correct answer is (B). Stowe describes
that Douglass’ book was published shows
the loss of her young child and relates it to
that people were interested in his story. The
how slaves must feel when they lose their
popularity of these works suggests that there
children in the slave market. Stowe’s loss
were also many others who were doing
was traumatic, and she recognizes the depth
similar work to advance the cause.
of the loss regardless of circumstance. Be-
cause Douglass escaped and would never 18. The correct answer is (D). The only
again see his loved ones, Stowe would be meaning of the word that makes sense in
sympathetic to his feelings of loss. the sentence is choice (D). Here Stowe is
talking about the intense pain she felt when

Master the New SAT®


Practice Test 4 819

answers Practice Test 4


discovering that the slave experience was alert was sounded, triggering the plans put in
even worse than she had thought. Although place for such an event. The alarms are set so
exquisite can mean beautiful or delightful, in that the orbits can be closely monitored and
this context its meaning is intensely. steps taken to prevent a collision should they
19. The correct answer is (A). In this part of be needed. Over that weekend, the alarms
the narrative, Douglass describes his feelings went off, but no action was needed because
about leaving his friends when he escapes. it was determined that the satellites were not
These feelings are part of his struggle to that close to each other after all, a fact the
make the final decision to attempt the escape, scientists could not predict earlier. It was
so in this context, he is contending with his not a false alarm since it was a possibility
feelings of pain about the decision. at the time that the alarm went off, choice
(C). It turned out that it was not a very close
20. The correct answer is (B). In this part
call after all, choice (D).
of the passage, Stowe expresses her deep
sorrow and says that what would make her 25. The correct answer is (A). The passage
feel better would be knowing that she could describes the procedures that the scientists at
use her loss for some greater purpose. Her the lab put in place in case of emergency. In
belief in the evil of slavery and acting on this case, the emergency isn’t one of human
that belief through the abolitionist movement life at risk, but the lives of the satellites, the
was one way she used to come to terms with data they are sending to Earth, and the time
her child’s death. and money spent on the project could be at
risk if there were to be a collision. These
21. The correct answer is (B). The fact that
are the same kinds of steps taken for people
Stow felt she had to document what she
who make plans in case of disasters such as
wrote in the book indicates that some people
floods, hurricanes, or major fires.
questioned the validity of the premise of
Uncle Tom’s Cabin. Her research lead her 26. The correct answer is (D). The two natural
to discover, not that the book got it wrong, satellites or moons can be determined by
but that the treatment of slaves was even eliminating the satellites that are named in
worse than she had imagined when writing the passage and using the graphic to identify
Uncle Tom’s Cabin. Choice (B) suggests all of the satellites in orbit.
that whatever ideas Stowe recorded in the 27. The correct answer is (C). In the graphic,
book were incomplete, and she needed more you can see that Deimos, one of the natu-
work to further the truth. ral satellites, orbits Mars at the greatest
22. The correct answer is (B). The main idea distance. It intersects with only one other
of the passage is how NASA and its partners satellite’s orbit, making it least likely to
monitor the satellites orbiting around Mars have a collision.
to make sure they don’t collide. A summary 28. The correct answer is (B). An elongated
must include the main idea along with some orbit means that the distance from the orbit to
other information that provides an overview Mars changes as the satellite makes its way
of the entire text. Choice (B) includes both around the planet. Because it is constantly
the main idea and the detail that explains the changing, it can’t be predicted accurately.
reason the satellites need monitoring. The 29. The correct answer is (C). A satellite
passage doesn’t discuss gathering informa- could collide if its path in orbit crosses the
tion, choice (A), and choice (D) is a detail. path in orbit of another satellite at the same
23. The correct answer is (C). The passage time. Choice (C) explains this relationship.
details the various spacecraft currently in None of the other choices describe this
orbit around Mars and their sponsoring relationship.
countries. Four belong to the United States, 30. The correct answer is (D). Conjunction
one to India, and one to the European Union. here is used to describe the meeting of the
24. The correct answer is (A). The passage celestial bodies, that is, when the satellites
describes a close call in which a collision are aligned. It does not refer to a collision;

www.petersons.com
820 PART VII: Five Practice Tests

rather, the conjunction is the alignment. If least one crop that succeeds and can feed
the conjunction is hazardous, it means the people. Sharing information and seeds is
satellites may be colliding. If conjunction important, but that doesn’t in itself reduce
included in its meaning the idea of collision, crop failure, choice (B), nor do gene banks
the word hazardous would be not necessary. guarantee that a particular variety of plant
31. The correct answer is (B). The word future will succeed where others have failed,
explains when the data will be used, from choices (C) and (D).
which it can be inferred that projection is 36. The correct answer is (C). Choice (C) is
something forecast for later or at a future best because it includes the main idea of the
time. In this case the data consists of forecasts passage—Peruvian farmers are working to
about the paths of the satellites in the future combat the effects of climate change—and
as they travel in their orbits. the idea that the strategies include both the
32. The correct answer is (A). Because climate old and the new. Choice (A) includes only
change is already affecting the potato crops one facet of this concept, and choices (B)
in Peru, especially those at high altitudes, and (D) are details that are not needed in a
diversity of crops can help determine which summary.
varieties are better suited for the higher 37. The correct answer is (D). The passage
altitudes and warmer temperatures. Some describes the effects of farming the potato,
ancient varieties could be resilient to warmer a staple crop for thousands of people, and
temperatures, but the statement does not ex- how climate change is reducing output of
plain how diversity of crops can be resilient the crop. Although farmers and scientists are
to the effects of climate change—choice (B). working together in Peru and other places,
Choice (C) is true of higher temperatures, it will be challenging to stay ahead of the
but it does not explain how crop diversity changes already occurring. These changes,
is related to climate change. Choice (D) as described in the passage, for example,
does not explain how the gene bank and having to move crops to higher altitudes,
preserving seeds from extinction is related may have limited use; as the article states,
to climate change. the land at higher altitudes is scarce and
33. The correct answer is (C). Diversity of rockier. The passage also states that efforts
crops can help alleviate disease and enhance to maintain a gene bank to increase crop di-
adaptation to different environments, and the versity are limited, and sometimes the crops
farmers are using more diverse plants and don’t respond well and/or not all farmers
seedlings. This tactic will help fight crop have access to the gene banks. These factors
failure and also help the crops to adapt to suggest that the problem could grow worse.
changing conditions as the farmers experi- 38. The correct answer is (B). Because they are
ment with breeds of plants and seeds. The deeply affected by the changes in climate,
partnerships with scientists are helpful, but Peruvian farmers have a big stake in what is
they are not necessarily all related to climate decided at the UN summit. If enough nations
change. The passage does not suggest that agree to pursue policies that will help allevi-
the farmers are trying out a different kind ate the effects, the farmers will be helped.
of staple food, choice (D). The problem is worldwide, as the passage
34. The correct answer is (B). The passage suggests, so the wishes of the farmers is not
describes how the farmers had to move dependent on the size of the country or who
their crops to higher altitudes for them to attends the conference. The last paragraph
survive because the temperature was too high indicates that there are deep problems faced
where they had been planted. This suggests by countries around the world and it is not
that the potatoes need cooler temperatures confined to issues of high altitudes.
since the air temperature would be cooler 39. The correct answer is (A). The text states
at higher altitudes. clearly the relationship between rising tem-
35. The correct answer is (A). Diversity is peratures—one aspect of climate change—
important to ensure that there is always at and the increase in farming problems like

Master the New SAT®


Practice Test 4 821

answers Practice Test 4


pest control and disease. Crop diversity, when he was called upon to do the same, he
choice (B) is one response to climate change, opted out because he didn’t want to upset his
as are gene banks and seed sharing, choice happy life. This decision was hypocritical,
(C) and sharing information and research, choice (C). Although he may have viewed
choice (D). his decision as rational, practical, or wise—
40. The correct answer is (C). The word “an- choices (A), (B), and (D)—those answers
cestors” suggests something that or someone are not correct. Ulysses eventually changed
who existed hundreds of generations ago, his mind and joined forces with the others
so the sentence in choice (C) indicates that in their attempt to capture Helen.
the potato is a food that was grown for food 45. The correct answer is (B). The apple began
perhaps thousands of years ago. Choices (A) the dispute among the goddesses, each try-
and (D) refer to past practices, but neither ing to prove that that she was “the fairest of
took place that long ago. all.” This set off a chain of events in which
41. The correct answer is (A). This sentence Helen was taken from her home by Paris,
refers to how the farmers care for their na- who wanted her for his wife. An earlier pact
tive plants; they are managing them as they among the Spartans had stipulated that they
try to find breeds that are hardier at higher would defend Helen at all costs. The war was
altitudes and withstand some of the effects an effort to recapture Helen from Paris and
of higher temperatures. None of the other bring her back to her husband Menelaus,
words convey this exact meaning—the farm- the King of Sparta.
ers can’t control the seedlings, choice (B); 46. The correct answer is (D). Lines 70–71
nor can they regulate them, choice (C). The states that Thetis was afraid that Achilles
farmers are not developing the crops, choice might perish if he went to war. The battle
(D); scientists are working on that aspect. was already beginning, so choice (A) is
42. The correct answer is (D). In some con- incorrect. Her objections were based on
texts, the word pronounced can mean to her fear for his safety, not any ideas about
articulate, choice (A), but here it is used his future marriage plans, making choice
to refer to how noticeable the temperature (B) incorrect. Her plan for Achilles to go
changes are, choice (D). None of the other to King Lycomedes was not to ask for help,
choices provide a meaning that makes sense choice (D), but rather so he could avoid
in the sentence. being drafted to fight by concealing his
true identity.
43. The correct answer is (A). The myth in-
cludes two examples of deception to achieve 47. The correct answer is (B). Paris wanted the
goals: one when Thetis sends Achilles to much-prized Helen for his wife. Although
the court of King Lycomedes disguised as she was married to Menelaus, he persuaded
a young girl and the other when Ulysses her to go with him and leave the palace, set-
disguises himself as a merchant to gain ting in motion a pact agreed to by the men
access to the royal palace. Both acts show of Sparta to protect the beautiful Helen.
people trying to deceive others to get what The men set out to bring Helen back to her
they want. The gods aren’t shown to control rightful husband.
events so much as cleverly intervene in 48. The correct answer is (A). Lines 58–59
human endeavors, so choice (B) is wrong. reveal that Telemachus is Ulysses’ son when
Marriage loyalties are shown as bonds that the young child is put in danger, and Ulysses
can be broken (Helen willingly went off with had to drop his guise as a madman to save
Paris though she was married to Menelaus), him. Both Telemachus and Ulysses were
making choice (C) incorrect. There are humans, but neither was royalty, making
several references to the dangers of war, choices (B) and (C) incorrect.
choice (D), but the men are encouraged to 49. The correct answer is (A). It describes the
join the war efforts. pledge made by the Spartans to find Helen
44. The correct answer is (C). Ulysses had and bring her back to Sparta. It was that
urged others to swear to defend Helen, but oath to defend Helen that started the war.

www.petersons.com
822 PART VII: Five Practice Tests

Ulysses’ pretense at anger was meant to 51. The correct answer is (B). The context of
deflect Palamedes’s requests that Ulysses the sentence indicates that prudent counsels
join forces with Achilles—choice (B). It is associated with something told to Achil-
was an action that contributed to the start of les by his mother. His mother had advised
the war. Thetis’s disguise was designed to Achilles to adopt the disguise to avoid
save him from fighting—choice (C)—it was participating in the war. A mother’s counsel
not a cause of the war. Choice (D) describes is generally thought of as wise, though in
how Ulysses persuaded Achilles to join the this case Ulysses may be questioning that
cause; the war had already been declared. description since he wants Achilles to ignore
50. The correct answer is (B). One of the roles her advice.
of the gods and goddesses was to protect 52. The correct answer is (A). Nuptials refers
the humans from their own human foibles to wedding ceremonies. This can be inferred
a well as from dangers from one another. from the context, which makes it clear that it
Lines 25–27 show shows that the gods were is an invitation-only event related to Peleus
offering to protect Paris on his journey. None and Thetis. There is nothing that suggests a
of the other choices illustrate text that shows religious context, choice (D), nor a ritual,
the powers of the gods. Choice (A) shows choice (C). The beauty contest, choice (B),
Eris in action, but it does not illustrate her was not part of the nuptials; the contest
powers. Choice (C) defines Thetis as one occurred when Eris was angry at being
of the gods, and choice (D) describes the excluded.
actions of a human.

Master the New SAT®


Practice Test 5 861

practice test
SECTION 1: READING TEST
65 Minutes • 52 Questions

TURN TO SECTION 1 OF YOUR ANSWER SHEET TO ANSWER THE QUESTIONS IN THIS


SECTION.

Directions: Each passage (or pair of passages) below is followed by a number of multiple-choice
questions. After reading each passage, select the best answer to each question based on what is
stated or implied in the passage or passages and in any supplementary material, such as a table,
graph, chart, or photograph.

QUESTIONS 1–11 ARE BASED ON THE now poses a serious threat to the native
FOLLOWING PASSAGE. snails and plants of Galapagos.
Invasive Giant African Land Snails
Snail-Sniffing Dogs in the Galapagos 20 were first detected on Santa Cruz Island
The Galapagos Islands, which belong to Ecuador, in 2010, and currently less than 20
are located approximately 906 km (563 mi.) hectares (50 acres) are infested—but the
west of the mainland. Because of their isolation, snails are expanding their range every wet
these volcanic islands are home to a variety of season. Experience has shown that once
unique species, such as gigantic land tortoises 25 an invasive species becomes established,
and marine iguanas. As more people have visited it is almost impossible to remove. At
and settled on the islands, however, it has become this point in time, it is still possible to
increasingly difficult to protect the native plants eradicate the GALS from Galapagos if
and animals. additional management techniques are
30 integrated into current activities.
The following text has been adapted from “Eco- Previously, staff from the Galapagos
system Restoration: Invasive Snail Detection Agency for the Regulation and Control
Dogs,” which was originally published by of Biosecurity and Quarantine (ABG)
Galapagos Conservancy (www.galapagos.org). had to search for and collect GALS
Galapagos Conservancy is a conservation group 35 on rainy nights using headlamps—an
that collaborates with scientists worldwide to extremely challenging and unsustainable
ensure protection of the Galapagos Islands. solution to the permanent eradication of
(For the complete article, see http://galapagos. the snails. Dogs, on the other hand, have
org/conservation.) an incredible sense of smell and can be
In Galapagos, native species are 40 trained to detect scents imperceptible to
threatened by introduced, invasive the human nose, making them ideal for
species such as goats, rats, pigs, and cats, the detection of the GALS. Detection
Line among many others. While much has dogs have been used for finding
5 been accomplished in the management contraband drugs and shark fins in
of existing invasive species, the islands 45 Galapagos, but not for other purposes.
are constantly at risk of new unwanted This project entails utilizing two scent
species arriving each day. The Giant detection dogs to detect GALS in order
African Land Snail (GALS)—the largest to help clear currently affected areas
10 species of snail found on land, growing and search for previously undetected
to nearly 8 inches in length—is one such 50 populations in the islands.
new invasive that has taken up residence During the first phase of the project,
in Galapagos. Known to consume at least which took place in the fall of 2014, two
500 different types of plants, scientists detection dogs were trained by Dogs for
15 consider the GALS to be one of the most Conservation (DFC) in the United States
destructive snail species in the world. It 55 to specifically detect GALS. Darwin, a
GO TO THE
NEXT PAGE
www.petersons.com
862 PART VII: Five Practice Tests

golden Labrador retriever, was rescued 1 What is the purpose of this article?
after he was unable to successfully
complete a service dog training program, (A) To bring tourists to the Galapagos
Line and Neville, a black Labrador retriever, (B) To raise money for the organization
60 was saved from a shelter. Darwin and
(C) To inform the public about the
Neville were selected for this project
problems of invasive species
based on their detection abilities and
temperament for working with multiple (D) To persuade people that it is
handlers, in preparation for work with important to keep species of animals
65 new handlers in Galapagos. In December and plants from becoming extinct
of 2014, the dogs were brought to
Galapagos where six ABG staff were 2 Which statement best represents the main
trained as handlers for this and future idea of the passage?
detection projects. Many had never
70 worked with dogs before and had to learn (A) Dogs can help reduce invasive
the basics of canine behavior, learning species in the Galapagos.
theory, scent theory, training methods, (B) Dogs can be trained to hunt for
and handling skills. New kennels were snails.
built by ABG personnel with materials
(C) Galapagos ecosystems include
75 funded through this project in order to
unique species.
house the dogs.
Both dogs required a period of (D) Organizations are working together
acclimation to Galapagos and to their new to rid the Galapagos of invasive
roles. The dogs could only be trained on species.
80 dead snails in the U.S. due to biosecurity
risks for this highly invasive species, so 3 Why do the Galapagos have a unique
additional training was needed upon their ecosystem?
arrival in Galapagos to transition them
to live snails and snail eggs. Darwin and (A) Islands can only support certain kinds
85 Neville have now been fully trained to of species.
detect the invasive snails, and the dogs (B) There were no mammals there until
will be regularly assisting with GALS humans brought them.
eradication and monitoring on Santa
(C) Only certain types of animals and
Cruz.
plants can live there because of the
90 DFC continues to provide guidance
climate.
and support to the GALS K9 team, with
whom they are in weekly communication. (D) It was isolated for a long time, so
Future updates to the project will be humans did not interfere with the
posted … as they occur. This project native species.
95 is also serving as a pilot to establish a
permanent canine detection program in 4 Which best describes the threat of GALS
the Galapagos. Expertly trained dogs and to the Galapagos islands?
experienced handlers will be a highly
cost-effective detection tool for ongoing (A) Their growth patterns
100 biosecurity programs aimed at eliminating (B) Their eating habits
targeted invasive species that threaten
(C) Their ability to hide from detection
the unique and fragile ecosystems of
Galapagos. (D) Their ability to survive in hot
climates

Master the New SAT®


Practice Test 5 863

practice test
5 How have people increased the fragility of 8 Why did the scientists decide to try using
the ecosystems in Galapagos? dogs to find the GALS?

(A) They brought trained dogs to the (A) They are friendly animals that are
islands. easy to train.
(B) They added to the wealth and (B) They can also be trained to find illicit
attractiveness of the islands. drugs.
(C) They introduced nonnative species to (C) They can smell things that humans
the islands. can’t.
(D) They made others aware of the native (D) They didn’t require special training
species on the islands. for the handlers.

6 What evidence does the passage provide 9 As used in lines 80–100, “biosecurity”
to explain the difficulty of maintaining the most nearly means
ecosystems in the Galapagos?
(A) safe handling of animals.
(A) Lines 13–16 (“Known to … the (B) safety from dangerous animals and
world”) plants.
(B) Lines 24–26 (“Experience has … to (C) protection to keep wildlife from
remove”) extinction.
(C) Lines 31–38 (“Previously, staff … (D) protection of an ecosystem from
the snails”) invasive species.
(D) Lines 79–81 (“The dogs …
biosecurity risks”) 10 As used in line 78, “acclimation” is best
defined as
7 What information from the text answers
the question of how dogs contribute to (A) adjusting to changes in the
biosecurity in the Galapagos? environment.
(B) conforming to one’s surroundings.
(A) Lines 38–41 (“Dogs, on … human
(C) adaptation of a species.
nose”)
(D) modification of behavior.
(B) Lines 52–55 (“Two detection …
detect GALS”)
11 Why are dogs considered an invasive spe-
(C) Lines 79–81 (“The dogs … invasive
species”) cies to the Galapagos?
(D) Lines 97–101 (“Expertly trained … (A) The dogs’ sense of smell helps them
invasive species”) find native species and use them for
food.
(B) The dogs there required time to get
acclimated to the environment.
(C) The dogs did not inhabit the
Galapagos until brought there by
humans.
(D) The dogs once thrived on the
Galapagos, but they had depleted
their limited food sources.

GO TO THE
NEXT PAGE
www.petersons.com
864 PART VII: Five Practice Tests

QUESTIONS 12–22 ARE BASED ON THE cooperate and make modest sacrifices, if
FOLLOWING PASSAGE. we learn to live thriftily and remember
the importance of helping our neighbors,
The following is an excerpt from a speech given 45 then we can find ways to adjust and to
by President Jimmy Carter, spoken and broadcast make our society more efficient and our
from the White House library two weeks after he own lives more enjoyable and productive.
took office. Prior to Carter’s election, the country Utility companies must promote
had faced a severe oil shortage and rising prices conservation and not consumption. Oil
for oil and related products. 50 and natural gas companies must be honest
Report to the American People with all of us about their reserves and
(February 2, 1977) profits. We will find out the difference
The extremely cold weather this winter between real shortages and artificial
has dangerously depleted our supplies ones. We will ask private companies to
of natural gas and fuel oil and forced 55 sacrifice, just as private citizens must do.
Line hundreds of thousands of workers off the All of us must learn to waste
5 job. I congratulate the Congress for its less energy. Simply by keeping our
quick action on the Emergency Natural thermostats, for instance, at 65 degrees in
Gas Act, which was passed today and the daytime and 55 degrees at night we
signed just a few minutes ago. But the 60 could save half the current shortage of
real problem—our failure to plan for the natural gas.
10 future or to take energy conservation There is no way that I, or anyone else
seriously—started long before this winter, in the Government, can solve our energy
and it will take much longer to solve. problems if you are not willing to help.
I realize that many of you have not 65 I know that we can meet this energy
believed that we really have an energy challenge if the burden is borne fairly
15 problem. But this winter has made all of among all our people—and if we realize
us realize that we have to act. that in order to solve our energy problems
Our program will emphasize we need not sacrifice the quality of our
conservation. The amount of energy 70 lives.
being wasted which could be saved is The Congress has made great
20 greater than the total energy that we are progress toward responsible strip-mining
importing from foreign countries. We will legislation, so that we can produce more
also stress development of our rich coal energy without unnecessary destruction
reserves in an environmentally sound 75 of our beautiful lands. My administration
way; we will emphasize research on will support these efforts this year. We
25 solar energy and other renewable energy will also ask Congress for its help with
sources; and we will maintain strict legislation which will reduce the risk of
safeguards on necessary atomic energy future oil tanker spills and help deal with
production. 80 those that do occur.
The responsibility for setting energy I would like to tell you now about one
30 policy is now split among more than of the things that I have already learned
50 different agencies, departments, and in my brief time in office. I have learned
bureaus in the Federal Government. Later that there are many things that a President
this month, I will ask the Congress for its 85 cannot do. There is no energy policy that
help in combining many of these agencies we can develop that would do more good
35 in a new energy department to bring order than voluntary conservation. There is no
out of chaos. Congressional leaders have economic policy that will do as much as
already been working on this for quite a shared faith in hard work, efficiency, and
while. 90 in the future of our system
We must face the fact that the energy
40 shortage is permanent. There is no way
we can solve it quickly. But if we all

Master the New SAT®


Practice Test 5 865

practice test
12 What is the most likely reason that 16 What does Carter think should be the
Carter gave this speech? foundation for conserving energy?

(A) He wanted to reassure people that as (A) The renewed use of strip mining to
president he was going to make the produce more energy
United States energy independent. (B) The reduction of indoor temperatures
(B) As a new president, he wanted to to conserve fuel
start a dialogue with the people. (C) The use of solar and other renewable
(C) He wanted to explain the severity of energy sources
the energy crisis and what needed to (D) A voluntary policy in which people
be done to address it. share in creating efficiency
(D) He wanted people to understand the
limitations of the president. 17 How does Carter try to convince the
public that everyone needs to participate
13 What is the theme of the speech? to solve the problem?

(A) The United States has a long-term (A) He describes how utility companies
energy problem. are also promoting conservation, not
(B) There are many ways to conserve consumption.
energy. (B) He explains that people waste a lot of
(C) People need to use less energy in energy.
their homes. (C) He tells the public to lower the
(D) The government cannot solve thermostats in their homes, which can
environmental problems. save natural gas.
(D) He explains that sacrifices must be
14 Which of the following actions does made by everyone alike.
Carter propose that the government take
to help solve the problem? 18 Which of the following best explains the
tone of the speech?
(A) Lowering the thermostats
(B) Forming a new energy department (A) Carter speaks bluntly about the
problem, but also tries to be
(C) Developing coal reserves
persuasive and optimistic in order to
(D) Protecting the environment from oil encourage everyone to work together.
spills
(B) Carter is speaking on national
television and wants his audience to
15 Which of the following best represents keep listening to him, so his tone is
the belief system illustrated in Carter’s light and informal, even though the
speech? topic is a serious one.
(C) Carter is deeply concerned over the
(A) The idea that the United States
energy problem so the tone of the
should be energy independent
speech is stern and authoritative
(B) The concept of shared sacrifice because he wants people to follow his
(C) The idea that the environment needs requests.
to be protected by volunteers (D) Carter wants to be taken seriously,
(D) The concept of equal powers among so he avoids persuasive language;
the three branches of government instead, he speaks with informative,
matter-of-fact neutrality.

GO TO THE
NEXT PAGE
www.petersons.com
866 PART VII: Five Practice Tests

19 What language in the speech indicates that 22 Explain how the word “reserves” is used
Carter has great respect for the land and the in line 21 and line 53.
concept of conservation?
(A) In the first use, “reserves” refers
(A) Lines 21–28 (“We will … energy to something protected in order
production”) to prevent easy access to it; in the
(B) Lines 39–40 (“We must … is second use, it refers to something that
permanent”) is difficult to obtain.
(C) Lines 73–75 (“we can … beautiful (B) In the first use, “reserves” refers to
lands”) something discarded; in the second
use; it refers to something set aside in
(D) Lines 68–70 (“in order … our lives”)
case of emergencies.
(C) In the first use, “reserves” refers to
20 Which of the following might be used
something saved in case of future
by some people as evidence to claim that needs; in the second use, it refers to
Carter was anti-business? being set aside in order to raise prices.
(A) Lines 17–18 (“Our program … (D) In the first use, “reserves” refers to
emphasize conservation”) something not used because there is
an excess; in the second use, it refers
(B) Lines 26–28 (“we will … energy
to something that belongs to someone
production”)
else.
(C) Lines 39–30 (“We must … is
permanent”) QUESTIONS 23–32 ARE BASED ON THE
(D) Lines 49–52 (“Oil and … and FOLLOWING PASSAGE.
profits”)
Rudyard Kipling (1865–1936) was one of the
most popular English writers of his era, authoring
21 As used in line 2, “depleted” most nearly
stories, novels, and poems, many of which take
means place in colonial India, where he was born and
lived as a young child and returned to as a young
(A) consumed.
adult. “The Arrest of Lieutenant Golightly,” one
(B) replaced. of his earliest stories, was first published in an
(C) weakened. English language newspaper in India where
(D) wasted. Kipling worked as a journalist. The following
is an excerpt from that story.
If there was one thing on which
Golightly prided himself more than
another, it was looking like “an Officer
Line and a gentleman.” He said it was for
5 the honor of the Service that he attired
himself so elaborately; but those who
knew him best said that it was just
personal vanity. There was no harm about
Golightly. … He recognized a horse
10 when he saw one, … he played a very
fair game at billiards, and was a sound
man at the whist-table. Everyone liked
him; and nobody ever dreamed of seeing
him handcuffed on a station platform as a
15 deserter. But this sad thing happened.
He was going down from Dalhousie,
at the end of his leave—riding down. He

Master the New SAT®


Practice Test 5 867

practice test
had cut his leave as fine as he dared, and 70 was flapping on his neck and the sides
wanted to come down in a hurry. stuck to his ears, but the leather band and
20 It was fairly warm at Dalhousie [a green lining kept things roughly together,
town in India in the hills, used as a so that the hat did not actually melt away
summer retreat for British personnel] where it flapped.
and knowing what to expect below, he 75 Presently, the pulp and the green stuff
descended in a new khaki suit—tight made a sort of slimy mildew which ran
25 fitting—of a delicate olive-green; a over Golightly in several directions—
peacock-blue tie, white collar, and a down his back and bosom for choice.
snowy white solah [a plant made into The khaki color ran too … and sections
fabric used in hat-making] helmet. He 80 of Golightly were brown, and patches
prided himself on looking neat even when were violet, and contours were ochre, and
30 he was riding post. He did look neat, and streaks were ruddy red, and blotches were
he was so deeply concerned about his nearly white, according to the nature and
appearance before he started that he quite peculiarities of the dye. When he took out
forgot to take anything but some small 85 his handkerchief to wipe his face and the
change with him. He left all his notes at green of the hat-lining and the purple stuff
35 the hotel. His servants had gone down the that had soaked through on to his neck
road before him, to be ready in waiting at from the tie became thoroughly mixed,
Pathankote with a change of gear. the effect was amazing.
Twenty-two miles out of Dalhousie it 90 He went to the Station-Master to
began to rain—not a mere hill-shower, negotiate for a first-class ticket to Khasa,
40 but a good, tepid monsoonish downpour. where he was stationed. The booking-
Golightly bustled on, wishing that he clerk said something to the Station-
had brought an umbrella. The dust on Master, the Station-Master said something
the roads turned into mud, and the pony 95 to the Telegraph Clerk, and the three
mired a good deal. So did Golightly’s looked at him with curiosity. They asked
45 khaki gaiters. But he kept on steadily and him to wait for half-an-hour, while they
tried to think how pleasant the coolth was. telegraphed to Umritsar for authority.
His next pony was rather a brute at So he waited, and four constables came
starting, and Golightly’s hands being 100 and grouped themselves picturesquely
slippery with the rain, contrived to get rid round him. Just as he was preparing to
50 of Golightly at a corner. He chased the ask them to go away, the Station-Master
animal, caught it, and went ahead briskly. said that he would give the Sahib [term
The spill had not improved his clothes or of respect; like calling someone “sir” in
his temper, and he had lost one spur. He 105 English] a ticket to Umritsar, if the Sahib
kept the other one employed. By the time would kindly come inside the booking-
55 that stage was ended, the pony had had as office. Golightly stepped inside, and the
much exercise as he wanted, and, in spite next thing he knew was that a constable
of the rain, Golightly was sweating freely. was attached to each of his legs and arms,
At the end of another miserable half-hour, 110 while the Station-Master was trying to
Golightly found the world disappear cram a mailbag over his head.
60 before his eyes in clammy pulp. The rain
had turned the pith of his huge and snowy 23 Which of the following best describes
solah-topee into an evil-smelling dough,
the tone of the story?
and it had closed on his head like a half-
opened mushroom. Also the green lining (A) Disgust
65 was beginning to run.
(B) Mocking
Golightly did not say anything worth
recording here. He tore off and squeezed (C) Ironic
up as much of the brim as was in his eyes (D) Proud
and ploughed on. The back of the helmet
GO TO THE
NEXT PAGE
www.petersons.com
868 PART VII: Five Practice Tests

24 Even though the author says “this sad thing 27 Why did Kipling provide such detail
happened,” which detail from the passage about Golightly’s looks?
shows that Kipling considers Golightly to
be responsible for what happened to him? (A) To make fun of his vanity and usual
appearance
(A) He did look neat, and he was (B) To show how intense the climate is in
so deeply concerned about his India
appearance before he started that
(C) To help the reader understand the
he quite forgot to take anything but
setting
some small change with him.
(D) To describe the problems of the
(B) He prided himself on looking neat
British military in India
even when he was riding post.
(C) Golightly bustled on, wishing that he
28 How is lines 66–67 distinguished from
had brought an umbrella.
the rest of the text in the passage?
(D) His next pony was rather a brute at
starting, and Golightly’s hands being (A) Kipling describes the character’s
slippery with the rain, contrived to verbal response to the situation.
get rid of Golightly at a corner.
(B) The narrator interjects his own
viewpoint.
25 Which of the following best explains the
(C) It adds internal dialogue to the story.
identity and actions of the main character,
(D) The narrator describes Golightly’s
Golightly?
thoughts rather than his appearance.
(A) He’s a proper military man leaving
his post. 29 Which description of Golightly shows
(B) He’s a British soldier trying to escape that Kipling was making fun of the char-
capture by the Indian government. acter?
(C) He’s an outlaw trying to escape
(A) Lines 50–51 (“He chased … ahead
capture.
briskly”)
(D) He’s a British businessman on a trip
(B) Lines 69–74 (“The back … it
overseas.
flapped”)
(C) Lines 79–84 (“The khaki … the
26 How does Kipling make fun of his char-
dye”)
acter Golightly?
(D) Lines 84–89 (“When he … was
(A) His explanation of Golightly’s vanity amazing”)
is satiric.
(B) His description of Golightly’s 30 Which of the following lines from the
experience shows how he has trouble text show that Golightly is relatively
coping with the severe rainstorms in wealthy?
India.
(A) Lines 4–6 (“He said … so
(C) He gives a detailed description of
elaborately”)
how silly Golightly looks.
(B) Lines 35–36 (“His servants …
(D) He shows that Golightly had
before him”)
difficulty riding the horse.
(C) Lines 47–48 (“His next … at
starting”)
(D) Lines 105–107 (“if the … the
booking office”)

Master the New SAT®


Practice Test 5 869

practice test
31 What is the meaning of the sentence: Line in direct object the establishment of an
“He had cut his leave as fine as he 5 absolute tyranny over her. To prove this,
dared.” (lines 17–19) let facts be submitted to a candid world.
He has never permitted her to exercise
(A) He dared to take leave without telling her inalienable right to the elective
the authorities. franchise.
(B) He arranged to take as much time as 10 He has compelled her to submit to
he could without getting in trouble. laws, in the formation of which she had
no voice.
(C) He was daring in leaving the military
He has withheld from her rights
post because it was dangerous.
which are given to the most ignorant
(D) He wanted to leave, but was afraid 15 and degraded men—both natives and
he’d get caught. foreigners.
Having deprived her of this first right
32 Which of the following surrounding of a citizen, the elective franchise, thereby
words best help you figure out the mean- leaving her without representation in the
ing of the word “mired” (line 44)? 20 halls of legislation, he has oppressed her
on all sides.
(A) “the pony” He has made her, if married, in the eye
(B) “roads turned into mud” of the law, civilly dead.
He has taken from her all right in
(C) “wishing that he had brought an
25 property, even to the wages she earns.
umbrella”
He has made her, morally, an
(D) “So did Golightly’s khaki gaiters” irresponsible being, as she can commit
many crimes with impunity, provided
QUESTIONS 33–42 ARE BASED ON THE they be done in the presence of her
FOLLOWING TWO PASSAGES AND 30 husband. In the covenant of marriage, she
SUPPLEMENTARY MATERIAL. is compelled to promise obedience to her
husband, he becoming, to all intents and
Passage 1 is an excerpt from a speech modeled purposes, her master—the law giving him
on the Declaration of Independence, written and power to deprive her of her liberty, and to
read by Elizabeth Cady Stanton at the Woman’s 35 administer chastisement.
Rights Convention, held in Seneca Falls, New He has so framed the laws of divorce,
York, July 19, 1848. About 300 people attended as to what shall be the proper causes,
the event and about a third (68 women and 32 and in the case of separation, to whom
men) signed the declaration. the guardianship of the children shall
Passage 2 is excerpted from The Narrative 40 be given, as to be wholly regardless of
of Sojourner Truth, the memoir of a slave in the happiness of women—the law, in all
pre-Civil War New York. Born into slavery as cases, going upon a false supposition of
Isabella, after being freed in 1827, she took the the supremacy of man, and giving all
name Sojourner Truth to express her strong faith. power into his hands.
Because Truth was illiterate, she dictated her 45 After depriving her of all rights as a
story to the writer Olive Gilbert, whom she had married woman, if single, and the owner
met in Massachusetts. The book was published in of property, he has taxed her to support
1850 and was widely distributed by Abolitionists a government which recognizes her only
to help further their cause. when her property can be made profitable
50 to it ….
Passage 1 He has endeavored, in every way
Declaration of Sentiments that he could, to destroy her confidence
The history of mankind is a history in her own powers, to lessen her self-
of repeated injuries and usurpations on respect, and to make her willing to lead a
the part of man toward woman, having 55 dependent and abject life. …
GO TO THE
NEXT PAGE
www.petersons.com
870 PART VII: Five Practice Tests

80 But Isabella inwardly determined that


she would remain quietly with him only
until she had spun his wool—about one
hundred pounds—and then she would
leave him, taking the rest of the time to
85 herself. “Ah!” she says, with emphasis
that cannot be written, “the slaveholders
are TERRIBLE for promising to give you
this or that, or such and such a privilege,
if you will do thus and so; and when the
90 time of fulfillment comes, and one claims
the promise, they, forsooth, recollect
nothing of the kind; and you are, like as
not, taunted with being a LIAR; or, at
best, the slave is accused of not having
95 performed his part or condition of the
contract.” “Oh!” said she, “I have felt as
if I could not live through the operation
sometimes. Just think of us! so eager for
our pleasures, and just foolish enough
100 to keep feeding and feeding ourselves
up with the idea that we should get what
had been thus fairly promised; and when
(Credit: Women’s Political Union, we think it is almost in our hands, find
1911. Library of Congress) ourselves flatly denied! Just think! how
Passage 2 105 could we bear it?”

From: The Narrative of Sojourner Truth


After emancipation had been decreed 33 What was Stanton’s purpose in writing
by the State, some years before the time and speaking the Declaration of Senti-
fixed for its consummation, Isabella’s ments?
master told her if she would do well,
60 and be faithful, he would give her “free
(A) She wanted to show that women
could write important documents.
papers,” one year before she was legally
free by statute. In the year 1826, she had (B) She wanted to shock the audience in
a badly diseased hand, which greatly upstate New York.
diminished her usefulness; but on the (C) She wanted to show why women
65 arrival of July 4, 1827, the time specified needed rights.
for her receiving her “free papers,” she (D) She wanted to gain support for equal
claimed the fulfillment of her master’s protection of minorities.
promise; but he refused granting it, on
account (as he alleged) of the loss he had
70 sustained by her hand. She plead that
she had worked all the time, and done
many things she was not wholly able
to do, although she knew she had been
less useful than formerly; but her master
75 remained inflexible. Her very faithfulness
probably operated against her now, and he
found it less easy than he thought to give
up the profits of his faithful Bell, who had
so long done him efficient service.

Master the New SAT®


Practice Test 5 871

practice test
34 What is the effect of repeating the phrase 38 What do the two passages suggest about
“He has”? what women and slaves had in common?

(A) It shows how strongly she feels about (A) Neither women nor slaves could get
how women were treated. paid for their work.
(B) It emphasizes the transgressions of (B) Both women and slaves had to take
men against women. care of the children in a family.
(C) It makes the speech dull because it (C) Women had to obey their husbands;
repeats the same words. slaves had to obey their masters.
(D) It makes the speech more like the (D) Men made and broke promises to
Declaration of Independence. both women and slaves.

35 Which lines in the text are best illus- 39 Which of the following statements is true
trated by the graphic? about the two passages?

(A) Lines 13–16: (“He has withheld … (A) They both display a tone of anger at
and foreigners”) their lack of freedom.
(B) Lines 22–23: (“He has made … (B) Stanton’s tone is angry and Truth’s
civilly dead.”) tone is sad.
(C) Lines 42–44: (“ … going upon a … (C) Truth’s tone is bitter; Stanton’s tone
into his hands.”). is outrage.
(D) Lines 47–50: (“ … he has taxed … (D) They both show a tone of frustration.
profitable to it”).
40 Which of the following lines from the
36 How does Stanton support her argument text indicate why Truth says her master
that women have been forced into obedi- did not want to let her be free as he had
ence? promised?

(A) She explains that women can’t work (A) Lines 75–76 (“Her very
outside the home. faithfulness … her now.”)
(B) She expresses anger at the idea that (B) Lines 76–78 (“he found it … faithful
women are not allowed to vote. Bell”)
(C) She expresses dismay at how (C) Lines 86–89 (“the slaveholders …
children can be taken from mothers thus and so”)
in cases of divorce or separation. (D) Lines 93–96 (“at best … of the
(D) She explains how marriage legally contract.”)
compels women to obey their
husbands. 41 What set Isabella’s master apart from
other slaveholders?
37 What does Stanton mean by “elective
franchise” as used in line 18? (A) He didn’t mistreat her.
(B) He allowed her to learn to read and
(A) The sport of elections write.
(B) The business of elections (C) He finally did set her free before he
(C) The team needed for elections had to.
(D) The right to vote (D) He made promises he didn’t keep.

GO TO THE
NEXT PAGE
www.petersons.com
872 PART VII: Five Practice Tests

42 In lines 75–76, Truth says that her “faith- what the prehistoric predator would have
fulness probably operated against her.” 25 looked like. These animals were lightly
What does she mean by this phrase? built with long legs and jaws lined with
teeth, and they are believed to be very
(A) Her faith in God would help her distant relatives of today’s birds.
through the difficulties operating …
against her. 30 The specimen, roughly 75
(B) Her loyalty made her more important million years old, was discovered
and valuable to her master. by paleontologist Robert Sullivan in
the Bisti/De-Na-Zin Wilderness Area
(C) She needed to be faithful to her God
of New Mexico in 1999. When first
so that her master would not break
35 described, scientists believed it was a
his promises.
member of Saurornitholestes langstoni,
(D) She needed to be faithful in the face a species of theropod dinosaurs in the
of her master’s inflexibility. Dromaeosauridae family that had been
found in present-day Alberta, Canada.
QUESTIONS 43–52 ARE BASED ON 40 But when Jasinski … began a
THE FOLLOWING PASSAGE AND comparative analysis of the specimen to
SUPPLEMENTARY MATERIAL. other S. langstoni specimens, he found
subtle differences. Notably, he observed
The following passage has been adapted from that the surface of the skull corresponding
“New Dinosaur’s Keen Nose Made it a Formi- 45 with the brain’s olfactory bulb was
dable Predator, Penn Study Finds” by Katherine unusually large. This finding implies a
Unger Baillie, originally published by the Uni- powerful sense of smell.
versity of Pennsylvania, May 11, 2015. (This “This feature means that
passage was edited for length. For the complete Saurornitholestes sullivani had a
article, see http://upenn.edu/pennnews.) 50 relatively better sense of smell than
New Dinosaur’s Keen Nose Made It a other dromaeosaurid dinosaurs,
Formidable Predator including Velociraptor, Dromaeosaurus,
A researcher from the University of and Bambiraptor,” Jasinski said. “This
Pennsylvania has identified a species of keen olfaction may have made S. sullivani
dinosaur closely related to Velociraptor, 55 an intimidating predator as well.”
Line the group of creatures made infamous
S. sullivani comes from the end of the
5 by the movie Jurassic Park. The newly
time of dinosaurs, or the Late Cretaceous,
named species likely possessed a keen and represents the only named
sense of smell that would have made it a dromaeosaur from this period in North
formidable predator. 60’ America south of Montana.
Steven Jasinski, a doctoral student At the time S. sullivani lived, North
10 in the School of Arts & Science’s
America was split into two continents
Department of Earth and Environmental separated by an inland sea. This
Science at Penn, and acting curator of dinosaur lived on the western shores
paleontology and geology at the State 65 in an area called Laramidia. Numerous
Museum of Pennsylvania, discovered dromaeosaurs, which are commonly
15 the new species while investigating
called raptors, are known from more
a specimen originally assigned to a northern areas in Laramidia, including
previously known species. His analysis Alberta, Canada, and Montana. However,
suggests the fossil—part of the dinosaur’s 70 S. sullivani represents the only named
skull—actually represents a brand new dromaeosaur from the Late Cretaceous of
20 species, which Jasinski has named
southern Laramidia.
Saurornitholestes sullivani. The creature’s S. sullivani shared its world with
genus name Saurornitholestes, which numerous other dinosaurs. …Though a
means “lizard bird thief,” gives a sense of 75 distinct species, S. sullivani appears to be

Master the New SAT®


Practice Test 5 873

practice test
closely related to S. langstoni. Finding findings of related species suggest the
the two as distinct species further shows 85 animal would have been agile and fast,
that differences existed between dinosaurs perhaps hunting in packs and using its
between the northern and southern parts acute sense of smell to track down prey.
80 of North America. “Although it was not large, this was
At less than 3 feet at its hip and not a dinosaur you would want to mess
roughly 6 feet in length, S. sullivani was 90 with,” Jasinski said.
not a large dinosaur. However, previous

A pair of S. sullivani attack a young hadrosaur. (Illustration by Mary P. Williams)

43 Which of the following is the best state- 44 What conclusion can you draw from
ment of the main idea of the article? the way the S. sullivani species was
discovered?
(A) The S. sullivani was a predator aided
by its keen sense of smell. (A) Students can sometimes make
(B) The new species resembled today’s amazing discoveries.
birds. (B) Scientific investigations can yield
(C) Scientific discoveries make important surprising results.
contributions to our knowledge of (C) Students do important work with
prehistoric animals. academics in their field of studies
(D) Recent fossil evidence identified a (D) Some discoveries are attributable to
new species of dinosaur. chance.

45 Why would a strong sense of smell help


a predator?

(A) It would help a predator find food.


(B) It could help a predator sense danger.
(C) It would make up for poor eyesight.
(D) It could help a predator find a mate.

GO TO THE
NEXT PAGE
www.petersons.com
874 PART VII: Five Practice Tests

46 What does the name of the new species 49 Which lines from the text best describe
tell us about this dinosaur? part of the scientific process?

(A) Dinosaurs are named after real (A) Lines 34–36 (“When first …
people. Saurornitholestes langstoni”)
(B) The name tells us where the dinosaur (B) Lines 40–42 (“But when … S.
fossils were found. langstoni specimens”)
(C) Dinosaurs are given names based on (C) Lines 76–80 (“Finding the … North
where they lived. America”)
(D) The name suggests what it might (D) Lines 81–83 (“At less … large
have looked like. dinosaur”)

47 How did scientists recognize that the fos- 50 Which lines from the text best show
sil they found was a new species? what made Jasinski’s work newsworthy?

(A) The fossil was larger than others that (A) Lines 1–5 (“A researcher … movie
they had found previously. Jurassic Park.”)
(B) By comparing it to other fossils, they (B) Lines 17–20 (“analysis suggests …
noticed differences in the skull. new species.”)
(C) By looking at its closest relatives, (C) Lines 30–34 (“The specimen … New
they realized this was a different Mexico.”)
species. (D) Lines 61–63 (“At the time … an
(D) The fossil had different markings inland sea.”)
than other species.
51 As used in line 8, “formidable” most
48 How is the information in the passage nearly means
supported by the graphic?
(A) dreadful.
(A) It shows that S. sullivani lived about (B) inspiring.
75 million years ago.
(C) impressive.
(B) It shows that S. sullivani stood less
(D) solid.
than 3 feet at its hip and about 6 feet
in length.
52 What words in lines 43–47 provide
(C) It shows that S. sullivani had a very
keen sense of smell, which helped it a clue to the meaning of the word
hunt other dinosaurs. “olfactory”?
(D) It shows that S. sullivani had legs and (A) Bulb
feet that resemble those of birds, and
(B) Sense of smell
in fact may be distantly related to
birds. (C) Surface of the skull
(D) Unusually large

STOP
If you finish before time is called, you may check your work on this section only.
Do not turn to any other section.

Master the New SAT®


Practice Test 5 907

answers Practice Test 5


ANSWER KEY AND EXPLANATIONS

Section 1: Reading Test

1. C 12. C 23. B 34. B 45. A


2. A 13. A 24. A 35. A 46. D
3. D 14. C 25. A 36. D 47. B
4. B 15. B 26. C 37. D 48. D
5. C 16. D 27. A 38. C 49. B
6. B 17. B 28. B 39. D 50. C
7. D 18. A 29. D 40. D 51. C
8. C 19. C 30. B 41. C 52. B
9. D 20. D 31. B 42. B
10. A 21. A 32. B 43. D
11. C 22. C 33. C 44. C

READING TEST RAW SCORE


(Number of correct answers)

1. The correct answer is (C). The overall 4. The correct answer is (B). The article
idea of the article emphasizes the problem explains that the GALS is very destructive
of GALS, an invasive species that threatens because it eats so many different types of
the ecosystems of the islands. The article plants and animals. Once this snail enters
is informational and does not attempt to a population, it takes over by reproducing
persuade readers, choice (D), or encourage quickly and eating all the other species that
them to visit, choice (A), or raise money, keep the ecosystems in balance. While their
choice (B). ability to hide, choice (C), makes them dif-
2. The correct answer is (A). The main dis- ficult to find, the threat comes from their
cussion of the article is about how scientists eating habits.
are trying to contain and eliminate GALS by 5. The correct answer is (C). The article ex-
using trained dogs to detect them. Choice plains that people first introduced animals
(C) is a true statement, but it is not the that were nonnative to the islands. The
main idea of the article. Choice (D) is also fact that GALS and likely other species as
true, but the article does not focus on how well have taken over can be ascribed to the
organizations are collaborating; rather its people who arrive with plants and animals—
emphasis is on training dogs to help solve sometimes intentionally and sometimes
the specific problem of one invasive species. unintentionally. These nonnative species
3. The correct answer is (D). The article have a tendency to take over and wipe out
mentions the location of the islands, which the native species, upsetting the ecosystem.
are quite a distance from any mainland. This The trained dogs were brought in to help
fact of geography likely kept people from relieve the problem, choice (A). By visiting
settling there for a long time. This meant the islands both for research and tourism,
that the natural ecosystems were in balance. people have learned much about the native

www.petersons.com
908 PART VII: Five Practice Tests

species and shared this information with 11. The correct answer is (C). Dogs are not
people off the islands. This has not made native to the Galapagos, which are islands
the islands more fragile. far from any mainland, inhabited mostly by
6. The correct answer is (B). Choice (B) species unique to the islands, such as giant
shows how getting rid of invasive species tortoises and iguanas. Dogs were brought to
is a problem and invasive species often the islands by people to help them find the
upset the balance in an ecosystem. Finding invasive species that are upsetting the bal-
GALS is difficult, but that’s not why the ance found on the islands. The dogs’ sense
ecosystem is hard to maintain. Choice (A) of smell makes them good hunters for the
is true, but it is not why maintaining the invasive species on the islands, but that does
ecosystem is difficult. not explain why they are invasive, so choice
(A) is incorrect. The text explains that the
7. The correct answer is (D). Dogs can track
dogs had to be acclimated both to the islands
and find GALS effectively, and the cost to
and their trainers, but their adjustment was
train and maintain them is reasonable rela-
so that they could be trained properly for
tive to the service they provide, so choice
their role as hunters. This acclimation does
(D) is the correct answer. The other choices
not explain why they would be considered
provide additional information about the
invasive, so choice (B) is incorrect. Choice
dogs and their training, but they don’t an-
(D) is incorrect because the text does not say
swer the question of how dogs contribute
that dogs ever thrived on the islands or were
to biosecurity.
predators of any native species; it only stated
8. The correct answer is (C). Dogs have a that the dogs brought by the project helped
very highly developed sense of smell and in finding a particular invasive species so
can be trained to follow a scent that humans that it could be eradicated.
cannot detect. Dogs were not chosen based
12. The correct answer is (C). Carter explains
on their ease of interaction with people or
in the beginning of the speech how impor-
for their ability to find drugs. The specific
tant it was for people to grasp the severity
dogs described in the article were able to
of the energy problem. He then goes on to
be trained for the specific task. Choice (D)
explain how he plans to address the problem.
is not true because the handlers did require
Although energy independence was cited as
special training.
a goal, it was not a reason for his speech.
9. The correct answer is (D). “Biosecurity” is He could start a dialogue with the public
a combined form of the prefix bio- (meaning at any time, choice (B), so this speech was
life) and security. As used in the passage, not a casual, friendly dialogue. Carter does
the word refers to security from harmful mention at the end of his speech how he
biological species. The word is not related understands the limitations of the office,
to handling of plants or animals, choices but that idea is not the reason for address-
(A) and (B). The article does not mention ing the public.
extinction, choice (C); the word is not used
13. The correct answer is (A). The overrid-
in this context. Here, “biosecurity” is protec-
ing idea that Carter emphasizes is that the
tion of the ecosystems.
energy problem is not new; it is complex;
10. The correct answer is (A). “Acclimation,” it will take a long time to fix it; and many
as used in the context provided, describes people, the public included, will have to
how the dogs had to adjust to the Galapagos, work together to change it. There are many
which was not their native environment. ways to conserve energy, choice (B), but
“Conforming,” choice (B), suggests blend- this is not the specific focus of the passage.
ing in rather than adjusting. “Adaptation,” Carter’s urging people to use less energy in
choice (C), is a process that takes many their homes, choice (C), is a detail. Carter
years, not a few days or weeks. Training is never claims that government cannot solve
a type of behavior modification, choice (D), environmental problems, choice (D).
but it is unrelated to acclimation.

Master the New SAT®


Practice Test 5 909

answers Practice Test 5


14. The correct answer is (C). Carter proposes national problem. Because he recognized the
that people lower their thermostats, but that depth of the problem and needs to persuade
is not a government action. He also proposes the citizenry about participating, his tone
consolidating different agencies to form a new is not merely informational, so choice (D)
cabinet-level department, but that is not an is incorrect.
action that would solve the energy problem. 19. The correct answer is (C). Carter’s refer-
Protection from oil spills would help mitigate ence to the land of America as “beautiful”
pollution and stop the waste of energy, but it indicates how he respects the land and wants
is not mentioned as a way to solve the prob- to make sure it is not destroyed. The other
lem. Developing coal reserves is an example choices are examples of ways to fix the
of how the United States could decrease its problem, and they show Carter’s resolve
dependence on foreign sources of energy. in accomplishing the task, but they do not
15. The correct answer is (B). Throughout his specifically show the respect of and awe at
speech, Carter references the public and its the beauty of the land.
role in addressing the problem. He empha- 20. The correct answer is (D). Choice (D) sug-
sizes that the problem affects everyone and gests that Carter doesn’t trust oil and gas com-
everyone needs to participate to help solve panies because he is questioning their honesty.
it. He says that the sacrifices needed to be Some might interpret that as anti-business,
made by the public should be “borne fairly whether or not he meant it that way. None
among all our people” (lines 66–67). Choice of the other choices specifically and directly
(A) is true, but it is not a belief system. references companies and/or business.
Choice (D) is referenced when he explains
21. The correct answer is (A). In the context of
the limitations of presidential power, but
this sentence, depleted means “consumed or
that is not a belief system; it is a part of the
used up.” Carter is speaking about the overall
Constitution.
supplies, not individual waste. The supplies
16. The correct answer is (D). Carter explains were used during the cold winter months.
how he believes that if everyone cooperates The energy sources were not weakened,
and contributes to conservation, the problem choice (C), or wasted, choice (D). They
will get solved more easily and quickly. Al- might be replaced by renewables, but he is
though he mentions all of the other choices not discussing those sources in this part of
as actions that can be taken, his main thrust the speech, so choice (B) is incorrect.
is to appeal to people acting together as a
22. The correct answer is (C). In both of these
nation to be more energy efficient.
sentences, the word reserves refers to some-
17. The correct answer is (B). By explaining thing that is saved for future use. In the first
various sources of waste, Carter shows people use (“We will also stress development of
that it won’t be all that difficult to conserve our rich coal reserves in an environmentally
energy; simple, small changes can make a sound way…”), the reserves are lands that
difference if enough people participate. The contain coal that has not yet been mined;
other choices are all ideas that he mentions, they are reserved for future use. In the second
but they are not statements that support his use (“Oil and natural gas companies must be
argument that people can make a difference honest with all of us about their reserves and
by conserving the energy they now waste. profits.”), the reserves are oil and gas sup-
18. The correct answer is (A). Because Carter is plies that are set aside in order to raise prices.
addressing the public, he makes suggestions 23. The correct answer is (B). The attention to
that people can easily do and tries to encour- detail is designed to show how ridiculous
age conservation. He speaks directly and Golightly looked. Contrasted with his van-
plainly to the people in an effort to get them ity, the picture Kipling paints is mocking.
on board. The fact that he is on television, The adjectives, for example, “clammy
choice (B), was irrelevant. The speech is not pulp” (line 60) and “evil-smelling dough”
authoritarian, choice (C)—it is the opposite: (line 62) go beyond a simple description
an effort to enlist the citizenry in solving a of someone caught in the rain. There is no

www.petersons.com
910 PART VII: Five Practice Tests

irony shown nor disgust, so choices (A) and 29. The correct answer is (D). Choice (D) shows
(C) are incorrect. The descriptions show a tongue-in-cheek response by the narrator
the character to be helpless in the face of a and is mocking in its tone: “When he took
normal event—a rainstorm. out his handkerchief to wipe his face and
24. The correct answer is (A). In the passage, the green of the hat-lining and the purple
Kipling states that Golightly’s focus on his stuff that had soaked through on to his neck
appearance led to him not taking enough from the tie became thoroughly mixed, the
money with him. This results in him negotiat- effect was amazing.” The other choices are
ing with the Station-Master for a first-class all descriptions of the environment and Go-
ticket and subsequent arrest. His pride, lack lightly’s responses to events as they unfold.
of an umbrella, and fall from his horse may 30. The correct answer is (B). Among the de-
have been contributing factors to his arrest, tails that illustrate that Golightly was a man
but none is the best answer: It was not pride of privilege is that he had servants who were
alone that led to the arrest; even with an meeting him so he could change his clothes.
umbrella, Golightly would still have had no Golightly’s attire was a result of his vanity, not
money for a ticket; and Golightly was not his wealth (choice A). The title Sahib indicates
responsible for the fall, as it was an accident. respect, not wealth, so choice (D) is incorrect.
25. The correct answer is (A). Golightly is 31. The correct answer is (B). This sentence
described as an officer. We are told he was from the passage is an idiomatic way of say-
taking a leave of his post, which would ing that Golightly was pushing the envelope
indicate a military post. There is no indica- to the greatest extent possible without ruf-
tion that he was an elected official or that fling feathers in the line of command. His
he worked in a company. capture by the authorities was a result of
26. The correct answer is (C). Kipling piles misidentification because he was so unrec-
on the missteps Golightly makes, each one ognizable from being out in the rainstorm
adding to the other, making Golightly look and falling off the horse. He was doing what
silly and inept. For a person whose title is was expected, so there was nothing to get
“officer,” Golightly does not show profes- caught doing, choice (D), except that he
sionalism or skill at handling a relatively looked suspicious to others in his disheveled
minor problem. The rainstorm, choice (B), is state. Choice (A) is contrary to the meaning
a backdrop for the setting in which he falters of the text; he had told his superiors. The
and is not in itself a way to make fun of the sentence is about how much leave he could
character. There is no satire shown, so choice take, not about leaving without permission.
(A) is incorrect. Golightly’s trouble with the 32. The correct answer is (B). The word mired
horse, choice (D), went beyond “difficulty.” means to become covered with dirt or muck.
27. The correct answer is (A). It is the detail In the sentence, the word mud is a clue that
of the descriptions that give the story its the word is not just related to the road con-
mocking tone and attitude. Kipling sets up dition, but to the effect of the conditions.
the character by describing his intense van- Although the sentence describes the pony,
ity, which contrasts with the way he looks choice (A), pony does not help establish the
when caught in the rainstorm. The details meaning of mired.
are not relevant to any of the other choices. 33. The correct answer is (C). The content of
28. The correct answer is (B). Throughout the the speech shows Stanton’s concern about
excerpt, Kipling uses an omniscient narrator how women were treated unfairly, with few
to tell the story. But in this line, the narrator rights. Although some people at the time
editorializes and injects his own opinion. In- may have been shocked by her speech, her
stead of describing the character’s response, purpose was not shock value, choice (B),
the narrator judges it and then continues to but to inform and persuade. Although she
describe Golightly’s behavior. The narra- was sympathetic to the plight of minorities/
tor does not report the character’s verbal African Americans, this speech was focused
response or add dialogue. on women’s issues, so choice (D) is incorrect.

Master the New SAT®


Practice Test 5 911

answers Practice Test 5


34. The correct answer is (B). Stanton repeats rule of law; for slaves, it was a given that
the phrase “He has” to list each of the trans- they had to obey their masters. In some cases,
gressions of men against women’s rights. women did get paid for their work, choice
The repetition emphasizes and magnifies (A), and women, whether or not they were
these transgressions and makes the point slaves, were generally in charge of children,
that they can’t be ignored. Stanton did feel whether their own or others’, choice (B).
strongly, choice (A), but the use of repetition While some men may break promises, only
does not indicate strength of feeling, it shows the Sojourner text discusses how her master
strength of her argument. Although Stanton continually broke his promises to her.
did use the Declaration of Independence as 39. The correct answer is (D). Stanton’s list
a model, choice (D), the repetition is not of grievances shows her frustration and re-
part of that model. sulting anger at the system that has denied
35. The correct answer is (A). The graphic women basic rights. Truth’s narrative is
is a cover from an old newspaper that was second-hand, but it expresses her feelings
published by a women’s group. The drawing of frustration at being promised her freedom
shows a policeman holding back the women only to have it denied. Truth doesn’t show
from voting while he points to the ballot box anger, choice (A); sadness, choice (B); or
for the inmates, who are shown by their ste- bitterness, choice (C); she shows resolve and
reotypical striped prison clothes. The message determination to get her freedom.
is clear: criminals can vote but woman can’t, 40. The correct answer is (D). In the lines
which is the sentiment reflected in choice “… at best, the slave is accused of not hav-
(A): “He has withheld from her rights which ing performed his part or condition of the
are given to the most ignorant and degraded contract.” Truth explains that the master’s
men—both natives and foreigners.” ploy is always to say that there is some piece
36. The correct answer is (D). In lines 30–35, of work that has not been completed, which
Stanton shows how women have been en- he claims voids their contract. Choice (A)
slaved by the legal system and by men, who describes what Truth thinks has become an
are considered superior. Some women did obstacle for her leaving—not a reason given
have jobs outside the home, as is implied by her master. Choice (C) shows Truth ex-
by lines 24–25, choice (A), but Stanton claiming that slaveholders in general don’t
explains that they were not allowed to keep keep their promises, but it does not show
their wages. Women were not allowed to why her master denied her freedom as he
vote, and Stanton includes this in her list of had promised. Although financial concerns
grievances, but voting rights is not directly may have played a role in why her master
related to obedience. Choice (C) is implied didn’t want to let her go, this was not what
by lines 38–41, but this issue is not directly Truth was told was the reason, so choice
related to obedience. (B) is incorrect.
37. The correct answer is (D). “Franchise” is 41. The correct answer is (C). Lines 60–62 tells
another word for the right to vote. By pairing us that he was willing to give her freedom
the word with elective, the focus is on the before he had to and the introduction tells us
right to vote in elections. Stanton emphasizes that she was granted her freedom in 1827,
how women are deprived of this basic right. before it was legally mandated. Forcing her
A more modern meaning of franchise is in to work longer than he had originally said
sports, indicating a team that is a member of was a form of mistreatment. He did not allow
a larger group or league. At the time of the her to learn to read and write, which was why
speech, there were no such sports leagues, her narrative is told through someone else’s
and the word was commonly used to refer voice, choice (B). Although he did make
to the right to vote. promises that he didn’t keep, choice (D),
38. The correct answer is (C). The common- Truth suggests that all slaveholders used this
ality is in the necessity for obedience. For tactic to get more work out of their slaves.
women, obedience to their husbands was the

www.petersons.com
912 PART VII: Five Practice Tests

42. The correct answer is (B). Although Truth did not know what its closest relatives were
had great religious faith, the faithfulness because they had not yet realized that it was
referred to here is not about religion, choices a different species, so choice (C) is incorrect.
(A) and (C); it is about loyalty. Truth remi- 48. The correct answer is (D). Looking at the
nisces here about her service to her master. drawing, the traits shown on the S. sullivani
He had asked that she be a faithful servant are as described in the text. The drawing is
(line xx), which she was, but then she rec- made to reflect what scientists think this
ognizes that, ironically, this is exactly why dinosaur looked like based on the fossil
she has become so valuable to her master. evidence and research. Choices (A) and
43. The correct answer is (D). The overall (C) are facts in the text that are unrelated to
main idea encompasses the discovery and its the drawing. Choice (B) describes its size,
meaning (a new species). Choices (A) and but the drawing displays only relative size.
(B) are details that support the main idea. 49. The correct answer is (B). Part of the scien-
Choice (C) is a general statement that could tific process is to test theories by comparing
be a conclusion, but it is not the main idea. evidence. Jasinski was comparing the fossils,
44. The correct answer is (C). The article which led to his discovery. The other choices
describes how the discovery was made by a indicate some of the data scientists had, but
student and other researchers. The discovery they do not illustrate the scientific process.
was important because it identified a new spe- 50. The correct answer is (B). The information
cies and will help scientists and researchers in the article was in the news because Jasin-
continue to add to our body of knowledge ski’s discovery identified a new species of
about prehistoric mammals. The discovery dinosaur, adding to our scientific knowledge
wasn’t totally by chance because Jasinski of evolution and paleontology. Choice (D)
was conducting related research, choice (D). is incorrect because the discovery itself was
Choices (A) and (B) are general statements not related to determining where the species
that could have general application, but they lived—that was determined after identifying
are not conclusions based on facts in the article. the species. The mention of the film Jurassic
45. The correct answer is (A). A predator kills Park helps readers envision the new spe-
and eats other animals. To find its prey, a cies, but the relationship of the discovery
strong sense of smell could be a useful in to the film is not the news, so choice (A) is
the hunt. Such a trait would probably not incorrect. Choice (C) is incorrect because
help detect danger. The text does not discuss the naming of the species is a detail that, in
eyesight or finding a mate. itself, is not newsworthy.
46. The correct answer is (D). The genus name 51. The correct answer is (C). “Formidable”
Saurornitholestes means “lizard bird thief,” means impressive. It can mean impressive
which indicates its appearance. None of in the sense of its size or power, and in this
the Latin names indicate either where the context the predator—the newly discovered
fossils were found, choice (B), or where the species—was one that would have been
dinosaurs lived, choice (C). The discovered threatening to other species. A predator
species was named after a human, as are might have been dreaded, but not dreadful,
others on occasion, but the name does not so choice (A) is incorrect.
reveal anything about the dinosaur. 52. The correct answer is (B). The two sen-
47. The correct answer is (B). The story ex- tences together connect the meaning by
plains the process of Jasinski’s discovery, indicating that the olfactory bulb is related
and he noticed that the species he was to the sense of smell. The organs that are
examining differed from the description of used for smell are contained in that part of
what was expected. In the comparison, he the skull, which was unusually large, but
realized that the fossil he had was actually the clue is in the next sentence that defines
a different species. He didn’t recognize this “olfactory.”
because of either the size of the fossil, choice
(A), or its markings, choice (D). Scientists

Master the New SAT®


Chapter 2: Diagnostic Test 35

diagnostic test
SECTION 1: READING TEST
65 Minutes • 52 Questions

TURN TO SECTION 1 OF YOUR ANSWER SHEET TO ANSWER THE QUESTIONS IN THIS


SECTION.

Directions: Each passage (or pair of passages) below is followed by a number of multiple-choice
questions. After reading each passage, select the best answer to each question based on what is
stated or implied in the passage or passages and in any supplementary material, such as a table,
graph, chart, or photograph.

QUESTIONS 1–10 ARE BASED ON THE betake themselves to the high grounds,
FOLLOWING PASSAGE. followed by their cubs. In such retired
parts of the country where there are no
John James Audubon (1785–1851) is known hilly grounds, it pays visits to the maize
primarily for his bird studies, but as this passage 35 fields, which it ravages for a while. After
from Ornithological Biography shows, he wrote this, the various species of nuts, acorns,
grapes, and other forest fruits, that form
about the behavior of other animals as well.
what in the western country is called
Black Bear mast, attract its attention. The Bear is then
The Black Bear (Ursus americanus), 40 seen rambling singly through the woods
however clumsy in appearance, is active, to gather this harvest, not forgetting to
vigilant, and persevering; possesses rob every Bee tree it meets with, Bears
Line great strength, courage, and address; and being, as you well know, expert at this
5 undergoes with little injury the greatest operation. You also know that they are
fatigues and hardships in avoiding the 45 good climbers, and may have been told,
pursuit of the hunter. Like the Deer, it or at least may now be told, that the
changes its haunts with the seasons, and Black Bear now and then houses itself in
for the same reason, namely, the desire of the hollow trunks of the larger trees for
10 obtaining suitable food, or of retiring to weeks together, when it is said to suck
the more inaccessible parts, where it can 50 its paws. You are probably not aware of
pass the time in security, unobserved by a habit in which it indulges, and which,
man, the most dangerous of its enemies. being curious, must be interesting to
During the spring months, it searches for you. At one season, the Black Bear may
15 food in the low rich alluvial lands that be seen examining the lower part of
border the rivers, or by the margins of 55 the trunk of a tree for several minutes
such inland lakes as, on account of their with much attention, at the same time
small size, are called by us ponds. There looking around, and snuffing the air, to
it procures abundance of succulent roots, assure itself that no enemy is near. It then
20 and of the tender juicy stems of plants, raises itself on its hind legs, approaches
on which it chiefly feeds at that season. 60 the trunk, embraces it with its forelegs,
During the summer heat, it enters the and scratches the bark with its teeth and
gloomy swamps, passes much of its time claws for several minutes in continuance.
wallowing in the mud, like a hog, and Its jaws clash against each other, until a
25 contents itself with crayfish, roots, and mass of foam runs down both sides of the
nettles, now and then, when hard pressed 65 mouth. After this it continues its rambles.
by hunger, seizing on a young pig, or In various portions of our country, many
perhaps a sow, or even a calf. As soon as of our woodsmen and hunters who have
the different kinds of berries which grow seen the Bear performing the singular
30 on the mountain begin to ripen, the Bears operation just described, imagine that it
GO TO THE
NEXT PAGE
www.petersons.com
36 PART II: Diagnosing Strengths and Weaknesses

70 does so for the purpose of leaving behind 4 Huntsmen and woodsmen claim that the
an indication of its size and power. They bear scratches tree bark with its teeth and
measure the height at which the scratches claws to
are made, and in this manner, can, in fact,
form an estimate of the magnitude of the (A) sharpen its teeth.
75 individual. My own opinion, however, (B) mark the tree for winter hibernation.
is different. It seems to me that the Bear
(C) ward off potential predators by
scratches on the trees, not for the purpose
showing its size.
of showing its size or its strength, but
merely for that of sharpening its teeth (D) mark the tree so that other animals
80 and claws, to enable it better to encounter can’t harvest its nuts and acorns.
a rival of its own species during the
amatory season. The Wild Boar of 5 What is the main rhetorical effect of lines
Europe clashes its tusks and scrapes the 22–28?
earth with its feet, and the Deer rubs its
85 antlers against the lower part of the stems (A) To show that the bear is an
of young trees or bushes, for the same exceptional predator
purpose. (B) To explain why humans might want
to hunt bears
1 As used in line 4, “address” refers to (C) To impress the reader with how
varied a bear’s diet is
(A) habitat.
(D) To create an image of a bear placidly
(B) focus. foraging for food
(C) skill.
(D) direction. 6 What evidence in the text does Audubon
use to support his theory about why bears
2 What is the most likely reason that Audu- mark a tree with their teeth and claws?
bon wrote about the black bear?
(A) When foaming at the mouth, the bear
(A) He wanted to provide more scares off predators.
information about another animal to (B) The marks indicate to predators the
his readers. huge size and power of the bear.
(B) He was fascinated by mammals. (C) Other animals have similar behaviors
(C) He didn’t think his readers knew designed to strengthen the animal.
anything about bears. (D) The practice makes the bear foam at
(D) He wanted to show the the mouth, which is a sign of strength
commonalities in behavioral patterns to other bears.
of bears and birds.
7 According to Audubon, how are the claws of
3 Which of the following quotes from the the black bear like the tusk of the wild boar?
text provides evidence to support the idea
(A) Both are parts of the body that serve
that the bear migrates from one habitat to
to get food for the animal.
another?
(B) Both animals use these parts of their
(A) Lines 7–8 (“Like … seasons”) bodies as a sign of strength.
(B) Line 19 (“it procures … roots”) (C) Both animals use these body parts
(C) Lines 53–55 (“the Black Bear … to defend themselves from human
tree”) predators.
(D) Line 65 (“After … rambles”) (D) Both are parts of the body that the
animal sharpens to better compete for
a mate.
Master the New SAT®
Chapter 2: Diagnostic Test 37

diagnostic test
8 The fact that Audubon calls man the and good will: I am sure you will find this
bear’s “most dangerous” enemy (line 13) to have been one of the more fortunate
indicates that he events of your life, as I have ever been
sensible it was of mine. I enclose you a
(A) is a hunter himself. 10 sketch of the sciences to which I would
(B) has some sympathy for hunted bears. wish you to apply in such order as Mr.
Wythe shall advise: I mention also the
(C) does not believe that bears are
books in them worth your reading, which
dangerous.
submit to his correction. Many of these
(D) thinks bears are more dangerous than 15 are among your father’s books, which you
people. should have brought to you. As I do not
recollect those of them not in his library,
9 What evidence in the text shows that black you must write to me for them, making
bears are not vegetarians? out a catalogue of such as you think you
20 shall have occasion for in 18 months from
(A) Lines 18–21 (“There it … that season”) the date of your letter, and consulting Mr.
(B) Lines 26–28 (“when hard … even a Wythe on the subject. To this sketch I will
calf.”) add a few particular observations.
1. Italian. I fear the learning of this
(C) Lines 34–35 (“it pays … for a while”)
25 language will confound your French and
(D) Lines 36–39 (“various species … its Spanish. Being all of them degenerated
attention”) dialects of the Latin, they are apt to
mix in conversation. I have never seen
10 How can you tell what assumptions Audu- a person speaking the three languages
bon makes about his reader? 30 who did not mix them. It is a delightful
language, but late events having rendered
(A) He gives detailed descriptions, the Spanish more useful, lay it aside to
suggesting that the reader is prosecute that.
interested. 2. Spanish. Bestow great attention on
(B) He addresses the reader directly to 35 this, and endeavor to acquire an accurate
provide information he thinks the knowledge of it. Our future connections
reader doesn’t have. with Spain and Spanish America
will render that language a valuable
(C) Using comparisons to the deer shows
acquisition. The ancient history of a great
that Audubon assumes readers are
40 part of America too is written in that
more familiar with deer than bears.
language. I send you a dictionary.
(D) Using third-person narration and 3. Moral philosophy. I think it lost
passive voice indicates that Audubon time to attend lectures in this branch. He
assumes all of the information is new who made us would have been a pitiful
to the reader. 45 bungler if he had made the rules of our
moral conduct a matter of science. For
QUESTIONS 11–21 ARE BASED ON THE one man of science, there are thousands
FOLLOWING PASSAGE. who are not. What would have become of
them? Man was destined for society. His
Thomas Jefferson wrote in 1787 to his nephew, 50 morality therefore was to be formed to
Peter Carr, a student at the College of William this object. He was endowed with a sense
and Mary. of right and wrong merely relative to this.
Paris, August 10, 1787 This sense is as much a part of his nature
Dear Peter, I have received your two as the sense of hearing, seeing, feeling;
letters of December 30 and April 18 and 55 it is the true foundation of morality. …
am very happy to find by them, as well as The moral sense, or conscience, is as
Line by letters from Mr. Wythe,* that you have
much a part of man as his leg or arm. It
5 been so fortunate as to attract his notice
GO TO THE
NEXT PAGE
www.petersons.com
38 PART II: Diagnosing Strengths and Weaknesses

is given to all human beings in a stronger 15 In lines 56–57, Jefferson compares con-
or weaker degree, as force of members is science to a physical limb of the body to
60 given them in a greater or less degree. … show
State a moral case to a ploughman and
a professor. The former will decide it (A) that it is natural and present in all
as well, and often better than the latter, human beings.
because he has not been led astray by (B) how easily we take it for granted.
65 artificial rules.
(C) that without it, humans are powerless.
*George Wythe, a well-respected scholar, the first
American law professor, and one of the signatories
(D) how mental and physical states are
of the Declaration of Independence, became an integrated.
important teacher and mentor to Thomas Jefferson.

16 What country does Jefferson think will


11 What is the best description of Mr. Wythe most closely aligned with the newly inde-
and his relationship to the Jefferson family? pendent colonies in the future?

(A) Teacher (A) England


(B) Cousin B) France
(C) Family friend C) Italy
(D) Public servant D) Spain

12 What is the purpose of Jefferson’s letter to 17 By “lost time” (lines 42–43), Jefferson
his nephew? means

(A) To advise him about his education (A) wasted time.


(B) To advise him about leading a moral (B) the past.
life (C) missing time.
(C) To make sure he will learn a second (D) youth.
language
(D) To keep in touch with his family 18 Jefferson tells his nephew not to study
while abroad
Italian because it’s

13 What does Jefferson mean by “lay it aside (A) a degenerated dialect.


to prosecute that” (lines 32–33)? (B) not necessary since he already knows
French.
(A) Support Spain as a world power
(C) not useful to be multilingual.
(B) Bring legal action against Italy
(D) too easy to get it mixed up with
(C) Pursue studies of Spanish Spanish.
(D) Engage in the study of Italian
19 Which of the following best summarizes
14 Which of the following is the best example Jefferson’s overall view of morality?
of the paternal tone Jefferson uses with his
nephew? (A) Morality is a science that can be
taught by professors and scholars.
(A) Lines 6–8 (“I am sure … your life”) (B) Moral philosophy is self-taught.
(B) Lines 22–23 (“I will add … (C) A sense of morality is part of human
observations.”) nature.
(C) Line 41 (“I send … a dictionary.”) (D) Humans are moral beings who need
(D) Line 49 (“Man was … society.”) rules to guide their behavior.

Master the New SAT®


Chapter 2: Diagnostic Test 39

diagnostic test
Three Generations of the Jefferson Family

Peter Jefferson (b: 1708; d: 1757) – [spouse] Jane Randolph (b: 1721; d: 1776)

[children: 10]

Thomas (b: 1743; d: 1826) – [spouse] Martha Wales (b: 1748; d: 1782) Martha (b: 1746; d: 1811) – [spouse] Dabney Carr [b: 1743; d: 1773]

[children: 6] [children: 6]

(others died before reaching adulthood)

Martha (b: 1772; d: 1836) Mary (b: 1778; d: 1782) | Peter (b: 1770; d: 1815)

20 How does the family tree best explain Jef- garbage) at sea. However, an unknown
ferson’s concern about Peter’s education? portion of the plastic produced each
year escapes into the environment—
(A) Jefferson was Peter’s uncle. instead of being landfilled, incinerated,
(B) Jefferson didn’t have any sons. 15 or recycled—and at least some of it
eventually makes its way to sea.
(C) Peter’s father died when Peter was 3.
Plastics that reach the ocean will
(D) Peter was the smartest student in the gradually break down into ever-smaller
family. pieces due to sunlight exposure,
20 oxidation, and the physical action of
21 Which of the following best describes the waves, currents, and grazing by fish and
tone of Jefferson’s letter to his nephew? birds. So-called microplastics—variably
defined in the scientific literature and
(A) Invested and paternal popular press as smaller than 1 or 5 mm
(B) Concerned and worried 25 in diameter—are understood to be the
most abundant type of plastic in the
(C) Objective and matter-of-fact
ocean. The 5 Gyres’ authors* found
(D) Distant and preoccupied microplastics almost everywhere they
sampled, from near-shore environments to
QUESTIONS 22–32 ARE BASED ON THE 30 the open ocean, in varying concentrations,
FOLLOWING EXCERPT. and they estimated that particles 4.75 mm
or smaller—about the size of a lentil—
This excerpt is from the article “New Link in made up roughly 90% of the total plastic
the Food Chain? Marine Plastic Pollution and pieces they collected.
Seafood Safety,” by Nate Seltenrich. It has been 35 But the degradation of larger pieces of
reproduced from the journal Environmental plastic is not the only way microplastics
Health Perspectives. end up in the ocean. Nurdles—the plastic
pellets used as a feedstock for producing
World plastics production has plastic goods—can spill from ships or
experienced almost constant growth 40 land-based sources, and “microbeads”
for more than half a century, rising used as scrubbing agents in personal
Line from approximately 1.9 tons in 1950 to care products such as skin cleansers,
5 approximately 330 million tons in 2013. toothpastes, and shampoos, can escape
The World Bank estimates that 1.4 billion water-treatment facilities and pass into
tons of trash are generated globally each 45 water-sheds with treated water. (In June
year, 10% of it plastic. The International 2014, Illinois became the first US state to
Maritime Organization has banned the ban the manufacture and sale of products
10 dumping of plastic waste (and most other containing microbeads, which have been
GO TO THE
NEXT PAGE
www.petersons.com
40 PART II: Diagnosing Strengths and Weaknesses

documented in the Great Lakes and plastics from the start. “We need to design
50 Chicago’s North Shore Channel.) the next generation of plastics to make
Marine organisms throughout the food them more biodegradable so that they
chain commonly consume plastics of don’t have a long half-life, they don’t
various sizes. The tiniest microplastics 105 accumulate in the oceans, and they don’t
are small enough to be mistaken for food have the opportunity to collect chemicals
55 by zooplankton, allowing them to enter long-term,” he says. “There’s just no way
the food chain at very low trophic levels. we can shield people from all exposures
Some larger predators are thought to that could occur. Let’s design safer
confuse nurdles (which typically measure 110 chemicals and make the whole problem
less than 5 mm in diameter) with fish moot.”
60 eggs or other food sources. *The 5 Gyres Institute addresses plastic pollution in
Once plastics have been consumed, the ocean.
laboratory tests show that chemical
additives and adsorbed pollutants and 22 Which of the following is the most com-
metals on their surface can desorb (leach
mon type of plastic found in the ocean?
65 out) and transfer into the guts and tissues
of marine organisms …. (A) Nurdles
Research has shown that harmful
(B) Microplastics
and persistent substances can both
bioaccumulate (or increase in (C) Microbeads
70 concentration as exposures persist) and (D) Plastic pellets
biomagnify (or increase in concentration
at higher trophic levels) within organisms 23 What evidence from the text helps ex-
as they assume some of the chemical
plain why scientists think it is important
burden of their prey or environment. Yet
to include the study of seafood in their
75 again, no research has yet demonstrated
investigations?
the bioaccumulation of sorbed pollutants
in the environment. (A) Lines 17–22 (“Plastics that … fish
Three key questions remain to be and birds.”)
determined. To what extent do plastics
(B) Lines 51–53 (“Marine organisms …
80 transfer pollutants and additives to
various sizes.”)
organisms upon ingestion? What
contribution are plastics making to the (C) Lines 62–66 (“chemical additives …
contaminant burden in organisms above marine organisms”)
and beyond their exposures through (D) Lines 86–88 (“what proportion …
85 water, sediments, and food? And, finally, through seafood?”)
what proportion of humans’ exposure to
plastic ingredients and environmental 24 Which best describes the overall tone of
pollutants occurs through seafood?
the article?
Researchers are moving carefully in the
90 direction of answers to these questions. … (A) Neutral and scientific
New laws … could require handling
(B) Emotional and persuasive
plastics more responsibly at the end
of their useful life through recycling, (C) Personal and human
proper disposal, and extended producer (D) Subjective and opinionated
95 responsibility.
Rolf Halden, director of the Center
for Environmental Security at the
Biodesign Institute at Arizona State
University, advocates for another
100 solution: manufacturing more sustainable

Master the New SAT®


Chapter 2: Diagnostic Test 41

diagnostic test
25 What solution does Rolf Halden support 26 This article was written to
to decrease the effects of pollution from
plastics on humans? (A) share the author’s opinion about the
environment.
(A) Passing laws to mandate more (B) inform the public of the problems of
rigorous recycling plastic in the ocean.
(B) Developing plastics that are (C) start a movement to halt all plastic
biodegradable production.
(C) Making plastics that are safe to ingest (D) get people to clean up the oceans.
(D) Requiring plastic manufacturers to
be responsible for the effects of their 27 What does Halden mean when he says he
products
wants to “make the whole problem moot”
(lines 110–111)?

(A) He wants to open it to more debate.


(B) He wants to make it go away.
(C) He wants it to be studied further.
(D) He wants to reduce its significance.

GO TO THE
NEXT PAGE
www.petersons.com
42 PART II: Diagnosing Strengths and Weaknesses

28 Which of the following statements could 31 What evidence does author use to support
be learned from the diagram about how the idea that plastics in the ocean are a
plastics disrupt the food chain? problem for humans?

(A) Plastics enter the food chain when (A) Ten percent of the world’s trash are
small fish eat small pieces of plastic. plastic.
(B) Sunlight breaks down larger pieces (B) Plastics transfer pollutants and
of plastic that have been tossed in the additives to organisms upon
ocean. ingestion.
(C) Unfiltered wastewater is often (C) Plastics were found in samples from
emptied directly into the ocean, even all parts of the ocean.
though it is illegal in many places. (D) Experts say we need to design
(D) People should only eat a limited a new kind of plastics that are
amount of fish and seafood because biodegradable.
they may contain unsafe levels of
contaminants. 32 The main disagreement between the author
and Rolf Halden about a solution to the
29 Why did Illinois ban the sale of certain problem of plastic pollution is that the
personal care products? author

(A) Residues from the products were (A) advocates new laws and Halden
ending up in the ocean. advocates new ways to manufacture
(B) The containers couldn’t be recycled. plastics.
(C) The products were determined to be (B) thinks recycling will resolve the
carcinogenic. problem, but Halden doesn’t think we
can ever be safe from exposure.
(D) The products contained microbeads
that were getting into the water (C) thinks the plastic manufacturers
system. should be more responsible for
their products, but Halden thinks
this would be an undue burden on
30 Which of the following words would be
business.
most helpful in figuring out the meaning
(D) thinks all plastics manufacturing
of the word “adsorbed” (line 63)?
should be stopped and Halden thinks
(A) Absolved research will find a solution.
(B) Adhered
(C) Absorbent
(D) Sorbet

Master the New SAT®


Chapter 2: Diagnostic Test 43

diagnostic test
QUESTIONS 33–42 ARE BASED ON THE It was a hard struggle for so small a
FOLLOWING PASSAGE. child.
I went on and on; the reeds were
Angel Decora was born Hinookmahiwikilinaka 45 waving their tasselled ends in the wind.
on the Winnebago Reservation in Nebraska in I stopped and looked at them. A reed,
1871. She worked as a book illustrator, particu- whirling in the wind, had formed a space
larly on books by and about Native Americans, round its stem, making a loose socket. I
and lectured and wrote about Indian art. The stood looking into the opening. The reed
story from which this excerpt is taken, “The Sick 50 must be rooted in the ground, and the
Child,” may be autobiographical. hole must follow the stem to the earth. If I
poured my offerings into the hole, surely
It was about sunset when I, a little they must reach the ground; so I said the
child, was sent with a handful of prayer I had been taught, and dropped my
powdered tobacco leaves and red feathers 55 tobacco and red feathers into the opening
Line to make an offering to the spirit who had that nature itself had created.
5 caused the sickness of my little sister. No sooner was the sacrifice
It had been a long, hard winter, and the accomplished than a feeling of doubt
snow lay deep on the prairie as far as the and fear thrilled me. What if my offering
eye could reach. The medicine-woman’s 60 should never reach the earth? Would my
directions had been that the offering must little sister die?
10 be laid upon the naked earth, and that to Not till I turned homeward did I realize
find it I must face toward the setting sun. how cold I was. When at last I reached
I was taught the prayer: “Spirit the house they took me in and warmed
grandfather, I offer this to thee. I pray 65 me, but did not question me, and I said
thee restore my little sister to health.” nothing. Everyone was sad, for the little
15 Full of reverence and a strong faith that one had grown worse.
I could appease the anger of the spirit, I The next day the medicine woman
started out to plead for the life of our little said my little sister was beyond hope; she
one. 70 could not live. Then bitter remorse was
But now where was a spot of earth mine, for I thought I had been unfaithful,
20 to be found in all that white monotony? and therefore my little sister was to be
They had talked of death at the house. I called to the spirit-land. I was a silent
hoped that my little sister would live, but child, and did not utter my feelings; my
I was afraid of nature. 75 remorse was intense.
I reached a little spring. I looked down My parents would not listen to what
25 to its pebbly bottom, wondering whether the medicine-woman had said, but clung
I should leave my offering there, or keep to hope. As soon as she had gone, they
on in search of a spot of earth. If I put my sent for a medicine-man who lived many
offering in the water, would it reach the 80 miles away.
bottom and touch the earth, or would it He arrived about dark. He was a
30 float away, as it had always done when I large man, with a sad, gentle face. His
made my offering to the water spirit? presence had always filled me with awe,
Once more I started on in my search of and that night it was especially so, for he
the bare ground. 85 was coming as a holy man. He entered
The surface was crusted in some the room where the baby lay, and took a
35 places, and walking was easy; in other seat, hardly noticing any one. There was
places I would wade through a foot or silence saving only for the tinkling of the
more of snow. Often I paused, thinking little tin ornaments on his medicine-bag.
to clear the snow away in some place and 90 He began to speak: “A soul has departed
there lay my offering. But no, my faith from this house, gone to the spirit-land.
40 must be in nature, and I must trust to it to As I came I saw luminous vapor above
lay bare the earth. the house. It ascended, it grew less, it was
GO TO THE
NEXT PAGE
www.petersons.com
44 PART II: Diagnosing Strengths and Weaknesses

gone on its way to the spirit-land. It was 37 Which title best expresses a theme of the
95 the spirit of the little child who is sick; passage?
she still breathes, but her spirit is beyond
our reach… (A) Life and Death on the Prairie
(B) A Child’s Sacred Memory
33 The narrator wants to place her offering (C) Native American Culture
correctly because she (D) The Offering
(A) will have to explain her choice to
everyone else. 38 When the girl says “bitter remorse was
(B) wants to be trusted with similar tasks mine” (lines 70–71), she
in the future.
(A) is sorry that she hurt her sister.
(C) thinks doing so will save her little
(B) feels badly because she didn’t listen
sister’s life.
to the medicine woman.
(D) is afraid of being punished if she
(C) feels angry about being given so
does it incorrectly.
much responsibility.
(D) feels guilty because she feels at fault.
34 How is the sentence “It was a hard struggle
for so small a child” (lines 42–43) different
39 Based on the passage, which choice best
from the rest of the passage?
describes the narrator’s relationship with
(A) It is a change in the voice. her parents?
(B) It is a change in tone.
(A) The parents seem to treat the narrator
(C) It is a change in tense. as if she were an adult.
(D) It is a change in the setting. (B) The narrator wishes her parents
would give her more responsibility.
35 Why didn’t the girl’s parents send for the (C) The parents love their youngest child,
medicine man in the first place? but not the narrator.
(D) The narrator receives warmth and
(A) He was busy helping another family
validation from her parents.
at the time.
(B) He had to come from a long distance.
40 If you were to describe this story to some-
(C) They thought the medicine woman
one who has not read it, which of the fol-
would be able to help their daughter.
lowing sentences would best summarize
(D) They preferred a woman to cure their it?
female child.
(A) A Native American recalls her
36 What evidence from the text shows the experience of losing her baby sister.
girl’s dilemma in following the medicine (B) A Native American child is called
woman’s directions? upon to make an offering to the
spirits.
(A) Lines 9–10 (“the offering … naked (C) A Native American family struggles
earth”) with illness in the depths of winter on
(B) Lines 19–20 (“But now … white the Plains.
monotony?”) (D) A Native American family uses their
(C) Lines 34–35 (“The surface … some religious beliefs to try to save their
places”) daughter.
(D) Lines 42–43 (“It was … small a
child.”)

Master the New SAT®


Chapter 2: Diagnostic Test 45

diagnostic test
41 Which of the following best describes the 42 What lines in the text might convince you
meaning of “thrilled” in line 59? that the passage is autobiographical?

(A) Excited (A) Line 21 (“They had … the house.”)


(B) Frightened (B) Lines 34–35 (“The surface … was
(C) Pierced easy”)
(D) Saddened (C) Lines 62–63 (“Not till … cold I was”)
(D) Lines 85–86 (“He entered … baby
lay”)

Directions: The two passages below deal with a related topic. Answer the questions based upon
what is stated or implied in the passages and in the additional material provided.

QUESTIONS 43–52 ARE BASED ON THE 25 With such an interpretation, a patriot


FOLLOWING PASSAGES. is a useful member of society capable
of enlarging all minds and bettering all
Passage 1: Fanny Wright was a reformer, author, hearts with which he comes in contact;
and orator; unusual occupations for a woman a useful member of the human family,
in the early nineteenth century. 30 capable of establishing fundamental
Passage 2: Young Robert Emmet was con- principles and of merging his own
demned to death for treason after organizing a interests, those of his associates, and
rebellion against the English in Ireland. He, too, those of his nation in the interests of
had achieved fame as an orator, with speeches the human race. Laurels and statues are
decrying tyranny. 35 vain things, and mischievous as they are
childish; but could we imagine them of
Passage 1—Fanny Wright to a Fourth- use, on such a patriot alone could they be
of-July Audience at New Harmony, Indiana with any reason bestowed. …
(1828)
In continental Europe, of late years, Passage 2—Robert Emmet to the Court
the words patriotism and patriot have That Condemned Him to Death (1803)
been used in a more enlarged sense than I am charged with being an emissary
40 of France. An emissary of France! and for
Line it is usual here to attribute to them, or
5 than is attached to them in Great Britain. what end? It is alleged that I wish to sell
Since the political struggles of France, the independence of my country; and for
Italy, Spain, and Greece, the word what end? …
patriotism has been employed, throughout No; I am no emissary. … Sell my
45 country’s independence to France! and
continental Europe, to express a love
10 of the public good; a preference for the for what? Was it a change of masters?
interests of the many to those of the No, but for ambition. Oh, my country!
few; a desire for the emancipation of the Was it personal ambition that could
human race from the thrall of despotism, influence me? Had it been the soul of my
50 actions, could I not, by my education and
religious and civil: in short, patriotism
15 there is used rather to express the interest fortune, by the rank and consideration of
felt in the human race in general than that my family, have placed myself amongst
felt for any country, or inhabitants of a the proudest of your oppressors? My
country, in particular. And patriot, in like country was my idol! To it I sacrificed
55 every selfish, every endearing sentiment;
manner, is employed to signify a lover of
20 human liberty and human improvement and for it I now offer up myself, O
rather than a mere lover of the country in God! No, my lords; I acted as an
which he lives, or the tribe to which he Irishman, determined on delivering my
belongs. Used in this sense, patriotism is country from the yoke of a foreign and
60 unrelenting tyranny, and the more galling
a virtue, and a patriot is a virtuous man.
GO TO THE
NEXT PAGE
www.petersons.com
46 PART II: Diagnosing Strengths and Weaknesses

yoke of a domestic faction, which is its 43 Which of the following statements from
joint partner. … It was the wish of my Emmet’s speech shows that he thinks he
heart to extricate my country from this is a martyr?
double riveted despotism—I wished to
65 place her independence beyond the reach (A) Lines 50–53 (“could I not … your
of any power on earth. I wished to exalt oppressors”)
her to that proud station in the world. (B) Lines 64–66 (“I wished to … on
Connection with France was, indeed, earth”)
intended, but only as far as mutual
(C) Lines 75–77 (“I wished to … to be
70 interest would sanction or require.
assisted”)
Were the French to assume any
authority inconsistent with the purest (D) Lines 86–87 (“Let no man … with
independence, it would be the signal for dishonor”)
their destruction ….
75 I wished to prove to France and to 44 Which of the following of Emmet’s state-
the world that Irishmen deserved to be ments shows that he thinks he is a patriot?
assisted … I wished to procure for my
country the guarantee which Washington (A) Line 44 (“No; …. emissary”)
procured for America—to procure an (B) Lines 48–49 (“Was it … influence
80 aid which … would perceive the good, me?”)
and polish the rough points of our
(C) Lines 75–76 (“I wished … the world”)
character. …hese were my objects; not to
receive new taskmasters, but to expel old (D) Lines 91–93 (“The proclamation …
tyrants. And it was for these ends I sought for our views”)
85 aid from France ….
Let no man dare, when I am dead, Timeline
to charge me with dishonor; let no 1707 Acts of Union between
man attaint my memory by believing Scotland and England create the
that I could have engaged in any cause Kingdom of Great Britain
90 but that of my country’s liberty and 1776–1783 American colonies declare and
independence …. The proclamation of the win independence
provisional government speaks for our 1789 French storm the Bastille (prison),
views; no inference can be tortured from fight to end French monarchy
it to countenance barbarity or debasement
1798 Society of United Irishmen
‘95 at home, or subjection, humiliation, or
treachery from abroad. I would not have rebel unsuccessfully against
submitted to a foreign oppressor, for British rule
the same reason that I would resist the 1800 British Parliament passes The
foreign and domestic oppressor. In the Act of Union, abolishing the
100 dignity of freedom, I would have fought Irish parliament
upon the threshold of my country, and 1801 United Kingdom of Great
its enemy should enter only by passing Britain and Ireland created
over my lifeless corpse. And am I, who 1803 United States purchases
lived but for my country, and who have Louisiana Territory from France
105 subjected myself to the dangers of the
jealous and watchful oppressor, and the Robert Emmet leads a rebellion
bondage of the grave, only to give my in Dublin against the union
countrymen their rights, and my country 1803–1815 Napoleonic Wars in Europe
its independence. (France vs. European powers)
1808–1833 Spanish wars of independence
1823 France invades Spain to help
restore monarchy
1828 Andrew Jackson elected
Master the New SAT® president of United States
Chapter 2: Diagnostic Test 47

diagnostic test
45 What events may have led Wright to her 48 Which one of the following statements is
conclusion that Europeans have a different true?
view of patriotism? Use the timeline to
determine your answer. (A) Although years apart, both Wright
and Emmet were advocating for
(A) The French storming of the Bastille freedom for their countries.
(B) The establishment of the United (B) Emmet was focused on freedom
Kingdom and independence for his own
(C) The election of Andrew Jackson as country, while Wright was focused
president on freedom and independence for all
humankind.
(D) The Napoleonic Wars changing
European government (C) Emmet loved his country more than
Wright loved her country.
46 Based on information shown in the time- (D) Wright didn’t understand tyranny
because she lived in a democracy, but
line, why might France have turned down
Emmet was fighting for the kind of
Emmet’s request for help?
independence the United States won
(A) France was in the midst of trying to from England.
restore the monarchy in Spain.
(B) France was engaged in the drawn-out 49 Both passages are excerpts from speeches.
Napoleonic Wars. How does the tone of Wright’s speech
(C) France had fought its own revolution compare with that of Emmet’s speech?
and didn’t want to get involved in
(A) Both express anger, although in
that of another country.
response to different causes.
(D) France was trying to keep the United
(B) Wright offers her audience a reasoned
States from taking Louisiana.
defense of her ideas while Emmet
expresses anger and passion.
47 Emmet would not fit Wright’s definition
(C) Emmet speaks calmly and rationally,
of a patriot because he and Wright speaks passionately.
(A) saw no dishonor in his actions. (D) Both provide a reasoned defense of
their positions.
(B) wanted to free his people.
(C) idolized his own country above all
50 Which of the following statements is most
others.
analogous to Emmet’s statement: “In the
(D) declared the court’s sentence to be
dignity of freedom, I would have fought
unjust.
upon the threshold of my country, and its
enemy should enter only by passing over
my lifeless corpse.” (lines 99–103)

(A) Never yield to force; never yield to


the apparently overwhelming might
of the enemy.
(B) With the enemy at their back, with
our bayonets at their breasts, in the
day of their distress, perhaps the
Americans would have submitted…
(C) Give me liberty or give me death.
(D) It is sweet and fitting to die for one’s
country.
GO TO THE
NEXT PAGE
www.petersons.com
48 PART II: Diagnosing Strengths and Weaknesses

51 Which of the following fits Wright’s defi- 52 When the British government claims
nition of a patriot? that Emmet is an “emissary” (line 44) of
France, they are accusing him of being
(A) A person willing to die for their
country (A) an ambassador.
(B) A person who fights for improving (B) a spy.
the lives of others (C) a minister.
(C) A person who enlists in the armed (D) a mercenary.
forces of his country
(D) A person who loves his country

STOP
If you finish before time is called, you may check your work on this section only.
Do not turn to any other section.

Master the New SAT®


Chapter 2: Diagnostic Test 83

ANSWER KEY AND EXPLANATIONS

answers Diagnostic Test


Section 1: Reading

1. C 12. A 23. D 33. C 43. D


2. A 13. C 24. A 34. A 44. B
3. A 14. A 25. B 35. C 45. D
4. C 15. A 26. B 36. B 46. C
5. D 16. D 27. D 37. B 47. B
6. C 17. A 28. A 38. D 48. B
7. D 18. D 29. D 39. A 49. B
8. B 19. C 30. B 40. A 50. C
9. B 20. C 31. C 41. C 51. B
10. B 21. A 32. A 42. C 52. B
11. A 22. B

READING TEST RAW SCORE


(Number of correct answers)

1. The correct answer is (C). The context food harvests and places where it is safe. All
here tells us that address is not being used animals seek food, so the bear finding places
to mean direction, choice (D), or habitat, that have lots of roots does not necessarily
choice (A). Address appears in the author’s show migratory patterns, choice (B). The
description of the black bear, including bear examines the tree trunk to mark terri-
qualities that the bear possesses. Since the tory, choice (C); it is not part of a migration
author later describes the bear as rambling pattern. It continues to wander about, choice
and wandering, address is not being used (D), with no direction that indicates a new
to mean focus, choice (B). Therefore, the season or a new habitat. Therefore, choice
correct answer is choice (C). (A) is correct.
2. The correct answer is (A). This excerpt is all 4. The correct answer is (C). The hunters and
about the black bear and does not make any woodsmen believe this behavior is designed
direct comparisons to birds, choice (D). It to scare off predators who would, from the
only mentions deer and boars in comparison markings on the tree, be able to estimate
to bears, so it does not indicate a particular the size of the bear, choice (C). Choice (A)
fascination with mammals, choice (B). The represents Audubon’s claim, not that of the
shift to second person (lines 43–53) shows huntsmen and woodsmen. There’s no indica-
that the author assumes that the reader has tion that the tree will be used for hibernation,
some knowledge about bears, which makes choice (B), nor that such behavior is a way
choice (C) incorrect. Perhaps his readers did of obtaining food, choice (D).
not know enough, so he provides further
5. The correct answer is (D). The correct
information. Therefore, choice (A) is correct.
answer is (D). Lines 22–28 include the fact
3. The correct answer is (A). According to that bears may eat livestock, but that is not
this text and the first paragraph, the bear the main focus, so choices (A) and (B) are
changes location with the seasons, following incorrect. While the lines do show that the

www.petersons.com
84 PART II: Diagnosing Strengths and Weaknesses

bear eats a wide range of foods, choice (C), eating and ability to climb). Greater details
use of phrases such as “wallowing in the do not necessarily mean greater interest by
mud, like a hog,” and “it enters the gloomy a reader, so choice (A) is incorrect. When
swamp,” are evidence that the author is comparing the bear to the deer, Audubon
creating an image as opposed to making a provides details, so he doesn’t assume the
scientific list of what the bear eats. reader already knows how they are similar,
so choice (C) is incorrect. Audubon switches
6. The correct answer is (C). Audubon ex-
voice to inject a more personal tone, which is
plains other animal behaviors that are similar
not an assumption about the reader, making
and serve the same purpose, leading him to
choice (D) incorrect.
believe that the reason is to keep the bear
stronger than his competitors when the mat- 11. The correct answer if (A). The first para-
ing season starts, choice (C). Foam forming graph of the letter indicates that Wythe and
at the mouth is a result of the continuous force Jefferson have an established relationship
used and is a biological response. There is no and Jefferson believes Wythe had a positive
indication in the text that suggests that the influence on Jefferson’s education, thus we
foam has any purpose, so choices (A) and can infer that Wythe was a teacher. The let-
(D) are incorrect. Choice (B) is the position ter does not imply any familial relationship
taken by the woodsmen and huntsmen, not or that Wythe and Jefferson are friends—
Audubon. choices (B) and (C)—rather, it shows a more
formal connection. There is no indication of
7. The correct answer is (D). Audubon ex-
how Wythe is employed—choice (D). While
plains that the bear sharpens its claws and
he could be a public servant of some kind,
the wild boar sharpens its tusk for the same
that conclusion could not be drawn from
purpose: to prepare themselves for competi-
the information provided.
tion during the mating season, choice (D),
not for defense from human predators, 12. The correct answer is (A). Jefferson’s
choice (C). The bear uses its claws to get overall concern is that his nephew should
food, but Audubon doesn’t discuss the use receive a solid education, choice (A). This
of the wild boar’s tusks for getting food, so letter has a greater purpose than merely
choice (A) is incorrect. The argument made staying in touch, choice (D). His advice
by the woodsmen and hunters applies to the about studying languages, choice (C) is
bear, but it is not the argument supported by part of the overall advise about education,
Audubon, so choice (B) is incorrect. and morality is also a part of his course of
study, choice (B).
8. The correct answer is (B). The correct
answer is choice (B). This is the only pos- 13. The correct answer is (C). The word pros-
sible conclusion one can reach among those ecute in this context is used to mean pursue,
listed. Nothing indicates that Audubon choice (C), as opposed to its more common
hunts, choice (A), and he never denies that meaning of using the legal system to sue
bears are dangerous, choice (D). someone, choice (B). Jefferson is advocating
for the study of Spanish, not Italian, choice
9. The correct answer is (B). The only choice
(D), or support of the government of Spain,
that references the bear eating meat is choice
choice (A).
(B). All of the other options include only
plants, nuts, seeds, and other non-meat items. 14. The correct answer is (A). Choice (A)
indicates that Jefferson is thinking about
10. The correct answer is (B). Choice (B)
his nephew’s future and how Peter might
is correct because Audubon expresses his
evaluate the things that he did as a younger
assumptions about the reader’s knowledge
man. The other options show no particular
when he draws back from the third person
feelings; they are neutral, choices (B) and
and addresses the reader directly (lines
(C), or express his philosophy, choice (D).
43–44), remarking that “you well know”
and “you also know” about bears’ honey

Master the New SAT®


Chapter 2: Diagnostic Test 85

answers Diagnostic Test


15. The correct answer is (A). Jefferson com- children, so choice (B) is incorrect; nor does
pares conscience to a physical limb of the it show anything about Peter’s intelligence,
body to show that it is natural and present choice (D).
in all human beings, choice (A). Choices
21. The correct answer is (A). The correct
(B), (C), and (D) may be true, but Jefferson
answer is (A). The letter shows Jefferson
only covers the first point; that morality is
to be very involved in ensuring that his
as natural as an arm or leg, and is given to
nephew gets the best education; he says he’s
all “in a stronger or weaker degree” (lines
glad Peter will get to study with Jefferson’s
58–59).
own mentor, Wythe, who he says greatly
16. The correct answer is (D). In advising Peter influenced his life. There’s no indication of
about language studies, Jefferson tells him preoccupation, choice (D) or worry, choice
to learn Spanish (lines 34–39). He predicts (B). The letter is personal, not objective (C).
that it will be valuable in the future because
22. The correct answer is (B). Microplastics
of “connections with Spain and Spanish
are the most abundant type in the ocean
America”, choice (D). He dismisses Italian,
(lines 22–27), choice (B). Nurdles, choice
choice (C), as unworthy of Peter’s attention
(A), are larger plastics that eventually break
and doesn’t discuss America’s future with
down to form microplastics. Microbeads,
France, choice (B), or England, choice (A).
choice (C), and plastic pellets, choice (D),
17. The correct answer is (A). By lost time, are types of microplastics.
Jefferson means wasted time, choice (A). In
23. The correct answer is (D). It cites one of the
his third point, Jefferson tells Peter that he
three key questions scientists are investigat-
shouldn’t bother to study moral philosophy
ing. Because humans are at the top of the food
because it is something everyone knows
chain, scientists believe that the accumula-
intuitively.
tion of plastics in the fish/seafood that we
18. The correct answer is (D). Jefferson claims eat could affect our health. Marine animals
that he has never seen anyone who spoke do consume plastics—choice (C)—but that
French, Spanish, and Italian who didn’t information does not explain the reason for
get them confused, choice (D). All three studying seafood in particular. Choice (A)
languages are derived from Latin, choice explains how plastics in the ocean can infect
(A), as Jefferson notes, but that is not the fish of all kinds, and choice (B) cites how
reason he gives to avoid the study of Italian. plastic enters all parts of the food chain.
Jefferson does think it is necessary to learn Neither of these supports why scientists
a language other than French, choice (B), want to study seafood in particular.
because he recommends learning Spanish.
24. The correct answer is (A). The overall tone
Peter is already multilingual, choice (C)
is neutral, choice (A). The author provides
because Jefferson claims studying Italian
facts and data using scientific terminology
will “confound” his French and Spanish
and explanations. Nowhere does he attempt
(lines 24–26).
to persuade the reader to a particular view,
19. The correct answer is (C). Jefferson says choice (B), inject a personal story, choice
morality is innate, therefore it doesn’t need (C), or offer his own opinion, choice (D),
to be taught. In fact, he argues, teaching it although he does offer differing viewpoints
can have a negative effect of being “led on possible solutions.
astray” (line 64).
25. The correct answer is (B). In lines 101–111,
20. The correct answer is (C). The family tree Halden’s quote shows that he believes
shows that Peter’s father died, choice (C). science can find a way to make plastics
Jefferson would be likely to help his sister biodegradable, choice (B), which he says
raise her family. The text says that Peter will eliminate the problem. He does not say
is Jefferson’s nephew, so although choice they will be safe to ingest, but that a new
(A) is correct, it is not the best answer. The manufacturing process would prevent them
illustration doesn’t show all of Jefferson’s

www.petersons.com
86 PART II: Diagnosing Strengths and Weaknesses

from accumulating and being potentially and (B) are facts that by themselves don’t
hazardous. The author states that news laws show harm. Choice (D) is a part of a direct
may help minimize the problem, choices (A) quote that supports a specific solution; it is
and (D), not Halden. not evidence of the problem.
26. The correct answer is (B). The article 32. The correct answer is (A). Choice (A) is
includes scientific data and explanations of correct: the author suggests ways to help
the effects of plastics in the ocean and wa- reduce the effects of plastics in the envi-
terways, offering information to the public, ronment now through legislation (lines
choice (B). He does not call for the public 91–95), and Halden thinks it is possible
to act in any way, choices (C) and (D), nor to design plastics that don’t harm humans
offer his own opinion about environmental (lines 101–111). Choice (B) states part of
issues, choice (A). the author’s suggestion, but implies that
Halden thinks there’s no solution. There’s
27. The correct answer is (D). In the context of
no indication that Halden thinks the author’s
Halden’s statement, he says he wants to make
position would be burdensome for business,
the problem so small that it will no longer be
choice (C), or that the author thinks we
a threat, choice (D). He acknowledges that it
should cease producing plastics, choice (D).
won’t go away completely (lines 107–109)
since he believes there will always be some 33. The correct answer is (C). The correct
degree of risk of exposure, choice (B). He answer is (C). No one at home asks her
does advocate further study, choice (C), to about the offering once she returns, choices
find a way to make the plastics biodegrad- (A) and (D). Although she has been given
able—but this is unrelated to what he means specific instructions, choice (B), there is
by the word moot. no indication that performing this task will
lead to other such tasks.
28. The correct answer is (A). Choice (A) is
the only choice that refers specifically to 34. The correct answer is (A). Choice (A) is
the food chain. The other choices may have correct. In this line, the adult is looking back
some bearing on how plastics impact the and comments on the scene; in the rest of the
ocean, but they don’t explain disruption to passage, the child narrates as she remembers
the food chain. the events. The tone, choice (B), doesn’t
change, and the entire passage is told in the
29. The correct answer is (D). The author
past tense, choice (C). The author describes
explains that the law was passed to prevent
the exterior setting in detail and omits details
infiltration of these tiny microbeads into the
about the girl’s home, but no new setting is
water system (lines 40–50), choice (D). The
introduced.
microbeads were ending up in the Great Lakes
and Chicago’s waterways. None of the other 35. The correct answer is (C). Although we
options are offered as a reason for the law. are told that the medicine man had to travel
quite a distance, choice (B), the family didn’t
30. The correct answer is (B). Knowing the
send for him until the medicine woman had
meaning of absorb(ent), choice (C), does
given up hope, choice (C). This indicates
not assist in understanding the meaning of
that they didn’t try to reach him earlier and
adsorb. The words share the same suffix,
thought the medicine woman would be able
but it’s the prefix that makes the word less
to help. This sequence suggests a preference
familiar. Absolve, choice (A), has the same
for the medicine woman, though there is no
prefix as absorb. Adhesive is something that
reason stated or implied.
sticks to something else; so adsorb must
mean when one substance sticks to another, 36. The correct answer is (B). It shows the girl
choice (D). contemplating how to follow the specific
instructions (the spot must be “naked earth”)
31. The correct answer is (C). Choice (C)
when the ground was covered with snow.
is the best answer because it includes the
Choice (A) describes the instructions given
concept of harm to humans. Choices (A)

Master the New SAT®


Chapter 2: Diagnostic Test 87

answers Diagnostic Test


by the medicine woman; it doesn’t explain 41. The correct answer is (C). The modern
why they were difficult to follow. Choice (C) meaning of thrilled is usually choice (A),
describes the variance in the terrain where but in lines 58–59, the author is talking
she was searching for the right spot, but it about fear and doubt. Choice (B) would be
does not describe any particular conflict. repetitive. Sadness, choice (D), is incorrect
Choice (D) shows the difficulty the girl had because the emotion the narrator feels is
because of her age and size, not the dilemma describing the feeling more importantly as
of the directions. being sudden and intense.
37. The correct answer is (B). Choices (A) 42. The correct answer is (C). Choice (C) uses
and (C) are far broader than the content of the first person to describe the author’s ac-
the excerpt; though both issues are touched tions and feelings as a child. The specificity
upon, they are not themes of the story. Choice of the detail makes the comment seem like
(D) narrowly describes one aspect of the a personal memory. The other options are
story, perhaps a main idea, but not a theme. either less personal descriptions, choice (B),
Choice (B) refers to something that occurred or general observations that could have been
long ago and also suggests the special place made up by any author, choices (A) and (D).
where it is kept in the adult’s memory.
43. The correct answer is (D). In choice (D),
38. The correct answer is (D). Although she Emmet acknowledges his death sentence
meticulously follows the instructions of and declares that he is dying for a cause.
the medicine woman, choice (B), the girl Choice (C) explains why he asked France
still feels as if she failed because in the end for assistance, and choice (B) explains that
her sister dies. She feels guilty, choice (D). he was fighting for his country’s indepen-
There is no indication that either choice (A) dence and freedom from tyranny. In choice
or choice (C) are true. (A), he defends his actions by saying that
he could have stood by and done nothing,
39. The correct answer is (A). The parents seem
given his family’s privileged position, but
oblivious to the natural emotions of a child.
chose instead to defend liberty.
Though they give the narrator responsibility
and take care of her physical needs (lines 44. The correct answer is (B). Choice (B) shows
64–65), they don’t seem to offer comfort or Emmet’s pride in his country and his intent
consolation. The narrator is probably given to secure independence. Neither choice (A),
more responsibility than she wants, which in which he rhetorically asks if he could be
makes choice (B) incorrect. The parents influenced by personal ambition, nor choice
naturally focus on the child likely to die, (B), in which he denies that he is an emis-
but there is no indication they prefer the sary, explains his patriotism. Choice (C) is
younger child, so choice (C) is incorrect. The a statement describing Emmet’s alignment
narrator seems very alone and her parents with an independent government; it is not
display little warmth or affection, making a display of patriotism.
choice (D) incorrect.
45. The correct answer is (D). The Napole-
40. The correct answer is (A). Choice (A) is onic Wars, which lasted for over a decade,
the best summary of the excerpt because continually realigned the monarchies and
it includes the point of view of a Native sovereignties of European nations, choice
American and the idea of loss. Choice (B) (D). These kinds of continual changes are
only narrowly focuses on the girl following likely to make people more responsive to a
the instructions of the medicine woman. broader concept of patriotism; one that ex-
Choice (C) does not capture the concept of tends beyond borders and includes freedom
the narrative and the point of view, and choice from tyranny, wherever it exists. None of
(D) inaccurately implies that the story centers the other options applies to all of Europe
on the reliance of specific religious tenets. and its ideology.

www.petersons.com
88 PART II: Diagnosing Strengths and Weaknesses

46. The correct answer is (C). Emmet loved his 50. The correct answer is (C). All the op-
country and declared his patriotism toward tions refer to patriotism, but choice (A) is
Ireland as his reason for his actions, choice mostly about perseverance. Choice (B) is
(C). Wright’s view was more inclusive, speculating on how Americans might react
defining it as freeing all people, whatever if attacked and suggests submission as one
the country. Emmet’s primary focus was possibility. Choice (D) is a generic, patriotic
on his own people, choice (B), not all of sentiment. Choice (C) sets up the either-or
humankind. He defended his actions, choice scenario in which there can only be one of
(A), but this action is unrelated to Wright’s two outcomes: liberty or death. Emmet’s
concept of a patriot, as is a court that renders statement is analogous because there are
an unjust sentence, choice (D). only two choices: if the enemy wishes to
come into his homeland (symbolized by
47. The correct answer is (B). Noting the date
the word threshold), he will defend it to the
of Emmet’s speech in his defense of his ac-
death.
tions prior to his execution, choice (A) is
not possible. The Louisiana Purchase by the 51. The correct answer is (B). Wright says
United States, choice (D), would have given Americans’ idea of a patriot is someone
France more resources to help. However, the who loves his/her country, choice (D), but
Napoleonic Wars were draining the French that idea is too narrow. She explains that
treasury, choice (B), leaving the country Europeans see patriotism as a more expan-
overburdened and uninterested in becoming sive concept that extends to freedom for all
involved elsewhere. The French Revolution humans. This idea, she explains, includes
had started in 1789, triggering the rise of working toward the best interests of all hu-
Napoleon, but this was not directly related man lives, wherever they are, so that they
to its refusal to aid Emmet. are free from despotism. This statement
suggests that she herself holds these ideas,
48. The correct answer is (B). Although both
choice (B), and wants others to consider
Emmet and Wright wanted freedom from
them. Physically fighting for one’s country,
tyranny, Emmet’s focus was on Ireland and
choices (A) and (C) is an example of vanity
Wright had a broader objective, choice (B).
Wright was focused on advancing an expan- 52. The correct answer is (B). In this context,
sive concept of patriotism that included other an emissary is a spy or an operative who
countries, which does not mean that she did is doing secret work for another country,
not love her country, choice (C), or did not choice (B), which is why the government
appreciate living in a democratic country, is upset. The government wouldn’t accuse
choice (D). She wanted everyone to have someone of being an ambassador—an of-
the same freedom. ficial position as a representative, choice (A).
Similarly, a minister is an official head of a
49. The correct answer is (B). Choice (D) does
government department (C). A mercenary is
not describe the tone of their speeches. A
a paid soldier; there is nothing that suggests
speech can be both logical and emotional.
the government thinks Emmet is a “hired
Wright does not show any anger, choice (A).
gun.”
It is Emmet who is passionate and angry and
Wright who is calm and reasonable, choice
(B), not the other way around, choice (C).

Master the New SAT®

You might also like